Sunteți pe pagina 1din 50

G.R. No.

179987               September 3, 2013 On February 23, 2007, the CA promulgated its decision reversing the RTC
and dismissing the application for registration of Malabanan. Citing the ruling
in Republic v. Herbieto (Herbieto),4 the CA declared that under Section 14(1)
HEIRS OF MARIO MALABANAN, (Represented by Sally A.
of the Property Registration Decree, any period of possession prior to the
Malabanan), Petitioners,
classification of the land as alienable and disposable was inconsequential and
vs.
should be excluded from the computation of the period of possession. Noting
REPUBLIC OF THE PHILIPPINES, Respondent.
that the CENRO-DENR certification stated that the property had been
declared alienable and disposable only on March 15, 1982, Velazco’s
RESOLUTION possession prior to March 15, 1982 could not be tacked for purposes of
computing Malabanan’s period of possession.
For our consideration and resolution are the motions for reconsideration of the
parties who both assail the decision promulgated on April 29, 2009, whereby Due to Malabanan’s intervening demise during the appeal in the CA, his heirs
we upheld the ruling of the Court of Appeals (CA) denying the application of elevated the CA’s decision of February 23, 2007 to this Court through a
the petitioners for the registration of a parcel of land situated in Barangay petition for review on certiorari.
Tibig, Silang, Cavite on the ground that they had not established by sufficient
evidence their right to the registration in accordance with either Section 14(1)
The petitioners assert that the ruling in Republic v. Court of Appeals and
or Section 14(2) of Presidential Decree No. 1529 (Property Registration
Corazon Naguit5 (Naguit) remains the controlling doctrine especially if the
Decree).
property involved is agricultural land. In this regard, Naguit ruled that any
possession of agricultural land prior to its declaration as alienable and
Antecedents disposable could be counted in the reckoning of the period of possession to
perfect title under the Public Land Act (Commonwealth Act No. 141) and the
Property Registration Decree. They point out that the ruling in Herbieto, to the
The property subject of the application for registration is a parcel of land effect that the declaration of the land subject of the application for registration
situated in Barangay Tibig, Silang Cavite, more particularly identified as Lot as alienable and disposable should also date back to June 12, 1945 or earlier,
9864-A, Cad-452-D, with an area of 71,324-square meters. On February 20, was a mere obiter dictum considering that the land registration proceedings
1998, applicant Mario Malabanan, who had purchased the property from therein were in fact found and declared void ab initio for lack of publication of
Eduardo Velazco, filed an application for land registration covering the the notice of initial hearing.
property in the Regional Trial Court (RTC) in Tagaytay City, Cavite, claiming
that the property formed part of the alienable and disposable land of the public
domain, and that he and his predecessors-in-interest had been in open, The petitioners also rely on the ruling in Republic v. T.A.N. Properties,
continuous, uninterrupted, public and adverse possession and occupation of Inc.6 to support their argument that the property had been ipso jure converted
the land for more than 30 years, thereby entitling him to the judicial into private property by reason of the open, continuous, exclusive and
confirmation of his title.1 notorious possession by their predecessors-in-interest of an alienable land of
the public domain for more than 30 years. According to them, what was
essential was that the property had been "converted" into private property
To prove that the property was an alienable and disposable land of the public through prescription at the time of the application without regard to whether
domain, Malabanan presented during trial a certification dated June 11, 2001 the property sought to be registered was previously classified as agricultural
issued by the Community Environment and Natural Resources Office land of the public domain.
(CENRO) of the Department of Environment and Natural Resources (DENR),
which reads:
As earlier stated, we denied the petition for review on certiorari because
Malabanan failed to establish by sufficient evidence possession and
This is to certify that the parcel of land designated as Lot No. 9864 Cad 452- occupation of the property on his part and on the part of his predecessors-in
D, Silang Cadastre as surveyed for Mr. Virgilio Velasco located at Barangay interest since June 12, 1945, or earlier.
Tibig, Silang, Cavite containing an area of 249,734 sq. meters as shown and
described on the Plan Ap-04-00952 is verified to be within the Alienable or
Disposable land per Land Classification Map No. 3013 established under Petitioners’ Motion for Reconsideration
Project No. 20-A and approved as such under FAO 4-1656 on March 15,
1982.2
In their motion for reconsideration, the petitioners submit that the mere
classification of the land as alienable or disposable should be deemed
After trial, on December 3, 2002, the RTC rendered judgment granting sufficient to convert it into patrimonial property of the State. Relying on the
Malabanan’s application for land registration, disposing thusly: rulings in Spouses De Ocampo v. Arlos,7 Menguito v. Republic8 and Republic
v. T.A.N. Properties, Inc.,9 they argue that the reclassification of the land as
alienable or disposable opened it to acquisitive prescription under the Civil
WHEREFORE, this Court hereby approves this application for registration Code; that Malabanan had purchased the property from Eduardo Velazco
and thus places under the operation of Act 141, Act 496 and/or P.D. 1529, believing in good faith that Velazco and his predecessors-in-interest had been
otherwise known as Property Registration Law, the lands described in Plan the real owners of the land with the right to validly transmit title and
Csd-04-0173123-D, Lot 9864-A and containing an area of Seventy One ownership thereof; that consequently, the ten-year period prescribed by
Thousand Three Hundred Twenty Four (71,324) Square Meters, as supported Article 1134 of the Civil Code, in relation to Section 14(2) of the Property
by its technical description now forming part of the record of this case, in Registration Decree, applied in their favor; and that when Malabanan filed the
addition to other proofs adduced in the name of MARIO MALABANAN, application for registration on February 20, 1998, he had already been in
who is of legal age, Filipino, widower, and with residence at Munting Ilog, possession of the land for almost 16 years reckoned from 1982, the time when
Silang, Cavite. the land was declared alienable and disposable by the State.

Once this Decision becomes final and executory, the corresponding decree of The Republic’s Motion for Partial Reconsideration
registration shall forthwith issue.

The Republic seeks the partial reconsideration in order to obtain a clarification


SO ORDERED.3 with reference to the application of the rulings in Naguit and Herbieto.

The Office of the Solicitor General (OSG) appealed the judgment to the CA, Chiefly citing the dissents, the Republic contends that the decision has
arguing that Malabanan had failed to prove that the property belonged to the enlarged, by implication, the interpretation of Section 14(1) of the Property
alienable and disposable land of the public domain, and that the RTC erred in Registration Decree through judicial legislation. It reiterates its view that an
finding that he had been in possession of the property in the manner and for applicant is entitled to registration only when the land subject of the
the length of time required by law for confirmation of imperfect title. application had been declared alienable and disposable since June 12, 1945 or
earlier.
Ruling Disposition of alienable public lands

We deny the motions for reconsideration. Section 11 of the Public Land Act (CA No. 141) provides the manner by
which alienable and disposable lands of the public domain, i.e., agricultural
lands, can be disposed of, to wit:
In reviewing the assailed decision, we consider to be imperative to discuss the
different classifications of land in relation to the existing applicable land
registration laws of the Philippines. Section 11. Public lands suitable for agricultural purposes can be disposed of
only as follows, and not otherwise:
Classifications of land according to ownership
(1) For homestead settlement;
(2) By sale;
Land, which is an immovable property, 10 may be classified as either of public
(3) By lease; and
dominion or of private ownership.11 Land is considered of public dominion if
(4) By confirmation of imperfect or incomplete titles;
it either: (a) is intended for public use; or (b) belongs to the State, without
(a) By judicial legalization; or
being for public use, and is intended for some public service or for the
(b) By administrative legalization (free patent).
development of the national wealth.12 Land belonging to the State that is not
of such character, or although of such character but no longer intended for
public use or for public service forms part of the patrimonial property of the
State.13 Land that is other than part of the patrimonial property of the State, The core of the controversy herein lies in the proper interpretation of Section
provinces, cities and municipalities is of private ownership if it belongs to a 11(4), in relation to Section 48(b) of the Public Land Act, which expressly
private individual. requires possession by a Filipino citizen of the land since June 12, 1945, or
earlier, viz:
Pursuant to the Regalian Doctrine (Jura Regalia), a legal concept first
introduced into the country from the West by Spain through the Laws of the Section 48. The following-described citizens of the Philippines, occupying
Indies and the Royal Cedulas,14 all lands of the public domain belong to the lands of the public domain or claiming to own any such lands or an interest
State.15 This means that the State is the source of any asserted right to therein, but whose titles have not been perfected or completed, may apply to
ownership of land, and is charged with the conservation of such patrimony.16 the Court of First Instance of the province where the land is located for
confirmation of their claims and the issuance of a certificate of title thereafter,
under the Land Registration Act, to wit:
All lands not appearing to be clearly under private ownership are presumed to
belong to the State. Also, public lands remain part of the inalienable land of
the public domain unless the State is shown to have reclassified or alienated xxxx
them to private persons.17
(b) Those who by themselves or through their predecessors-in-interest have
Classifications of public lands according to alienability been in open, continuous, exclusive, and notorious possession and occupation
of alienable and disposable lands of the public domain, under a bona fide
Whether or not land of the public domain is alienable and disposable primarily claim of acquisition of ownership, since June 12, 1945, or earlier, immediately
rests on the classification of public lands made under the Constitution. Under preceding the filing of the applications for confirmation of title, except when
the 1935 Constitution,18 lands of the public domain were classified into three, prevented by war or force majeure. These shall be conclusively presumed to
namely, agricultural, timber and mineral.19 Section 10, Article XIV of the have performed all the conditions essential to a Government grant and shall be
1973 Constitution classified lands of the public domain into seven, entitled to a certificate of title under the provisions of this chapter. (Bold
specifically, agricultural, industrial or commercial, residential, resettlement, emphasis supplied)
mineral, timber or forest, and grazing land, with the reservation that the law
might provide other classifications. The 1987 Constitution adopted the Note that Section 48(b) of the Public Land Act used the words "lands of the
classification under the 1935 Constitution into agricultural, forest or timber, public domain" or "alienable and disposable lands of the public domain" to
and mineral, but added national parks.20 Agricultural lands may be further clearly signify that lands otherwise classified, i.e., mineral, forest or timber, or
classified by law according to the uses to which they may be devoted.21 The national parks, and lands of patrimonial or private ownership, are outside the
identification of lands according to their legal classification is done coverage of the Public Land Act. What the law does not include, it excludes.
exclusively by and through a positive act of the Executive Department.22 The use of the descriptive phrase "alienable and disposable" further limits the
coverage of Section 48(b) to only the agricultural lands of the public domain
Based on the foregoing, the Constitution places a limit on the type of public as set forth in Article XII, Section 2 of the 1987 Constitution. Bearing in mind
land that may be alienated. Under Section 2, Article XII of the 1987 such limitations under the Public Land Act, the applicant must satisfy the
Constitution, only agricultural lands of the public domain may be alienated; following requirements in order for his application to come under Section
all other natural resources may not be. 14(1) of the Property Registration Decree,28 to wit:

Alienable and disposable lands of the State fall into two categories, to wit: (a) 1. The applicant, by himself or through his predecessor-in-interest,
patrimonial lands of the State, or those classified as lands of private ownership has been in possession and occupation of the property subject of
under Article 425 of the Civil Code,23 without limitation; and (b) lands of the the application;
public domain, or the public lands as provided by the Constitution, but with 2. The possession and occupation must be open, continuous,
the limitation that the lands must only be agricultural. Consequently, lands exclusive, and notorious;
classified as forest or timber, mineral, or national parks are not susceptible of 3. The possession and occupation must be under a bona fide claim
alienation or disposition unless they are reclassified as agricultural. 24 A of acquisition of ownership;
positive act of the Government is necessary to enable such 4. The possession and occupation must have taken place since June
reclassification,25 and the exclusive prerogative to classify public lands under 12, 1945, or earlier; and
existing laws is vested in the Executive Department, not in the courts. 26 If, 5. The property subject of the application must be an agricultural
however, public land will be classified as neither agricultural, forest or timber, land of the public domain.
mineral or national park, or when public land is no longer intended for public
service or for the development of the national wealth, thereby effectively
removing the land from the ambit of public dominion, a declaration of such Taking into consideration that the Executive Department is vested with the
conversion must be made in the form of a law duly enacted by Congress or by authority to classify lands of the public domain, Section 48(b) of the Public
a Presidential proclamation in cases where the President is duly authorized by Land Act, in relation to Section 14(1) of the Property Registration Decree,
law to that effect.27 Thus, until the Executive Department exercises its presupposes that the land subject of the application for registration must have
prerogative to classify or reclassify lands, or until Congress or the President been already classified as agricultural land of the public domain in order for
declares that the State no longer intends the land to be used for public service the provision to apply. Thus, absent proof that the land is already classified as
or for the development of national wealth, the Regalian Doctrine is applicable. agricultural land of the public domain, the Regalian Doctrine applies, and
overcomes the presumption that the land is alienable and disposable as laid private land of the State, the applicable provision concerning disposition and
down in Section 48(b) of the Public Land Act. However, emphasis is placed registration is no longer Section 48(b) of the Public Land Act but the Civil
on the requirement that the classification required by Section 48(b) of the Code, in conjunction with Section 14(2) of the Property Registration
Public Land Act is classification or reclassification of a public land as Decree.35 As such, prescription can now run against the State.
agricultural.
To sum up, we now observe the following rules relative to the disposition of
The dissent stresses that the classification or reclassification of the land as public land or lands of the public domain, namely:
alienable and disposable agricultural land should likewise have been made on
June 12, 1945 or earlier, because any possession of the land prior to such
(1) As a general rule and pursuant to the Regalian Doctrine, all
classification or reclassification produced no legal effects. It observes that the
lands of the public domain belong to the State and are inalienable.
fixed date of June 12, 1945 could not be minimized or glossed over by mere
Lands that are not clearly under private ownership are also
judicial interpretation or by judicial social policy concerns, and insisted that
presumed to belong to the State and, therefore, may not be
the full legislative intent be respected.
alienated or disposed;

We find, however, that the choice of June 12, 1945 as the reckoning point of
(2) The following are excepted from the general rule, to wit:
the requisite possession and occupation was the sole prerogative of Congress,
the determination of which should best be left to the wisdom of the
lawmakers. Except that said date qualified the period of possession and (a) Agricultural lands of the public domain are rendered
occupation, no other legislative intent appears to be associated with the fixing alienable and disposable through any of the exclusive
of the date of June 12, 1945. Accordingly, the Court should interpret only the modes enumerated under Section 11 of the Public Land
plain and literal meaning of the law as written by the legislators. Act. If the mode is judicial confirmation of imperfect
title under Section 48(b) of the Public Land Act, the
agricultural land subject of the application needs only to
Moreover, an examination of Section 48(b) of the Public Land Act indicates
be classified as alienable and disposable as of the time
that Congress prescribed no requirement that the land subject of the
of the application, provided the applicant’s possession
registration should have been classified as agricultural since June 12, 1945, or
and occupation of the land dated back to June 12, 1945,
earlier. As such, the applicant’s imperfect or incomplete title is derived only
or earlier. Thereby, a conclusive presumption that the
from possession and occupation since June 12, 1945, or earlier. This means
applicant has performed all the conditions essential to a
that the character of the property subject of the application as alienable and
government grant arises,36 and the applicant becomes
disposable agricultural land of the public domain determines its eligibility for
the owner of the land by virtue of an imperfect or
land registration, not the ownership or title over it.
incomplete title. By legal fiction, the land has already
ceased to be part of the public domain and has become
Alienable public land held by a possessor, either personally or through his private property.37
predecessors-in-interest, openly, continuously and exclusively during the
prescribed statutory period is converted to private property by the mere lapse
(b) Lands of the public domain subsequently classified
or completion of the period.29 In fact, by virtue of this doctrine, corporations
or declared as no longer intended for public use or for
may now acquire lands of the public domain for as long as the lands were
the development of national wealth are removed from
already converted to private ownership, by operation of law, as a result of
the sphere of public dominion and are considered
satisfying the requisite period of possession prescribed by the Public Land
converted into patrimonial lands or lands of private
Act.30 It is for this reason that the property subject of the application of
ownership that may be alienated or disposed through
Malabanan need not be classified as alienable and disposable agricultural land
any of the modes of acquiring ownership under the
of the public domain for the entire duration of the requisite period of
Civil Code. If the mode of acquisition is prescription,
possession.
whether ordinary or extraordinary, proof that the land
has been already converted to private ownership prior
To be clear, then, the requirement that the land should have been classified as to the requisite acquisitive prescriptive period is a
alienable and disposable agricultural land at the time of the application for condition sine qua non in observance of the law (Article
registration is necessary only to dispute the presumption that the land is 1113, Civil Code) that property of the State not
inalienable. patrimonial in character shall not be the object of
prescription.
The declaration that land is alienable and disposable also serves to determine
the point at which prescription may run against the State. The imperfect or To reiterate, then, the petitioners failed to present sufficient evidence to
incomplete title being confirmed under Section 48(b) of the Public Land Act establish that they and their predecessors-in-interest had been in possession of
is title that is acquired by reason of the applicant’s possession and occupation the land since June 12, 1945. Without satisfying the requisite character and
of the alienable and disposable agricultural land of the public domain. Where period of possession - possession and occupation that is open, continuous,
all the necessary requirements for a grant by the Government are complied exclusive, and notorious since June 12, 1945, or earlier - the land cannot be
with through actual physical, open, continuous, exclusive and public considered ipso jure converted to private property even upon the subsequent
possession of an alienable and disposable land of the public domain, the declaration of it as alienable and disposable. Prescription never began to run
possessor is deemed to have acquired by operation of law not only a right to a against the State, such that the land has remained ineligible for registration
grant, but a grant by the Government, because it is not necessary that a under Section 14(1) of the Property Registration Decree. Likewise, the land
certificate of title be issued in order that such a grant be sanctioned by the continues to be ineligible for land registration under Section 14(2) of the
courts.31 Property Registration Decree unless Congress enacts a law or the President
issues a proclamation declaring the land as no longer intended for public
service or for the development of the national wealth.1âwphi1
If one follows the dissent, the clear objective of the Public Land Act to
adjudicate and quiet titles to unregistered lands in favor of qualified Filipino
citizens by reason of their occupation and cultivation thereof for the number WHEREFORE, the Court DENIES the petitioners' Motion for
of years prescribed by law32 will be defeated. Indeed, we should always bear Reconsideration and the respondent's Partial Motion for Reconsideration for
in mind that such objective still prevails, as a fairly recent legislative their lack of merit.
development bears out, when Congress enacted legislation (Republic Act No.
10023)33 in order to liberalize stringent requirements and procedures in the
SO ORDERED.
adjudication of alienable public land to qualified applicants, particularly
residential lands, subject to area limitations.34

On the other hand, if a public land is classified as no longer intended for


public use or for the development of national wealth by declaration of
Congress or the President, thereby converting such land into patrimonial or
petitioners Sps. Valenciano halt construction of said building, but the latter
refused to do so. Hence, he filed the instant case.

Petitioners Sps. Valenciano, on the other hand, claimed that the ongoing
construction was with the consent and conformity of the DPWH and in fact
the improvements found in the property were introduced by the residents
thereof, including its first residents, William and Gloria Banuca, and not by
respondent. The premises on which petitioners Sps. Valenciano are
constructing their house were leveled after the earthquake in 1990 by the
Banuca spouses. Petitioners Sps. Valenciano are just starting the construction
because the permission was only given now by Gloria Banuca.5

On 18 August 1998, petitioner filed before the MCTC of Tuba, Benguet, a


complaint for Forcible Entry with Prayer for Preliminary Mandatory
Injunction with Application for Issuance of a Temporary Restraining Order
plus Damages.6 The complaint was amended on 27 August 1998.7 Per Order
dated 19 August 1998, a Temporary Restraining Order (TRO) was issued
ordering respondents to desist and cease and refrain from continuing the
construction of a house on the land in question.8

On 27 August 1998, respondent spouses Juanito and Amalia Valenciano filed


their Answer with Opposition to the Prayer for Issuance of Writ of
Preliminary Injunction.9 On 07 September 1998, they filed an Answer to the
Amended Complaint10 to which petitioner filed a Reply.11

On 15 September 1998, the MCTC issued another TRO.12

The pre-trial order dated 6 November 1998 contained, among other things,
petitioner's admission that he was temporarily not operating any business in
the area, and respondents' admission regarding the issuance of Tax
Declarations on the property in dispute in petitioner's name.13

[G.R. NO. 158687 - January 27, 2006] Trial ensued. Petitioner presented Mariano Suyam and Tonsing Binay-an, two
of his former truck drivers from 1981 to 1985 in his business of hauling sand,
gravel and other aggregates at Riverside, Camp 3, Tuba, Benguet.
FRISCO F. DOMALSIN, Petitioner, v. SPOUSES JUANITO
VALENCIANO and AMALIA VALENCIANO, Respondents.
Mariano Suyam testified that sometime in 1981, petitioner caused the
construction of a private road leading to the Bued River from Kennon Road.
DECISION He added that petitioner constructed two houses, the first was located along
the road-right-of-way of Kennon Road where respondents are now
Before Us is a Petition for Review which seeks to set aside the decision1 of the constructing their house, while the second was located below the private road
Court of Appeals in CA-G.R. SP No. 69415 dated 20 August 2002 which around 40 to 60 meters down from Kennon Road. He explained that the first
reversed and set aside the decision2 of Branch 63 of the Regional Trial Court house was used for sleeping quarters and resting center for laborers, while
(RTC) of La Trinidad, Benguet, in Civil Case No. 01-CV-1582(150) dated 23 petitioner used the second one as his quarters. He said William Banuca was
January 2002, which affirmed the decision3 of the Municipal Circuit Trial hired as foreman in 1983 and that the latter and his family stayed in the
Court (MCTC) of Tuba-Sablan, Tuba, Benguet, in Civil Case No. 150 dated second house.
20 November 2000, declaring petitioner Frisco F. Domalsin the actual
possessor of the lot in dispute and ordering, inter alia, respondent spouses Tonsing Binay-an corroborated the testimony of Suyam as regards the two
Juanito and Amalia Valenciano to vacate and deliver the physical possession houses constructed by petitioner and added that petitioner was the manager of
thereof to the former, and its Resolution4 dated 20 May 2003 denying Salamander Enterprises and had a concession permit from the Bureau of
petitioner's motion for reconsideration. Mines to haul gravel and sand.

The respective allegations of the parties as contained in the complaint and Petitioner testified that he is a lawyer-businessman formerly engaged in
answer are substantially summarized by the Court of Appeals as follows: trucking business, hauling sand and gravel, and operated under the name
Salamander Enterprises.14 He narrated that while he was passing Kennon
The property subject of this action for forcible entry is a parcel of land located Road, he discovered that a portion of the Bued River, Camp 3, Tuba Benguet,
at sitio Riverside, Camp 3, Tuba, Benguet. Respondent Frisco B. Domalsin can be a potential source of supplies for his business. Though the area was
claims to be the lawful owner and possessor of said parcel of land since 1979 steep and deep, he scouted a place where he can construct a road from Kennon
up to the present. He declared it for taxation purposes in 1983 as (per) Tax Road to the Bued River. In the course of cleaning the area, his workers
Declaration No. 9540 issued on September 12, 1983 by the Municipal noticed that the place had been tilled. A certain Castillo Binay-an appeared
Assessor of Tuba Benguet. He allegedly introduced improvements consisting informing him that he was the occupant of the site of the proposed private
of levelling, excavation, riprapping of the earth and a private road to the river, road. After agreeing on the consideration, the former executed a Deed of
fruitbearing trees and other agricultural plants of economic value. He was in Waiver and Quitclaim15 over the land in his favor.
continuous, adverse possession and in the concept of an owner for the past
nineteen (19) years. Thereafter, the Office of the Highway District Engineer of Baguio, Ministry
of Public Highways (now Department of Public Works and Highways
On August 1, 1998, petitioners Spouses Juanito Valenciano and Amalia [DPWH]) issued a permit in favor of petitioner to extract construction
Valenciano (Sps. Valenciano, for brevity) allegedly entered the premises to materials at Camp 3, Tuba, Benguet,16 which was followed by the issuance on
construct a building made of cement and strong materials, without the 1 October 1981 of Commercial Permit No. 147 by the Office of the Mines
authority and consent of respondent, by means of force and strategy, and Regional Officer, Mineral Region No. 1, Bureau of Mines and Geo-Sciences
without a building permit from the Department of Public Works and (Bureau of Mines).17 The Commercial Permit, which was renewable every
Highways (DPWH, for brevity). Respondent protested and demanded that year, was last renewed in 1987.18
Based on the Deed of Waiver and Quitclaim executed by Castillo Binay-an, Adriano Jularbal to Gloria Banuca of improvements found near the house of
petitioner was able to apply for, and was issued, a tax declaration over the the latter in the amount of P1,000.00.
land covering one hectare. Tax Declaration No. 954019 dated 12 September
1983 was issued to petitioner describing the land bounded on the North by
The MCTC found that what is being contested is the possession of a portion
Bued River, on the South by Kennon Road, on the East by Kennon Road, and
of the road-right-of way of Kennon Road which is located in front of a parcel
on the West by a Creek. With the revision of the fair market value and
of land that petitioner bought by way of Deed of Waiver and Quitclaim from
assessed value of lands, Tax Declaration No. 94-004-00327 dated 12
Castillo Binay-an. It held that petitioner had prior material possession over the
November 1994 was issued to him.20 From 1983 up to 1998, petitioner has
subject land. It ruled that the destruction of his house built thereon by the
been regularly paying real property taxes over the land.
earthquake in 1990, and later cannibalized without being reconstructed was
not tantamount to abandonment of the site by the petitioner because it was
Petitioner disclosed that in 1983, William Banuca applied for, and was destroyed by a fortuitous event which was beyond his control. It explained
accepted, as foreman.21 Due to the nature of his job, Banuca was permitted to that his possession over the land must be recognized by respondents who
stay in the second house beside the private road.22 Banuca now lives came later after the earthquake. It brushed aside respondents' allegation that
permanently in said house after petitioner gave it to him. Petitioner revealed the land in dispute was abandoned by the latter after he stopped operating his
that the houses his former laborers constructed were awarded to them as a sand and gravel business in 1985 and never returned anymore, and when the
kind gesture to them. As to the land he occupied along the Kennon Road house erected on it was destroyed during the 1990 earthquake, it was no
where the first house was erected, he claims that same still belongs to him. longer reconstructed and was subsequently leveled or demolished by Gloria
This house, which his laborers and drivers used as a resting area, was Banuca. However, it pronounced that respondents' action to occupy the land
cannibalized and leveled, and the land over which it once stood was taken was done in good faith considering that their occupation of the land was with
possession by respondents who are now building their house thereon. the assurance of the seller (Gloria Banuca) and that they were armed with the
permit issued by the DPWH for him to construct his house thereon.
Gloria Banuca testified for respondents. She disclosed that it was she who
invited respondents to come and reside at Riverside, Camp 3, Tuba, Benguet. On 20 November 2000, the MCTC came out with its decision, the decretal
She said she knew petitioner to be engaged in the sand and gravel business in portion of which reads:
Tuba, Benguet, from 1981 to 1985, and that the latter stopped in 1985 and
never returned to haul sand and gravel at the Bued River. She claimed she
WHEREFORE PREMISES CONSIDERED, decision is hereby rendered in
never saw petitioner introduce any improvements on the land he claimed he
favor of plaintiff, FRISCO DOMALSIN, and against defendants, JUANITO
bought from Castillo Binay-an, and that it was she and the other residents who
VALENCIANO and AMALIA VALENCIANO, with the following:
introduced the existing improvements.

1. Order to declare the injunction permanent.


She narrated that in 1983, she planted fruit-bearing trees in the area where
respondents were constructing their house which is located along the Kennon
Road's road-right-of-way, fronting petitioner's property. After the earthquake 2. Order the plaintiff as the actual possessor of the lot in question.
of 1990, the private road constructed by petitioner became impassable and it
was she who hired the equipment used to clear the same. She even leveled the
3. Order the defendant(s) to vacate and deliver the physical possession
area where respondents were building their home. Based on the ocular
voluntarily of the disputed land to plaintiff within 60 days from receipt of this
inspection, she said this area is within the 15-meter radius from the center of
decision.
the road. This area, she claims, was sold to her by the Spouses Jularbal.
However, the agreement between them shows that what was sold to her were
the improvements near her house which was 40 meters down from Kennon 4. Order defendant(s) to remove his structure within from receipt of this
Road and the improvements along Kennon Road.23 decision.

Agustin Domingo next testified for respondents. He testified that in 1986, 5. Order the defendant(s) to (sic) plaintiff the amount of P10,000.00, as
upon the invitation of Gloria Banuca, he transferred his residence to sitio litigation expenses.
Riverside because of its proximity to his place of work. He stayed there for
good and even buried his father near his house. He said that in 1990, the
private road constructed by petitioner was covered by boulders, soil and rocks, 6. Order defendant(s) to pay the cost of suit26
and it was Mrs. Banuca who initiated the clearing of the road. Finally, he
declared that since 1986, he never saw petitioner introduce any improvement Respondents appealed the decision to the RTC.27 In affirming the decision in
in the area. toto the RTC ratiocinated:

Respondent Juanito Valenciano revealed that he is the cousin of Gloria It may be well to consider that even after plaintiff's business ceased operation,
Banuca. He narrated that in 1984, he went to Riverside to see the latter whose he religiously paid the taxes due thereon.
husband, William Banuca, was working as foreman of petitioner. At that time,
the lot under litigation was still a hill. It was Gloria Banuca who leveled the
hill and told him to construct his house there. Finding the place to be an ideal Appellant's theory that the plaintiff-appellee abandoned the property does not
place to build his house, he paid the Banucas P10,000.00 for the sit well and finds no support in the record. Notice that since 1985 up to mid-
improvements. 1990, the Banucas never laid claim over the property taking into consideration
that they were already residents of the place. This only goes to show that they
acknowledged and respected the prior possession of the plaintiff-appellee.
He explained that before he started building his house, he sought the Besides, what right has Gloria to cause the leveling of the property destroying
permission of the Benguet District Engineer, DPWH, which the latter granted. the natural contour thereof, to presume that plaintiff-appellee abandoned it
In August 1998, he received a notice24 to stop and desist from continuing the and to invite and allow other persons to settle thereat? Absolutely none.
construction of a permanent one-storey house made of hollow blocks and Knowing fully well that the plaintiff-appellee has prior possession of the
cement since the condition was only to utilize light materials. Thereafter, a property, Gloria's actions are unjustified, to say the least. Her consummated
letter dated 22 January 1999 was sent to him informing him that the temporary act of leveling the property without the knowledge of the plaintiff-appellee is
permit issued to him for the improvement/utilization of a portion of the viewed as a test to determine whether or not the latter is still interested in the
national road along Kennon Road had been revoked for non-submission of the property. From then on until 1998 (but before the construction), the Banucas
waiver as required by the Office of the District Engineer and his non- still recognize the plaintiff's possession. But as Gloria claims to have heard no
compliance with the condition that no permanent structures are to be word from the plaintiff, she unilaterally declared that the place is now
constructed within the road-right-of-way. He, however, denied receiving said abandoned as she "invited and allowed" the defendants to live and construct
letter. their house thereat.

Juan de Vera, a retired DPWH foreman, testified last for the respondents. He Contrary to the assertion of the appellants, there was no abandonment simply
claimed he witnessed the execution of the document25 regarding the sale by because plaintiff-appellee continuously paid the corresponding taxes due
thereon and that he promptly objected to the construction of the defendants- forcible entry against petitioners Sps. Valenciano, which he should have filed
appellants' house. These are clear manifestations of his intention not to against Gloria Banuca, petitioners Sps. Valenciano's predecessor-in-interest.
abandon the property. Sad to say though that here is a former employer. By But more than a year had passed and his right to do so lapsed. Thus,
passing off such property to be hers is so unkind, unfair and against social respondent's prior possession is material only as against Gloria Banuca and
order. It is very clear that the Banucas knew of the prior possession of the only within a period of one year from the time she wrested possession of the
plaintiff way back then so that they themselves never personally build property from respondent.
construction over the property. If they honestly believe that they now "own"
the land, why will they still have to invite other people who are not their
We view with distate Gloria Banuca's ingratitude toward her husband's former
relatives to settle thereat? Why the preference of strangers over relatives? The
employer. Her actions smack of the proverbial hand being offered in aid but
Court does not believe that they did not receive any compensation for having
the person to whom it is offered would rather have the whole arm instead.
"allowed" strangers, the defendants included, to settle on the land.
This is an instance where it is the employees who commit injustice against
their employer. Nonetheless, petitioners Sps. Valenciano should not suffer
From all the foregoing, Gloria is clearly in bad faith. And her being in bad because of Gloria Banuca's ingratitude for the former came across the
faith must be corrected and if warranted, must be meted appropriate penalty. If property in good faith.
the Banucas are in bad faith, then the appellants cannot have better rights
either. The Banucas transferred nothing to them. Defendants-appellants cannot
But respondent is also reminded that he only has himself to blame. His failure
even be considered as builders in good faith. It must be noted that they were
to assert his right for an unreasonable and unexplained length of time allowed
prohibited by the plaintiff from going further but they ignored it. They shall
Gloria Banuca to wrest possession from him. Especially in this case where
lose what was built (Art. 449, Civil Code). Again, if the Banucas believe that
they do not and cannot own the subject property, actual possession becomes
they have an action or a right to deprive the plaintiff's possession, why did
particularly important.29
they not invoke judicial interference as required under Art. 536 of the same
code? Nonetheless, notwithstanding the fact of leveling without the
knowledge of the plaintiff-appellee, the same did not affect his possession The case was disposed as follows:
(Art. 537, Civil Code).28
WHEREFORE, in view of the foregoing, the petition is GRANTED and the
Via a Petition for Review, respondents appealed to the Court of Appeals. The decision of the Municipal Circuit Trial Court of tuba-Sablan dated November
Court of Appeals made a sudden turn-around and reversed the decision under 20, 2000 as affirmed by the Regional Trial Court on January 23, 2002 is
review. Its decision dated 20 August 2002 reads in part: hereby REVERSED and SET ASIDE.30

[T]here is a need to clarify a few things. What is undisputed are the identity The Motion for Reconsideration filed by petitioner was denied in a
and nature of the property subject of the action for forcible entry. The subject resolution31 dated 20 May 2003.
of the action concerns a portion of the road-right-of-way along Kennon Road
just above the private road constructed by respondent. The problem, however,
Petitioner is now before us seeking redress. He assigns the following as the
is that petitioners Sps. Valenciano started constructing a house on the same
errors committed by the Court of Appeals:
spot where a house belonging to respondent once stood. Both parties are now
asserting that they are entitled to the possession of said lot. But the decision of
the lower court seems to imply that respondent's right to possess the subject I.
property stems from his acquisition of the one-hectare property below it. That
is not the case.
THE HONORABLE COURT OF APPEALS ERRED IN HOLDING THAT
PRIVATE RESPONDENT (NOW PETITIONER) FRISCO DOMALSIN
We must emphasize that the subject of the deed of quitclaim and waiver of ABANDONED THE PROPERTY SUBJECT OF THE LITIGATION.
rights of Castillo Binay-an was not the road-right-of-way but the sloping
terrain below it. This was the property acquired by the respondent to have
access to the sand and gravel on the Bued River. It did not include the road- II.
right-of-way. As regards Gloria Banucas's claims, the evidence show that her
agreement with Jularbal involved only the improvements near her residence THE HONORABLE COURT OF APPEALS ERRED IN REVERSING AND
down the private road and not the road-right-of-way. Since the subject SETTING ASIDE THE DECISION OF THE REGIONAL TRIAL COURT
property is a road-right-of-way, it forms part of the public dominion. It is not OF LA TRINIDAD, BENGUET, BRANCH 63 WHICH AFFIRMED THE
susceptible to private acquisition or ownership. Prolonged occupation thereof, DECISION OF THE MUNICIPAL CIRCUIT TRIAL COURT OF TUBA-
improvements introduced thereat or payment of the realty taxes thereon will SABLAN.
never ripen into ownership of said parcel of land. Thus, what We have are two
parties, neither of which can be owners, only possessors of the subject
property. Beyond these two, only the government has a better right to the At the outset, it must be made clear that the property subject of this case is a
subject property which right it may exercise at any time. This bears portion of the road-right-of way of Kennon Road which is located in front of a
emphasizing because if either party has possessory rights to the subject parcel of land that petitioner bought by way of Deed of Waiver and Quitclaim
property, it is not predicated on ownership but only on their actual possession from Castillo Binay-an.32 The admission33 of petitioner in his Amended
of the subject property. Complaint that respondents started constructing a building within the Kennon
Road road-right-of-way belies his claim that the lot in question is his.

xxxx
In light of this exposition, it is clear that neither the petitioner nor the
respondents can own nor possess the subject property the same being part of
There is no doubt that respondent had prior physical possession of the subject the public dominion. Property of public dominion is defined by Article 420 of
property. He entered and acquired possession of the subject property when he the Civil Code as follows:
built his house thereon. The house was destroyed during the 1990 earthquake
and respondent did not rebuild it. The mound on which it stood was later
leveled by Gloria Banuca and in 1998 petitioners Sps. Valenciano began ART. 420. The following things are property of public dominion:
construction thereat. Petitioners Sps. Valenciano claim there was
abandonment, but the lower court ruled that respondent did not abandon the (1) Those intended for public use such as roads, canals, rivers, torrents, ports
subject property as he continued to pay the realty taxes thereon and objected and bridges constructed by the State, banks, shores, roadsteads, and other of
to petitioners Sps. Valenciano's construction. We believe, and so hold, that at similar character.
this point in time, it is immaterial whether or not there was abandonment by
respondent. The fact remains that Gloria Banuca took possession of the
subject property soon after the earthquake. She leveled the mound and the (2) Those which belong to the State, without being for public use, and are
ruins of respondent's house, yet respondent remained silent. Respondent intended for some public service or for the development of the national
objected only after petitioners Sps. Valenciano started construction of the wealth.
house on the subject property. Respondent cannot now interpose an action for
Properties of public dominion are owned by the general public.34 Public use is Abandonment of a thing is the voluntary renunciation of all rights which a
"use that is not confined to privileged individuals, but is open to the indefinite person may have in a thing, with the intent to lose such thing.40 A thing is
public."35 As the land in controversy is a portion of Kennon Road which is for considered abandoned and possession thereof lost if the spes recuperandi (the
the use of the people, there can be no dispute that same is part of public hope of recovery) is gone and the animus revertendi (the intention of
dominion. This being the case, the parties cannot appropriate the land for returning) is finally given up.41
themselves. Thus, they cannot claim any right of possession over it. This is
clear from Article 530 of the Civil Code which provides:
In the case before us, we find that petitioner never abandoned the subject land.
His opposition to the construction of respondents' house upon learning of the
ART. 530. Only things and rights which are susceptible of being appropriated same and the subsequent filing of the instant case are clear indicia of non-
may be the object of possession. abandonment; otherwise, he could have just allowed the latter to continue with
the construction. Moreover, the fact that the house petitioner built was
destroyed by the earthquake in 1990, was never rebuilt nor repaired and that
Notwithstanding the foregoing, it is proper to discuss the position of the Court
same was leveled to the ground by Gloria Banuca do not signify
of Appeals for comprehensive understanding of the facts and the law
abandonment. Although his house was damaged by the earthquake, Gloria
involved.
Banuca, the person who supposedly demolished said house, had no right to do
the same. Her act of removing the house and depriving petitioner of
Petitioner maintains that the Court of Appeals erred when it ruled that he possession of the land was an act of forcible entry. The entry of respondents in
abandoned the land being disputed contrary to the rulings of the MCTC and 1998 was likewise an act of forcible entry.
RTC. The MCTC found there was no abandonment of the land because the
house erected thereon was destroyed by a fortuitous event (earthquake), while
The next question is: Was the action filed the correct one and was it timely
the RTC ruled there was no abandonment because petitioner paid taxes due on
filed?
the land and that he promptly objected to the construction of respondents'
house which are clear manifestations of his intention not to abandon the
property. Well-settled is the rule that what determines the nature of the action as well as
the court which has jurisdiction over the case are the allegations in the
complaint.42 In actions for forcible entry, the law tells us that two allegations
A reading of the decision of the Court of Appeals shows that it did not reverse
are mandatory for the municipal court to acquire jurisdiction: First, the
the two lower courts on the issue of abandonment. It merely declared that such
plaintiff must allege prior physical possession of the property. Second, he
issue is not material in the resolution of the case at bar. It faulted petitioner for
must also allege that he was deprived of his possession by any of the means
not asserting his right for a long time allowing Gloria Banuca to wrest the
provided for in Section 1, Rule 70 of the Rules of Court. 43 To effect the
possession of the land in question from petitioner by leveling the house he
ejectment of an occupant or deforciant on the land, the complaint should
built thereon and pronounced that actual possession becomes important in a
embody such a statement of facts as to bring the party clearly within the class
case where parties do not and cannot own the land in question.
of cases for which the statutes provide a remedy, as these proceedings are
summary in nature. The complaint must show enough on its face to give the
From the foregoing it appears that the Court of Appeals did not give weight or court jurisdiction without resort to parol evidence.44
importance to the fact that petitioner had prior physical possession over the
subject land. It anchored its decision on the fact that the parties do not and
A look at the Amended Complaint filed by petitioner clearly shows a case for
cannot own the land and that respondents now have actual possession over it.
forcible entry. Petitioner alleged therein that he has been in possession of the
subject land for the last nineteen years and that respondents, in the first week
Ejectment proceedings are summary proceedings intended to provide an of August 1998, without his permission and consent, entered the land by
expeditious means of protecting actual possession or right to possession of means of force, strategy and stealth and started the construction of a building
property. Title is not involved. The sole issue to be resolved is the question as thereon; and upon being informed thereof, he requested them to stop their
to who is entitled to the physical or material possession of the premises or construction but respondents refused to vacate the land forcing him to file the
possession de facto.36 instant case to recover possession thereof.

The Court of Appeals erred when it preferred the present and actual The Court of Appeals pronounced that petitioner cannot interpose an action
possession of respondents vis - à-vis the prior possession of petitioner on the for forcible entry against respondents and that the same should have been filed
ground that the parties do not and cannot own the lot in question. Regardless against Gloria Banuca. It added that the right to file against the latter had
of the actual condition of the title to the property, the party in peaceable, quiet already lapsed because more than a year had passed by from the time she
possession shall not be thrown out by a strong hand, violence or terror. wrestled possession of the property from the petitioner.
Neither is the unlawful withholding of property allowed. Courts will always
uphold respect for prior possession. Thus, a party who can prove prior
We find such pronouncement to be flawed. An action of forcible entry and
possession can recover such possession even against the owner himself.
detainer may be maintained only against one in possession at the
Whatever may be the character of his possession, if he has in his favor prior
commencement of the action, and not against one who does not in fact hold
possession in time, he has the security that entitles him to remain on the
the land.45 Under Section 1,46 Rule 70 of the Rules of Court, the action may be
property until a person with a better right lawfully ejects him.37
filed against persons unlawfully withholding or depriving possession or any
person claiming under them. Considering that respondents are the ones in
The fact that the parties do not and cannot own the property under litigation present actual possession and are depriving petitioner of the possession of the
does not mean that the issue to be resolved is no longer priority of possession. land in question, it is proper that they be the ones to be named defendants in
The determining factor for one to be entitled to possession will be prior the case. The fact that Gloria Banuca was supposedly the one who first
physical possession and not actual physical possession. Since title is never in committed forcible entry when she allegedly demolished the house of
issue in a forcible entry case, the Court of Appeals should have based its petitioner does not make her the proper party to be sued because she is no
decision on who had prior physical possession. The main thing to be proven in longer in possession or control of the land in controversy.
an action for forcible entry is prior possession and that same was lost through
force, intimidation, threat, strategy and stealth, so that it behooves the court to
As regards the timeliness of the filing of the case for forcible entry, we find
restore possession regardless of title or ownership.38
that same was filed within the one-year prescriptive period. We have ruled
that where forcible entry was made clandestinely, the one-year prescriptive
Inasmuch as prior physical possession must be respected, the Court of period should be counted from the time the person deprived of possession
Appeals should have ruled squarely on the issue of abandonment because it demanded that the deforciant desist from such dispossession when the former
gave precedence to the actual present possession of respondents. If, indeed, learned thereof.47 As alleged by petitioner in the Amended Complaint, he was
there was abandonment of the land under consideration by petitioner, only deprived of his possession over the land by force, strategy and stealth.
then should respondents be given the possession of the same since Considering that one of the means employed was stealth because the intrusion
abandonment is one way by which a possessor may lose his possession.39 was done by respondents without his knowledge and consent, the one-year
period should be counted from the time he made the demand to respondents to
vacate the land upon learning of such dispossession. The record shows that
upon being informed that respondents were constructing a building in the
subject land sometime in the first week of August 1998, petitioner
immediately protested and advised the former to stop; but to no avail. The
one-year period within which to file the forcible entry case had not yet expired
when the ejectment suit was filed on 18 August 1998 with the MCTC.

Despite the foregoing findings, this Court finds that the MCTC and the RTC,
as well as the Court of Appeals, to be in error when they respectively declared
that petitioner and respondents to be entitled to the possession of the land in
dispute. The parties should not be permitted to take possession of the land,
G.R. No. L40474 August 29, 1975
much more, claim ownership thereof as said lot is part of the public dominion.

CEBU OXYGEN & ACETYLENE CO., INC., petitioner,


WHEREFORE, the foregoing considered, the instant petition is hereby
vs.
PARTIALLY GRANTED. Nonetheless, there being a finding that the subject
HON. PASCUAL A. BERCILLES Presiding Judge, Branch XV, 14th
property is a part of the public dominion, of which neither party is entitled to
Judicial District, and JOSE L. ESPELETA, Assistant Provincial Fiscal,
own nor possess, the decisions of the Court of Appeals dated 20 August 2002,
Province of Cebu, representing the Solicitor General's Office and the
the Regional Trial Court of La Trinidad, Benguet, dated 23 January 2002, and
Bureau of Lands, respondents.
the Municipal Circuit Trial Court of Tuba-Sablan, Tuba, Benguet, dated 20
November 2000 are SET ASIDE. Respondents Juanito and Amalia
Valenciano are ordered to remove their structure on the subject land within This is a petition for the review of the order of the Court of First Instance of
sixty (60) days from receipt of this decision, and to vacate and deliver the Cebu dismissing petitioner's application for registration of title over a parcel
physical possession thereof to the Office of the District Engineer, Benguet of land situated in the City of Cebu.
Engineering District, Department of Public Works and Highways.
The parcel of land sought to be registered was only a portion of M. Borces
SO ORDERED. Street, Mabolo, Cebu City. On September 23, 1968, the City Council of Cebu,
through Resolution No. 2193, approved on October 3, 1968, declared the
terminal portion of M. Borces Street, Mabolo, Cebu City, as an abandoned
road, the same not being included in the City Development
Plan.1 Subsequently, on December 19, 1968, the City Council of Cebu passed
Resolution No. 2755, authorizing the Acting City Mayor to sell the land
through a public bidding.2 Pursuant thereto, the lot was awarded to the herein
petitioner being the highest bidder and on March 3, 1969, the City of Cebu,
through the Acting City Mayor, executed a deed of absolute sale to the herein
petitioner for a total consideration of P10,800.00.3 By virtue of the aforesaid
deed of absolute sale, the petitioner filed an application with the Court of First
instance of Cebu to have its title to the land registered.4

On June 26, 1974, the Assistant Provincial Fiscal of Cebu filed a motion to
dismiss the application on the ground that the property sought to be registered
being a public road intended for public use is considered part of the public
domain and therefore outside the commerce of man. Consequently, it cannot
be subject to registration by any private individual.5

After hearing the parties, on October 11, 1974 the trial court issued an order
dismissing the petitioner's application for registration of title.6 Hence, the
instant petition for review.

For the resolution of this case, the petitioner poses the following questions:

(1) Does the City Charter of Cebu City (Republic Act


No. 3857) under Section 31, paragraph 34, give the City
of Cebu the valid right to declare a road as abandoned?
and

(2) Does the declaration of the road, as abandoned,


make it the patrimonial property of the City of Cebu
which may be the object of a common contract?

(1) The pertinent portions of the Revised Charter of Cebu City provides:

Section 31. Legislative Powers. Any provision of law


and executive order to the contrary notwithstanding, the
City Council shall have the following legislative
powers:

xxx xxx xxx

(34) ...; to close any city road, street or alley, boulevard,


avenue, park or square. Property thus withdrawn from
public servitude may be used or conveyed for any
purpose for which other real property belonging to the
City may be lawfully used or conveyed.

From the foregoing, it is undoubtedly clear that the City of Cebu is


empowered to close a city road or street. In the case of Favis vs. City of
Baguio,7 where the power of the city Council of Baguio City to close city
streets and to vacate or withdraw the same from public use was similarly
assailed, this court said:

5. So it is, that appellant may not challenge the city


council's act of withdrawing a strip of Lapu-Lapu Street
at its dead end from public use and converting the
remainder thereof into an alley. These are acts well
within the ambit of the power to close a city street. The
city council, it would seem to us, is the authority
competent to determine whether or not a certain
property is still necessary for public use.

Such power to vacate a street or alley is discretionary.


And the discretion will not ordinarily be controlled or
interfered with by the courts, absent a plain case of
abuse or fraud or collusion. Faithfulness to the public
trust will be presumed. So the fact that some private
interests may be served incidentally will not invalidate
the vacation ordinance.

(2) Since that portion of the city street subject of petitioner's application for
registration of title was withdrawn from public use, it follows that such
withdrawn portion becomes patrimonial property which can be the object of
an ordinary contract.

Article 422 of the Civil Code expressly provides that "Property of public
dominion, when no longer intended for public use or for public service, shall
form part of the patrimonial property of the State." G.R. No. 133250           July 9, 2002

Besides, the Revised Charter of the City of Cebu heretofore quoted, in very FRANCISCO I. CHAVEZ, petitioner,
clear and unequivocal terms, states that: "Property thus withdrawn from public vs.
servitude may be used or conveyed for any purpose for which other real PUBLIC ESTATES AUTHORITY and AMARI COASTAL BAY
property belonging to the City may be lawfully used or conveyed." DEVELOPMENT CORPORATION, respondents.

Accordingly, the withdrawal of the property in question from public use and This is an original Petition for Mandamus with prayer for a writ of
its subsequent sale to the petitioner is valid. Hence, the petitioner has a preliminary injunction and a temporary restraining order. The petition seeks to
registerable title over the lot in question. compel the Public Estates Authority ("PEA" for brevity) to disclose all facts
on PEA's then on-going renegotiations with Amari Coastal Bay and
Development Corporation ("AMARI" for brevity) to reclaim portions of
WHEREFORE, the order dated October 11, 1974, rendered by the respondent Manila Bay. The petition further seeks to enjoin PEA from signing a new
court in Land Reg. Case No. N-948, LRC Rec. No. N-44531 is hereby set agreement with AMARI involving such reclamation.
aside, and the respondent court is hereby ordered to proceed with the hearing
of the petitioner's application for registration of title.
The Facts

SO ORDERED.
On November 20, 1973, the government, through the Commissioner of Public
Highways, signed a contract with the Construction and Development
Corporation of the Philippines ("CDCP" for brevity) to reclaim certain
foreshore and offshore areas of Manila Bay. The contract also included the
construction of Phases I and II of the Manila-Cavite Coastal Road. CDCP
obligated itself to carry out all the works in consideration of fifty percent of
the total reclaimed land.

On February 4, 1977, then President Ferdinand E. Marcos issued Presidential


Decree No. 1084 creating PEA. PD No. 1084 tasked PEA "to reclaim land,
including foreshore and submerged areas," and "to develop, improve, acquire,
x x x lease and sell any and all kinds of lands." 1 On the same date, then
President Marcos issued Presidential Decree No. 1085 transferring to PEA the
"lands reclaimed in the foreshore and offshore of the Manila Bay" 2 under the
Manila-Cavite Coastal Road and Reclamation Project (MCCRRP).

On December 29, 1981, then President Marcos issued a memorandum


directing PEA to amend its contract with CDCP, so that "[A]ll future works in
MCCRRP x x x shall be funded and owned by PEA." Accordingly, PEA and
CDCP executed a Memorandum of Agreement dated December 29, 1981,
which stated:
"(i) CDCP shall undertake all reclamation, construction, and such On April 13, 1998, Antonio M. Zulueta filed before the Court a Petition for
other works in the MCCRRP as may be agreed upon by the parties, Prohibition with Application for the Issuance of a Temporary Restraining
to be paid according to progress of works on a unit price/lump sum Order and Preliminary Injunction docketed as G.R. No. 132994 seeking to
basis for items of work to be agreed upon, subject to price nullify the JVA. The Court dismissed the petition "for unwarranted disregard
escalation, retention and other terms and conditions provided for in of judicial hierarchy, without prejudice to the refiling of the case before the
Presidential Decree No. 1594. All the financing required for such proper court."12
works shall be provided by PEA.
On April 27, 1998, petitioner Frank I. Chavez ("Petitioner" for brevity) as a
xxx taxpayer, filed the instant Petition for Mandamus with Prayer for the Issuance
of a Writ of Preliminary Injunction and Temporary Restraining Order.
Petitioner contends the government stands to lose billions of pesos in the sale
(iii) x x x CDCP shall give up all its development rights and
by PEA of the reclaimed lands to AMARI. Petitioner prays that PEA publicly
hereby agrees to cede and transfer in favor of PEA, all of the
disclose the terms of any renegotiation of the JVA, invoking Section 28,
rights, title, interest and participation of CDCP in and to all the
Article II, and Section 7, Article III, of the 1987 Constitution on the right of
areas of land reclaimed by CDCP in the MCCRRP as of December
the people to information on matters of public concern. Petitioner assails the
30, 1981 which have not yet been sold, transferred or otherwise
sale to AMARI of lands of the public domain as a blatant violation of Section
disposed of by CDCP as of said date, which areas consist of
3, Article XII of the 1987 Constitution prohibiting the sale of alienable lands
approximately Ninety-Nine Thousand Four Hundred Seventy
of the public domain to private corporations. Finally, petitioner asserts that he
Three (99,473) square meters in the Financial Center Area covered
seeks to enjoin the loss of billions of pesos in properties of the State that are
by land pledge No. 5 and approximately Three Million Three
of public dominion.
Hundred Eighty Two Thousand Eight Hundred Eighty Eight
(3,382,888) square meters of reclaimed areas at varying elevations
above Mean Low Water Level located outside the Financial Center After several motions for extension of time,13 PEA and AMARI filed their
Area and the First Neighborhood Unit."3 Comments on October 19, 1998 and June 25, 1998, respectively. Meanwhile,
on December 28, 1998, petitioner filed an Omnibus Motion: (a) to require
PEA to submit the terms of the renegotiated PEA-AMARI contract; (b) for
On January 19, 1988, then President Corazon C. Aquino issued Special Patent
issuance of a temporary restraining order; and (c) to set the case for hearing on
No. 3517, granting and transferring to PEA "the parcels of land so reclaimed
oral argument. Petitioner filed a Reiterative Motion for Issuance of a TRO
under the Manila-Cavite Coastal Road and Reclamation Project (MCCRRP)
dated May 26, 1999, which the Court denied in a Resolution dated June 22,
containing a total area of one million nine hundred fifteen thousand eight
1999.
hundred ninety four (1,915,894) square meters." Subsequently, on April 9,
1988, the Register of Deeds of the Municipality of Parañaque issued Transfer
Certificates of Title Nos. 7309, 7311, and 7312, in the name of PEA, covering In a Resolution dated March 23, 1999, the Court gave due course to the
the three reclaimed islands known as the "Freedom Islands" located at the petition and required the parties to file their respective memoranda.
southern portion of the Manila-Cavite Coastal Road, Parañaque City. The
Freedom Islands have a total land area of One Million Five Hundred Seventy
On March 30, 1999, PEA and AMARI signed the Amended Joint Venture
Eight Thousand Four Hundred and Forty One (1,578,441) square meters or
Agreement ("Amended JVA," for brevity). On May 28, 1999, the Office of
157.841 hectares.
the President under the administration of then President Joseph E. Estrada
approved the Amended JVA.
On April 25, 1995, PEA entered into a Joint Venture Agreement ("JVA" for
brevity) with AMARI, a private corporation, to develop the Freedom Islands.
Due to the approval of the Amended JVA by the Office of the President,
The JVA also required the reclamation of an additional 250 hectares of
petitioner now prays that on "constitutional and statutory grounds the
submerged areas surrounding these islands to complete the configuration in
renegotiated contract be declared null and void."14
the Master Development Plan of the Southern Reclamation Project-MCCRRP.
PEA and AMARI entered into the JVA through negotiation without public
bidding.4 On April 28, 1995, the Board of Directors of PEA, in its Resolution The Issues
No. 1245, confirmed the JVA.5 On June 8, 1995, then President Fidel V.
Ramos, through then Executive Secretary Ruben Torres, approved the JVA.6
The issues raised by petitioner, PEA15 and AMARI16 are as follows:

On November 29, 1996, then Senate President Ernesto Maceda delivered a


I. WHETHER THE PRINCIPAL RELIEFS PRAYED FOR IN
privilege speech in the Senate and denounced the JVA as the "grandmother of
THE PETITION ARE MOOT AND ACADEMIC BECAUSE OF
all scams." As a result, the Senate Committee on Government Corporations
SUBSEQUENT EVENTS;
and Public Enterprises, and the Committee on Accountability of Public
Officers and Investigations, conducted a joint investigation. The Senate
Committees reported the results of their investigation in Senate Committee II. WHETHER THE PETITION MERITS DISMISSAL FOR
Report No. 560 dated September 16, 1997.7 Among the conclusions of their FAILING TO OBSERVE THE PRINCIPLE GOVERNING THE
report are: (1) the reclaimed lands PEA seeks to transfer to AMARI under the HIERARCHY OF COURTS;
JVA are lands of the public domain which the government has not classified
as alienable lands and therefore PEA cannot alienate these lands; (2) the
certificates of title covering the Freedom Islands are thus void, and (3) the III. WHETHER THE PETITION MERITS DISMISSAL FOR
JVA itself is illegal. NON-EXHAUSTION OF ADMINISTRATIVE REMEDIES;

On December 5, 1997, then President Fidel V. Ramos issued Presidential IV. WHETHER PETITIONER HAS LOCUS STANDI TO BRING
Administrative Order No. 365 creating a Legal Task Force to conduct a study THIS SUIT;
on the legality of the JVA in view of Senate Committee Report No. 560. The
members of the Legal Task Force were the Secretary of Justice,8 the Chief V. WHETHER THE CONSTITUTIONAL RIGHT TO
Presidential Legal Counsel,9 and the Government Corporate Counsel.10 The INFORMATION INCLUDES OFFICIAL INFORMATION ON
Legal Task Force upheld the legality of the JVA, contrary to the conclusions ON-GOING NEGOTIATIONS BEFORE A FINAL
reached by the Senate Committees.11 AGREEMENT;

On April 4 and 5, 1998, the Philippine Daily Inquirer and Today published VI. WHETHER THE STIPULATIONS IN THE AMENDED
reports that there were on-going renegotiations between PEA and AMARI JOINT VENTURE AGREEMENT FOR THE TRANSFER TO
under an order issued by then President Fidel V. Ramos. According to these AMARI OF CERTAIN LANDS, RECLAIMED AND STILL TO
reports, PEA Director Nestor Kalaw, PEA Chairman Arsenio Yulo and retired BE RECLAIMED, VIOLATE THE 1987 CONSTITUTION; AND
Navy Officer Sergio Cruz composed the negotiating panel of PEA.
VII. WHETHER THE COURT IS THE PROPER FORUM FOR Lastly, there is a need to resolve immediately the constitutional issue raised in
RAISING THE ISSUE OF WHETHER THE AMENDED JOINT this petition because of the possible transfer at any time by PEA to AMARI of
VENTURE AGREEMENT IS GROSSLY title and ownership to portions of the reclaimed lands. Under the Amended
DISADVANTAGEOUS TO THE GOVERNMENT. JVA, PEA is obligated to transfer to AMARI the latter's seventy percent
proportionate share in the reclaimed areas as the reclamation progresses. The
Amended JVA even allows AMARI to mortgage at any time
The Court's Ruling
the entire reclaimed area to raise financing for the reclamation project.21

First issue: whether the principal reliefs prayed for in the petition are moot
Second issue: whether the petition merits dismissal for failing to observe the
and academic because of subsequent events.
principle governing the hierarchy of courts.

The petition prays that PEA publicly disclose the "terms and conditions of the
PEA and AMARI claim petitioner ignored the judicial hierarchy by seeking
on-going negotiations for a new agreement." The petition also prays that the
relief directly from the Court. The principle of hierarchy of courts applies
Court enjoin PEA from "privately entering into, perfecting and/or executing
generally to cases involving factual questions. As it is not a trier of facts, the
any new agreement with AMARI."
Court cannot entertain cases involving factual issues. The instant case,
however, raises constitutional issues of transcendental importance to the
PEA and AMARI claim the petition is now moot and academic because public.22 The Court can resolve this case without determining any factual issue
AMARI furnished petitioner on June 21, 1999 a copy of the signed Amended related to the case. Also, the instant case is a petition for mandamus which
JVA containing the terms and conditions agreed upon in the renegotiations. falls under the original jurisdiction of the Court under Section 5, Article VIII
Thus, PEA has satisfied petitioner's prayer for a public disclosure of the of the Constitution. We resolve to exercise primary jurisdiction over the
renegotiations. Likewise, petitioner's prayer to enjoin the signing of the instant case.
Amended JVA is now moot because PEA and AMARI have already signed
the Amended JVA on March 30, 1999. Moreover, the Office of the President
Third issue: whether the petition merits dismissal for non-exhaustion of
has approved the Amended JVA on May 28, 1999.
administrative remedies.

Petitioner counters that PEA and AMARI cannot avoid the constitutional issue
PEA faults petitioner for seeking judicial intervention in compelling PEA to
by simply fast-tracking the signing and approval of the Amended JVA before
disclose publicly certain information without first asking PEA the needed
the Court could act on the issue. Presidential approval does not resolve the
information. PEA claims petitioner's direct resort to the Court violates the
constitutional issue or remove it from the ambit of judicial review.
principle of exhaustion of administrative remedies. It also violates the rule that
mandamus may issue only if there is no other plain, speedy and adequate
We rule that the signing of the Amended JVA by PEA and AMARI and its remedy in the ordinary course of law.
approval by the President cannot operate to moot the petition and divest the
Court of its jurisdiction. PEA and AMARI have still to implement the
PEA distinguishes the instant case from Tañada v. Tuvera 23 where the Court
Amended JVA. The prayer to enjoin the signing of the Amended JVA on
granted the petition for mandamus even if the petitioners there did not initially
constitutional grounds necessarily includes preventing its implementation if in
demand from the Office of the President the publication of the presidential
the meantime PEA and AMARI have signed one in violation of the
decrees. PEA points out that in Tañada, the Executive Department had
Constitution. Petitioner's principal basis in assailing the renegotiation of the
an affirmative statutory duty under Article 2 of the Civil Code24 and Section 1
JVA is its violation of Section 3, Article XII of the Constitution, which
of Commonwealth Act No. 63825 to publish the presidential decrees. There
prohibits the government from alienating lands of the public domain to private
was, therefore, no need for the petitioners in Tañada to make an initial
corporations. If the Amended JVA indeed violates the Constitution, it is the
demand from the Office of the President. In the instant case, PEA claims it has
duty of the Court to enjoin its implementation, and if already implemented, to
no affirmative statutory duty to disclose publicly information about its
annul the effects of such unconstitutional contract.
renegotiation of the JVA. Thus, PEA asserts that the Court must apply the
principle of exhaustion of administrative remedies to the instant case in view
The Amended JVA is not an ordinary commercial contract but one which of the failure of petitioner here to demand initially from PEA the needed
seeks to transfer title and ownership to 367.5 hectares of reclaimed lands information.
and submerged areas of Manila Bay to a single private corporation. It now
becomes more compelling for the Court to resolve the issue to insure the
The original JVA sought to dispose to AMARI public lands held by PEA, a
government itself does not violate a provision of the Constitution intended to
government corporation. Under Section 79 of the Government Auditing
safeguard the national patrimony. Supervening events, whether intended or
Code,26 the disposition of government lands to private parties requires public
accidental, cannot prevent the Court from rendering a decision if there is a
bidding. PEA was under a positive legal duty to disclose to the public the
grave violation of the Constitution. In the instant case, if the Amended JVA
terms and conditions for the sale of its lands. The law obligated PEA to
runs counter to the Constitution, the Court can still prevent the transfer of title
make this public disclosure even without demand from petitioner or from
and ownership of alienable lands of the public domain in the name of
anyone. PEA failed to make this public disclosure because the original JVA,
AMARI. Even in cases where supervening events had made the cases moot,
like the Amended JVA, was the result of a negotiated contract, not of a public
the Court did not hesitate to resolve the legal or constitutional issues raised to
bidding. Considering that PEA had an affirmative statutory duty to make the
formulate controlling principles to guide the bench, bar, and the public.17
public disclosure, and was even in breach of this legal duty, petitioner had the
right to seek direct judicial intervention.
Also, the instant petition is a case of first impression. All previous decisions
of the Court involving Section 3, Article XII of the 1987 Constitution, or its
Moreover, and this alone is determinative of this issue, the principle of
counterpart provision in the 1973 Constitution,18 covered agricultural
exhaustion of administrative remedies does not apply when the issue involved
lands sold to private corporations which acquired the lands from private
is a purely legal or constitutional question.27 The principal issue in the instant
parties. The transferors of the private corporations claimed or could claim the
case is the capacity of AMARI to acquire lands held by PEA in view of the
right to judicial confirmation of their imperfect titles 19 under Title II of
constitutional ban prohibiting the alienation of lands of the public domain to
Commonwealth Act. 141 ("CA No. 141" for brevity). In the instant case,
private corporations. We rule that the principle of exhaustion of administrative
AMARI seeks to acquire from PEA, a public corporation, reclaimed lands and
remedies does not apply in the instant case.
submerged areas for non-agricultural purposes by purchase under PD No.
1084 (charter of PEA) and Title III of CA No. 141. Certain undertakings by
AMARI under the Amended JVA constitute the consideration for the Fourth issue: whether petitioner has locus standi to bring this suit
purchase. Neither AMARI nor PEA can claim judicial confirmation of their
titles because the lands covered by the Amended JVA are newly reclaimed or
PEA argues that petitioner has no standing to institute mandamus proceedings
still to be reclaimed. Judicial confirmation of imperfect title requires open,
to enforce his constitutional right to information without a showing that PEA
continuous, exclusive and notorious occupation of agricultural lands of the
refused to perform an affirmative duty imposed on PEA by the Constitution.
public domain for at least thirty years since June 12, 1945 or earlier. Besides,
PEA also claims that petitioner has not shown that he will suffer any concrete
the deadline for filing applications for judicial confirmation of imperfect title
injury because of the signing or implementation of the Amended JVA. Thus,
expired on December 31, 1987.20
there is no actual controversy requiring the exercise of the power of judicial — a right guaranteed under Section 7, Article III of the 1987
review. Constitution. Petitioner, a former solicitor general, is a Filipino
citizen. Because of the satisfaction of the two basic requisites laid
down by decisional law to sustain petitioner's legal standing, i.e.
The petitioner has standing to bring this taxpayer's suit because the petition
(1) the enforcement of a public right (2) espoused by a Filipino
seeks to compel PEA to comply with its constitutional duties. There are two
citizen, we rule that the petition at bar should be allowed."
constitutional issues involved here. First is the right of citizens to information
on matters of public concern. Second is the application of a constitutional
provision intended to insure the equitable distribution of alienable lands of the We rule that since the instant petition, brought by a citizen, involves the
public domain among Filipino citizens. The thrust of the first issue is to enforcement of constitutional rights - to information and to the equitable
compel PEA to disclose publicly information on the sale of government lands diffusion of natural resources - matters of transcendental public importance,
worth billions of pesos, information which the Constitution and statutory law the petitioner has the requisite locus standi.
mandate PEA to disclose. The thrust of the second issue is to prevent PEA
from alienating hundreds of hectares of alienable lands of the public domain
Fifth issue: whether the constitutional right to information includes official
in violation of the Constitution, compelling PEA to comply with a
information on on-going negotiations before a final agreement.
constitutional duty to the nation.

Section 7, Article III of the Constitution explains the people's right to


Moreover, the petition raises matters of transcendental importance to the
information on matters of public concern in this manner:
public. In Chavez v. PCGG,28 the Court upheld the right of a citizen to bring a
taxpayer's suit on matters of transcendental importance to the public, thus -
"Sec. 7. The right of the people to information on matters of public
concern shall be recognized. Access to official records, and to
"Besides, petitioner emphasizes, the matter of recovering the ill-
documents, and papers pertaining to official acts, transactions,
gotten wealth of the Marcoses is an issue of 'transcendental
or decisions, as well as to government research data used as basis
importance to the public.' He asserts that ordinary taxpayers have a
for policy development, shall be afforded the citizen, subject to
right to initiate and prosecute actions questioning the validity of
such limitations as may be provided by law." (Emphasis supplied)
acts or orders of government agencies or instrumentalities, if the
issues raised are of 'paramount public interest,' and if they
'immediately affect the social, economic and moral well being of The State policy of full transparency in all transactions involving public
the people.' interest reinforces the people's right to information on matters of public
concern. This State policy is expressed in Section 28, Article II of the
Constitution, thus:
Moreover, the mere fact that he is a citizen satisfies the
requirement of personal interest, when the proceeding involves the
assertion of a public right, such as in this case. He invokes several "Sec. 28. Subject to reasonable conditions prescribed by law, the
decisions of this Court which have set aside the procedural matter State adopts and implements a policy of full public disclosure of
of locus standi, when the subject of the case involved public all its transactions involving public interest." (Emphasis supplied)
interest.
These twin provisions of the Constitution seek to promote transparency in
xxx policy-making and in the operations of the government, as well as provide the
people sufficient information to exercise effectively other constitutional
rights. These twin provisions are essential to the exercise of freedom of
In Tañada v. Tuvera, the Court asserted that when the issue
expression. If the government does not disclose its official acts, transactions
concerns a public right and the object of mandamus is to obtain the
and decisions to citizens, whatever citizens say, even if expressed without any
enforcement of a public duty, the people are regarded as the real
restraint, will be speculative and amount to nothing. These twin provisions are
parties in interest; and because it is sufficient that petitioner is a
also essential to hold public officials "at all times x x x accountable to the
citizen and as such is interested in the execution of the laws, he
people,"29 for unless citizens have the proper information, they cannot hold
need not show that he has any legal or special interest in the result
public officials accountable for anything. Armed with the right information,
of the action. In the aforesaid case, the petitioners sought to
citizens can participate in public discussions leading to the formulation of
enforce their right to be informed on matters of public concern, a
government policies and their effective implementation. An informed
right then recognized in Section 6, Article IV of the 1973
citizenry is essential to the existence and proper functioning of any
Constitution, in connection with the rule that laws in order to be
democracy. As explained by the Court in Valmonte v. Belmonte, Jr.30 –
valid and enforceable must be published in the Official Gazette or
otherwise effectively promulgated. In ruling for the petitioners'
legal standing, the Court declared that the right they sought to be "An essential element of these freedoms is to keep open a
enforced 'is a public right recognized by no less than the continuing dialogue or process of communication between the
fundamental law of the land.' government and the people. It is in the interest of the State that the
channels for free political discussion be maintained to the end that
the government may perceive and be responsive to the people's
Legaspi v. Civil Service Commission, while reiterating Tañada,
will. Yet, this open dialogue can be effective only to the extent that
further declared that 'when a mandamus proceeding involves the
the citizenry is informed and thus able to formulate its will
assertion of a public right, the requirement of personal interest is
intelligently. Only when the participants in the discussion are
satisfied by the mere fact that petitioner is a citizen and, therefore,
aware of the issues and have access to information relating thereto
part of the general 'public' which possesses the right.'
can such bear fruit."

Further, in Albano v. Reyes, we said that while expenditure of


PEA asserts, citing Chavez v. PCGG,31 that in cases of on-going negotiations
public funds may not have been involved under the questioned
the right to information is limited to "definite propositions of the
contract for the development, management and operation of the
government." PEA maintains the right does not include access to "intra-
Manila International Container Terminal, 'public interest [was]
agency or inter-agency recommendations or communications during the stage
definitely involved considering the important role [of the subject
when common assertions are still in the process of being formulated or are in
contract] . . . in the economic development of the country and the
the 'exploratory stage'."
magnitude of the financial consideration involved.' We concluded
that, as a consequence, the disclosure provision in the Constitution
would constitute sufficient authority for upholding the petitioner's Also, AMARI contends that petitioner cannot invoke the right at the pre-
standing. decisional stage or before the closing of the transaction. To support its
contention, AMARI cites the following discussion in the 1986 Constitutional
Commission:
Similarly, the instant petition is anchored on the right of the people
to information and access to official records, documents and papers
"Mr. Suarez. And when we say 'transactions' which should be Requiring a consummated contract will keep the public in the dark until the
distinguished from contracts, agreements, or treaties or whatever, contract, which may be grossly disadvantageous to the government or even
does the Gentleman refer to the steps leading to the consummation illegal, becomes a fait accompli. This negates the State policy of full
of the contract, or does he refer to the contract itself? transparency on matters of public concern, a situation which the framers of the
Constitution could not have intended. Such a requirement will prevent the
citizenry from participating in the public discussion of any proposed contract,
Mr. Ople: The 'transactions' used here, I suppose is generic and
effectively truncating a basic right enshrined in the Bill of Rights. We can
therefore, it can cover both steps leading to a contract and
allow neither an emasculation of a constitutional right, nor a retreat by the
already a consummated contract, Mr. Presiding Officer.
State of its avowed "policy of full disclosure of all its transactions involving
public interest."
Mr. Suarez: This contemplates inclusion of negotiations leading
to the consummation of the transaction.
The right covers three categories of information which are "matters of public
concern," namely: (1) official records; (2) documents and papers pertaining to
Mr. Ople: Yes, subject only to reasonable safeguards on the official acts, transactions and decisions; and (3) government research data
national interest. used in formulating policies. The first category refers to any document that is
part of the public records in the custody of government agencies or officials.
The second category refers to documents and papers recording, evidencing,
Mr. Suarez: Thank you."32 (Emphasis supplied) establishing, confirming, supporting, justifying or explaining official acts,
transactions or decisions of government agencies or officials. The third
AMARI argues there must first be a consummated contract before petitioner category refers to research data, whether raw, collated or processed, owned by
can invoke the right. Requiring government officials to reveal their the government and used in formulating government policies.
deliberations at the pre-decisional stage will degrade the quality of decision-
making in government agencies. Government officials will hesitate to express The information that petitioner may access on the renegotiation of the JVA
their real sentiments during deliberations if there is immediate public includes evaluation reports, recommendations, legal and expert opinions,
dissemination of their discussions, putting them under all kinds of pressure minutes of meetings, terms of reference and other documents attached to such
before they decide. reports or minutes, all relating to the JVA. However, the right to information
does not compel PEA to prepare lists, abstracts, summaries and the like
We must first distinguish between information the law on public bidding relating to the renegotiation of the JVA.34 The right only affords access to
requires PEA to disclose publicly, and information the constitutional right to records, documents and papers, which means the opportunity to inspect and
information requires PEA to release to the public. Before the consummation copy them. One who exercises the right must copy the records, documents and
of the contract, PEA must, on its own and without demand from anyone, papers at his expense. The exercise of the right is also subject to reasonable
disclose to the public matters relating to the disposition of its property. These regulations to protect the integrity of the public records and to minimize
include the size, location, technical description and nature of the property disruption to government operations, like rules specifying when and how to
being disposed of, the terms and conditions of the disposition, the parties conduct the inspection and copying.35
qualified to bid, the minimum price and similar information. PEA must
prepare all these data and disclose them to the public at the start of the The right to information, however, does not extend to matters recognized as
disposition process, long before the consummation of the contract, because the privileged information under the separation of powers. 36 The right does not
Government Auditing Code requires public bidding. If PEA fails to make this also apply to information on military and diplomatic secrets, information
disclosure, any citizen can demand from PEA this information at any time affecting national security, and information on investigations of crimes by law
during the bidding process. enforcement agencies before the prosecution of the accused, which courts
have long recognized as confidential.37 The right may also be subject to other
Information, however, on on-going evaluation or review of bids or proposals limitations that Congress may impose by law.
being undertaken by the bidding or review committee is not immediately
accessible under the right to information. While the evaluation or review is There is no claim by PEA that the information demanded by petitioner is
still on-going, there are no "official acts, transactions, or decisions" on the privileged information rooted in the separation of powers. The information
bids or proposals. However, once the committee makes its official does not cover Presidential conversations, correspondences, or discussions
recommendation, there arises a "definite proposition" on the part of the during closed-door Cabinet meetings which, like internal deliberations of the
government. From this moment, the public's right to information attaches, and Supreme Court and other collegiate courts, or executive sessions of either
any citizen can access all the non-proprietary information leading to such house of Congress,38 are recognized as confidential. This kind of information
definite proposition. In Chavez v. PCGG,33 the Court ruled as follows: cannot be pried open by a co-equal branch of government. A frank exchange
of exploratory ideas and assessments, free from the glare of publicity and
"Considering the intent of the framers of the Constitution, we pressure by interested parties, is essential to protect the independence of
believe that it is incumbent upon the PCGG and its officers, as well decision-making of those tasked to exercise Presidential, Legislative and
as other government representatives, to disclose sufficient public Judicial power.39 This is not the situation in the instant case.
information on any proposed settlement they have decided to take
up with the ostensible owners and holders of ill-gotten wealth. We rule, therefore, that the constitutional right to information includes official
Such information, though, must pertain to definite propositions of information on on-going negotiations before a final contract. The
the government, not necessarily to intra-agency or inter-agency information, however, must constitute definite propositions by the
recommendations or communications during the stage when government and should not cover recognized exceptions like privileged
common assertions are still in the process of being formulated or information, military and diplomatic secrets and similar matters affecting
are in the "exploratory" stage. There is need, of course, to observe national security and public order.40 Congress has also prescribed other
the same restrictions on disclosure of information in general, as limitations on the right to information in several legislations.41
discussed earlier – such as on matters involving national security,
diplomatic or foreign relations, intelligence and other classified
information." (Emphasis supplied) Sixth issue: whether stipulations in the Amended JVA for the transfer to
AMARI of lands, reclaimed or to be reclaimed, violate the Constitution.
Contrary to AMARI's contention, the commissioners of the 1986
Constitutional Commission understood that the right to The Regalian Doctrine
information "contemplates inclusion of negotiations leading to the
consummation of the transaction." Certainly, a consummated contract is not
The ownership of lands reclaimed from foreshore and submerged areas is
a requirement for the exercise of the right to information. Otherwise, the
rooted in the Regalian doctrine which holds that the State owns all lands and
people can never exercise the right if no contract is consummated, and if one
waters of the public domain. Upon the Spanish conquest of the Philippines,
is consummated, it may be too late for the public to expose its
ownership of all "lands, territories and possessions" in the Philippines passed
defects.1âwphi1.nêt
to the Spanish Crown.42 The King, as the sovereign ruler and representative of
the people, acquired and owned all lands and territories in the Philippines Article 341 of the Civil Code of 1889 governed the re-classification of
except those he disposed of by grant or sale to private individuals. property of public dominion into private property, to wit:

The 1935, 1973 and 1987 Constitutions adopted the Regalian doctrine "Art. 341. Property of public dominion, when no longer devoted to
substituting, however, the State, in lieu of the King, as the owner of all lands public use or to the defense of the territory, shall become a part of
and waters of the public domain. The Regalian doctrine is the foundation of the private property of the State."
the time-honored principle of land ownership that "all lands that were not
acquired from the Government, either by purchase or by grant, belong to the
This provision, however, was not self-executing. The legislature, or the
public domain."43 Article 339 of the Civil Code of 1889, which is now Article
executive department pursuant to law, must declare the property no longer
420 of the Civil Code of 1950, incorporated the Regalian doctrine.
needed for public use or territorial defense before the government could lease
or alienate the property to private parties.45
Ownership and Disposition of Reclaimed Lands
Act No. 1654 of the Philippine Commission
The Spanish Law of Waters of 1866 was the first statutory law governing the
ownership and disposition of reclaimed lands in the Philippines. On May 18,
On May 8, 1907, the Philippine Commission enacted Act No. 1654 which
1907, the Philippine Commission enacted Act No. 1654 which provided for
regulated the lease of reclaimed and foreshore lands. The salient provisions of
the lease, but not the sale, of reclaimed lands of the government to
this law were as follows:
corporations and individuals. Later, on November 29, 1919, the Philippine
Legislature approved Act No. 2874, the Public Land Act, which
authorized the lease, but not the sale, of reclaimed lands of the government "Section 1. The control and disposition of the foreshore as
to corporations and individuals. On November 7, 1936, the National defined in existing law, and the title to all Government or public
Assembly passed Commonwealth Act No. 141, also known as the Public Land lands made or reclaimed by the Government by dredging or
Act, which authorized the lease, but not the sale, of reclaimed lands of the filling or otherwise throughout the Philippine Islands, shall be
government to corporations and individuals. CA No. 141 continues to this retained by the Government without prejudice to vested rights and
day as the general law governing the classification and disposition of lands of without prejudice to rights conceded to the City of Manila in the
the public domain. Luneta Extension.

The Spanish Law of Waters of 1866 and the Civil Code of 1889 Section 2. (a) The Secretary of the Interior shall cause all
Government or public lands made or reclaimed by the Government
by dredging or filling or otherwise to be divided into lots or blocks,
Under the Spanish Law of Waters of 1866, the shores, bays, coves, inlets and
with the necessary streets and alleyways located thereon, and shall
all waters within the maritime zone of the Spanish territory belonged to the
cause plats and plans of such surveys to be prepared and filed with
public domain for public use.44 The Spanish Law of Waters of 1866 allowed
the Bureau of Lands.
the reclamation of the sea under Article 5, which provided as follows:

(b) Upon completion of such plats and plans the Governor-


"Article 5. Lands reclaimed from the sea in consequence of works
General shall give notice to the public that such parts of the
constructed by the State, or by the provinces, pueblos or private
lands so made or reclaimed as are not needed for public
persons, with proper permission, shall become the property of the
purposes will be leased for commercial and business purposes, x
party constructing such works, unless otherwise provided by the
x x.
terms of the grant of authority."

xxx
Under the Spanish Law of Waters, land reclaimed from the sea belonged to
the party undertaking the reclamation, provided the government issued the
necessary permit and did not reserve ownership of the reclaimed land to the (e) The leases above provided for shall be disposed of to the
State. highest and best bidder therefore, subject to such regulations and
safeguards as the Governor-General may by executive order
prescribe." (Emphasis supplied)
Article 339 of the Civil Code of 1889 defined property of public dominion as
follows:
Act No. 1654 mandated that the government should retain title to all lands
reclaimed by the government. The Act also vested in the government control
"Art. 339. Property of public dominion is –
and disposition of foreshore lands. Private parties could lease lands reclaimed
by the government only if these lands were no longer needed for public
1. That devoted to public use, such as roads, canals, rivers, purpose. Act No. 1654 mandated public bidding in the lease of government
torrents, ports and bridges constructed by the State, riverbanks, reclaimed lands. Act No. 1654 made government reclaimed lands sui
shores, roadsteads, and that of a similar character; generis in that unlike other public lands which the government could sell to
private parties, these reclaimed lands were available only for lease to private
parties.
2. That belonging exclusively to the State which, without being of
general public use, is employed in some public service, or in the
development of the national wealth, such as walls, fortresses, and Act No. 1654, however, did not repeal Section 5 of the Spanish Law of
other works for the defense of the territory, and mines, until Waters of 1866. Act No. 1654 did not prohibit private parties from reclaiming
granted to private individuals." parts of the sea under Section 5 of the Spanish Law of Waters. Lands
reclaimed from the sea by private parties with government permission
remained private lands.
Property devoted to public use referred to property open for use by the public.
In contrast, property devoted to public service referred to property used for
some specific public service and open only to those authorized to use the Act No. 2874 of the Philippine Legislature
property.
On November 29, 1919, the Philippine Legislature enacted Act No. 2874, the
Property of public dominion referred not only to property devoted to public Public Land Act.46 The salient provisions of Act No. 2874, on reclaimed
use, but also to property not so used but employed to develop the national lands, were as follows:
wealth. This class of property constituted property of public dominion
although employed for some economic or commercial activity to increase the
national wealth.
"Sec. 6. The Governor-General, upon the recommendation of the sell government reclaimed, foreshore and marshy lands of the public domain,
Secretary of Agriculture and Natural Resources, shall from time a policy first enunciated in 1907 in Act No. 1654. Government reclaimed,
to time classify the lands of the public domain into – foreshore and marshy lands remained sui generis, as the only alienable or
disposable lands of the public domain that the government could not sell to
private parties.
(a) Alienable or disposable,
(b) Timber, and
(c) Mineral lands, x x x. The rationale behind this State policy is obvious. Government reclaimed,
foreshore and marshy public lands for non-agricultural purposes retain their
inherent potential as areas for public service. This is the reason the
government prohibited the sale, and only allowed the lease, of these lands to
Sec. 7. For the purposes of the government and disposition of
private parties. The State always reserved these lands for some future public
alienable or disposable public lands, the Governor-General, upon
service.
recommendation by the Secretary of Agriculture and Natural
Resources, shall from time to time declare what lands are open to
disposition or concession under this Act." Act No. 2874 did not authorize the reclassification of government reclaimed,
foreshore and marshy lands into other non-agricultural lands under Section 56
(d). Lands falling under Section 56 (d) were the only lands for non-
Sec. 8. Only those lands shall be declared open to disposition or
agricultural purposes the government could sell to private parties. Thus, under
concession which have been officially delimited or classified x x
Act No. 2874, the government could not sell government reclaimed, foreshore
x.
and marshy lands to private parties, unless the legislature passed a law
allowing their sale.49
xxx
Act No. 2874 did not prohibit private parties from reclaiming parts of the sea
Sec. 55. Any tract of land of the public domain which, being pursuant to Section 5 of the Spanish Law of Waters of 1866. Lands reclaimed
neither timber nor mineral land, shall be classified as suitable for from the sea by private parties with government permission remained private
residential purposes or for commercial, industrial, or other lands.
productive purposes other than agricultural purposes, and shall
be open to disposition or concession, shall be disposed of under the
Dispositions under the 1935 Constitution
provisions of this chapter, and not otherwise.

On May 14, 1935, the 1935 Constitution took effect upon its ratification by
Sec. 56. The lands disposable under this title shall be classified
the Filipino people. The 1935 Constitution, in adopting the Regalian doctrine,
as follows:
declared in Section 1, Article XIII, that –

(a) Lands reclaimed by the Government by dredging,


"Section 1. All agricultural, timber, and mineral lands of the public
filling, or other means;
domain, waters, minerals, coal, petroleum, and other mineral oils,
(b) Foreshore;
all forces of potential energy and other natural resources of the
(c) Marshy lands or lands covered with water
Philippines belong to the State, and their disposition, exploitation,
bordering upon the shores or banks of navigable lakes
development, or utilization shall be limited to citizens of the
or rivers;
Philippines or to corporations or associations at least sixty per
(d) Lands not included in any of the foregoing classes.
centum of the capital of which is owned by such citizens, subject
x x x.
to any existing right, grant, lease, or concession at the time of the
inauguration of the Government established under this
Sec. 58. The lands comprised in classes (a), (b), and (c) of section Constitution. Natural resources, with the exception of public
fifty-six shall be disposed of to private parties by lease only and agricultural land, shall not be alienated, and no license,
not otherwise, as soon as the Governor-General, upon concession, or lease for the exploitation, development, or
recommendation by the Secretary of Agriculture and Natural utilization of any of the natural resources shall be granted for a
Resources, shall declare that the same are not necessary for the period exceeding twenty-five years, renewable for another twenty-
public service and are open to disposition under this chapter. The five years, except as to water rights for irrigation, water supply,
lands included in class (d) may be disposed of by sale or lease fisheries, or industrial uses other than the development of water
under the provisions of this Act." (Emphasis supplied) power, in which cases beneficial use may be the measure and limit
of the grant." (Emphasis supplied)
Section 6 of Act No. 2874 authorized the Governor-General to "classify lands
of the public domain into x x x alienable or disposable" 47 lands. Section 7 of The 1935 Constitution barred the alienation of all natural resources except
the Act empowered the Governor-General to "declare what lands are open to public agricultural lands, which were the only natural resources the State
disposition or concession." Section 8 of the Act limited alienable or could alienate. Thus, foreshore lands, considered part of the State's natural
disposable lands only to those lands which have been "officially delimited and resources, became inalienable by constitutional fiat, available only for lease
classified." for 25 years, renewable for another 25 years. The government could alienate
foreshore lands only after these lands were reclaimed and classified as
alienable agricultural lands of the public domain. Government reclaimed and
Section 56 of Act No. 2874 stated that lands "disposable under this title 48 shall marshy lands of the public domain, being neither timber nor mineral lands,
be classified" as government reclaimed, foreshore and marshy lands, as well fell under the classification of public agricultural lands.50 However,
as other lands. All these lands, however, must be suitable for residential, government reclaimed and marshy lands, although subject to classification as
commercial, industrial or other productive non-agricultural purposes. These disposable public agricultural lands, could only be leased and not sold to
provisions vested upon the Governor-General the power to classify inalienable private parties because of Act No. 2874.
lands of the public domain into disposable lands of the public domain. These
provisions also empowered the Governor-General to classify further such
disposable lands of the public domain into government reclaimed, foreshore The prohibition on private parties from acquiring ownership of government
or marshy lands of the public domain, as well as other non-agricultural lands. reclaimed and marshy lands of the public domain was only a statutory
prohibition and the legislature could therefore remove such prohibition. The
1935 Constitution did not prohibit individuals and corporations from
Section 58 of Act No. 2874 categorically mandated that disposable lands of acquiring government reclaimed and marshy lands of the public domain that
the public domain classified as government reclaimed, foreshore and marshy were classified as agricultural lands under existing public land laws. Section
lands "shall be disposed of to private parties by lease only and not 2, Article XIII of the 1935 Constitution provided as follows:
otherwise." The Governor-General, before allowing the lease of these lands to
private parties, must formally declare that the lands were "not necessary for
the public service." Act No. 2874 reiterated the State policy to lease and not to
"Section 2. No private corporation or association may acquire, productive purposes other than agricultural, and is open to
lease, or hold public agricultural lands in excess of one thousand disposition or concession, shall be disposed of under the
and twenty four hectares, nor may any individual acquire such provisions of this chapter and not otherwise.
lands by purchase in excess of one hundred and forty hectares,
or by lease in excess of one thousand and twenty-four hectares,
Sec. 59. The lands disposable under this title shall be classified
or by homestead in excess of twenty-four hectares. Lands adapted
as follows:
to grazing, not exceeding two thousand hectares, may be leased to
an individual, private corporation, or association." (Emphasis
supplied) (a) Lands reclaimed by the Government by dredging,
filling, or other means;
(b) Foreshore;
Still, after the effectivity of the 1935 Constitution, the legislature did not
(c) Marshy lands or lands covered with water
repeal Section 58 of Act No. 2874 to open for sale to private parties
bordering upon the shores or banks of navigable lakes
government reclaimed and marshy lands of the public domain. On the
or rivers;
contrary, the legislature continued the long established State policy of
(d) Lands not included in any of the foregoing classes.
retaining for the government title and ownership of government reclaimed and
marshy lands of the public domain.

Sec. 60. Any tract of land comprised under this title may be leased
Commonwealth Act No. 141 of the Philippine National Assembly
or sold, as the case may be, to any person, corporation, or
association authorized to purchase or lease public lands for
On November 7, 1936, the National Assembly approved Commonwealth Act agricultural purposes. x x x.
No. 141, also known as the Public Land Act, which compiled the then existing
laws on lands of the public domain. CA No. 141, as amended, remains to this
Sec. 61. The lands comprised in classes (a), (b), and (c) of section
day the existing general law governing the classification and disposition of
fifty-nine shall be disposed of to private parties by lease only and
lands of the public domain other than timber and mineral lands.51
not otherwise, as soon as the President, upon recommendation by
the Secretary of Agriculture, shall declare that the same are not
Section 6 of CA No. 141 empowers the President to classify lands of the necessary for the public service and are open to disposition under
public domain into "alienable or disposable"52 lands of the public domain, this chapter. The lands included in class (d) may be disposed of
which prior to such classification are inalienable and outside the commerce of by sale or lease under the provisions of this Act." (Emphasis
man. Section 7 of CA No. 141 authorizes the President to "declare what lands supplied)
are open to disposition or concession." Section 8 of CA No. 141 states that the
government can declare open for disposition or concession only lands that are
Section 61 of CA No. 141 readopted, after the effectivity of the 1935
"officially delimited and classified." Sections 6, 7 and 8 of CA No. 141 read
Constitution, Section 58 of Act No. 2874 prohibiting the sale of government
as follows:
reclaimed, foreshore and marshy disposable lands of the public domain. All
these lands are intended for residential, commercial, industrial or other non-
"Sec. 6. The President, upon the recommendation of the agricultural purposes. As before, Section 61 allowed only the lease of such
Secretary of Agriculture and Commerce, shall from time to time lands to private parties. The government could sell to private parties only
classify the lands of the public domain into – lands falling under Section 59 (d) of CA No. 141, or those lands for non-
agricultural purposes not classified as government reclaimed, foreshore and
marshy disposable lands of the public domain. Foreshore lands, however,
(a) Alienable or disposable,
became inalienable under the 1935 Constitution which only allowed the lease
(b) Timber, and
of these lands to qualified private parties.
(c) Mineral lands,

and may at any time and in like manner transfer such lands from Section 58 of CA No. 141 expressly states that disposable lands of the public
one class to another,53 for the purpose of their administration and domain intended for residential, commercial, industrial or other productive
disposition. purposes other than agricultural "shall be disposed of under the provisions of
this chapter and not otherwise." Under Section 10 of CA No. 141, the term
"disposition" includes lease of the land. Any disposition of government
Sec. 7. For the purposes of the administration and disposition of
reclaimed, foreshore and marshy disposable lands for non-agricultural
alienable or disposable public lands, the President, upon
purposes must comply with Chapter IX, Title III of CA No. 141,54 unless a
recommendation by the Secretary of Agriculture and Commerce,
subsequent law amended or repealed these provisions.
shall from time to time declare what lands are open to disposition
or concession under this Act.
In his concurring opinion in the landmark case of Republic Real Estate
Corporation v. Court of Appeals,55 Justice Reynato S. Puno summarized
Sec. 8. Only those lands shall be declared open to disposition or
succinctly the law on this matter, as follows:
concession which have been officially delimited and
classified and, when practicable, surveyed, and which have not
been reserved for public or quasi-public uses, nor appropriated by "Foreshore lands are lands of public dominion intended for public
the Government, nor in any manner become private property, nor use. So too are lands reclaimed by the government by dredging,
those on which a private right authorized and recognized by this filling, or other means. Act 1654 mandated that the control and
Act or any other valid law may be claimed, or which, having been disposition of the foreshore and lands under water remained in the
reserved or appropriated, have ceased to be so. x x x." national government. Said law allowed only the 'leasing' of
reclaimed land. The Public Land Acts of 1919 and 1936 also
declared that the foreshore and lands reclaimed by the government
Thus, before the government could alienate or dispose of lands of the public
were to be "disposed of to private parties by lease only and not
domain, the President must first officially classify these lands as alienable or
otherwise." Before leasing, however, the Governor-General, upon
disposable, and then declare them open to disposition or concession. There
recommendation of the Secretary of Agriculture and Natural
must be no law reserving these lands for public or quasi-public uses.
Resources, had first to determine that the land reclaimed was not
necessary for the public service. This requisite must have been met
The salient provisions of CA No. 141, on government reclaimed, foreshore before the land could be disposed of. But even then, the foreshore
and marshy lands of the public domain, are as follows: and lands under water were not to be alienated and sold to
private parties. The disposition of the reclaimed land was only by
lease. The land remained property of the State." (Emphasis
"Sec. 58. Any tract of land of the public domain which, being supplied)
neither timber nor mineral land, is intended to be used for
residential purposes or for commercial, industrial, or other
As observed by Justice Puno in his concurring opinion, "Commonwealth Act Sec. 67. The lease or sale shall be made by oral bidding; and
No. 141 has remained in effect at present." adjudication shall be made to the highest bidder. x x x."
(Emphasis supplied)
The State policy prohibiting the sale to private parties of government
reclaimed, foreshore and marshy alienable lands of the public domain, first Thus, CA No. 141 mandates the Government to put to public auction all
implemented in 1907 was thus reaffirmed in CA No. 141 after the 1935 leases or sales of alienable or disposable lands of the public domain.58
Constitution took effect. The prohibition on the sale of foreshore lands,
however, became a constitutional edict under the 1935 Constitution.
Like Act No. 1654 and Act No. 2874 before it, CA No. 141 did not repeal
Foreshore lands became inalienable as natural resources of the State, unless
Section 5 of the Spanish Law of Waters of 1866. Private parties could still
reclaimed by the government and classified as agricultural lands of the public
reclaim portions of the sea with government permission. However,
domain, in which case they would fall under the classification of government
the reclaimed land could become private land only if classified as alienable
reclaimed lands.
agricultural land of the public domain open to disposition under CA No.
141. The 1935 Constitution prohibited the alienation of all natural resources
After the effectivity of the 1935 Constitution, government reclaimed and except public agricultural lands.
marshy disposable lands of the public domain continued to be only leased and
not sold to private parties.56 These lands remained sui generis, as the only
The Civil Code of 1950
alienable or disposable lands of the public domain the government could not
sell to private parties.
The Civil Code of 1950 readopted substantially the definition of property of
public dominion found in the Civil Code of 1889. Articles 420 and 422 of the
Since then and until now, the only way the government can sell to private
Civil Code of 1950 state that –
parties government reclaimed and marshy disposable lands of the public
domain is for the legislature to pass a law authorizing such sale. CA No. 141
does not authorize the President to reclassify government reclaimed and "Art. 420. The following things are property of public dominion:
marshy lands into other non-agricultural lands under Section 59 (d). Lands
classified under Section 59 (d) are the only alienable or disposable lands for
(1) Those intended for public use, such as roads, canals, rivers,
non-agricultural purposes that the government could sell to private parties.
torrents, ports and bridges constructed by the State, banks, shores,
roadsteads, and others of similar character;
Moreover, Section 60 of CA No. 141 expressly requires congressional
authority before lands under Section 59 that the government previously
(2) Those which belong to the State, without being for public use,
transferred to government units or entities could be sold to private parties.
and are intended for some public service or for the development of
Section 60 of CA No. 141 declares that –
the national wealth.

"Sec. 60. x x x The area so leased or sold shall be such as shall, in


x x x.
the judgment of the Secretary of Agriculture and Natural
Resources, be reasonably necessary for the purposes for which
such sale or lease is requested, and shall not exceed one hundred Art. 422. Property of public dominion, when no longer intended
and forty-four hectares: Provided, however, That this limitation for public use or for public service, shall form part of the
shall not apply to grants, donations, or transfers made to a patrimonial property of the State."
province, municipality or branch or subdivision of the Government
for the purposes deemed by said entities conducive to the public
interest; but the land so granted, donated, or transferred to a Again, the government must formally declare that the property of public
province, municipality or branch or subdivision of the dominion is no longer needed for public use or public service, before the same
Government shall not be alienated, encumbered, or otherwise could be classified as patrimonial property of the State.59 In the case of
disposed of in a manner affecting its title, except when government reclaimed and marshy lands of the public domain, the declaration
authorized by Congress: x x x." (Emphasis supplied) of their being disposable, as well as the manner of their disposition, is
governed by the applicable provisions of CA No. 141.

The congressional authority required in Section 60 of CA No. 141 mirrors the


legislative authority required in Section 56 of Act No. 2874. Like the Civil Code of 1889, the Civil Code of 1950 included as property of
public dominion those properties of the State which, without being for public
use, are intended for public service or the "development of the national
One reason for the congressional authority is that Section 60 of CA No. 141 wealth." Thus, government reclaimed and marshy lands of the State, even if
exempted government units and entities from the maximum area of public not employed for public use or public service, if developed to enhance the
lands that could be acquired from the State. These government units and national wealth, are classified as property of public dominion.
entities should not just turn around and sell these lands to private parties in
violation of constitutional or statutory limitations. Otherwise, the transfer of
lands for non-agricultural purposes to government units and entities could be Dispositions under the 1973 Constitution
used to circumvent constitutional limitations on ownership of alienable or
disposable lands of the public domain. In the same manner, such transfers The 1973 Constitution, which took effect on January 17, 1973, likewise
could also be used to evade the statutory prohibition in CA No. 141 on the adopted the Regalian doctrine. Section 8, Article XIV of the 1973
sale of government reclaimed and marshy lands of the public domain to Constitution stated that –
private parties. Section 60 of CA No. 141 constitutes by operation of law a
lien on these lands.57
"Sec. 8. All lands of the public domain, waters, minerals, coal,
petroleum and other mineral oils, all forces of potential energy,
In case of sale or lease of disposable lands of the public domain falling under fisheries, wildlife, and other natural resources of the Philippines
Section 59 of CA No. 141, Sections 63 and 67 require a public bidding. belong to the State. With the exception of agricultural, industrial
Sections 63 and 67 of CA No. 141 provide as follows: or commercial, residential, and resettlement lands of the public
domain, natural resources shall not be alienated, and no license,
"Sec. 63. Whenever it is decided that lands covered by this chapter concession, or lease for the exploration, development, exploitation,
are not needed for public purposes, the Director of Lands shall ask or utilization of any of the natural resources shall be granted for a
the Secretary of Agriculture and Commerce (now the Secretary of period exceeding twenty-five years, renewable for not more than
Natural Resources) for authority to dispose of the same. Upon twenty-five years, except as to water rights for irrigation, water
receipt of such authority, the Director of Lands shall give notice by supply, fisheries, or industrial uses other than the development of
public advertisement in the same manner as in the case of leases or water power, in which cases, beneficial use may be the measure
sales of agricultural public land, x x x. and the limit of the grant." (Emphasis supplied)
The 1973 Constitution prohibited the alienation of all natural resources with xxx
the exception of "agricultural, industrial or commercial, residential, and
resettlement lands of the public domain." In contrast, the 1935 Constitution
(i) To hold lands of the public domain in excess of the area
barred the alienation of all natural resources except "public agricultural
permitted to private corporations by statute.
lands." However, the term "public agricultural lands" in the 1935 Constitution
encompassed industrial, commercial, residential and resettlement lands of the
public domain.60 If the land of public domain were neither timber nor mineral (j) To reclaim lands and to construct work across, or otherwise,
land, it would fall under the classification of agricultural land of the public any stream, watercourse, canal, ditch, flume x x x.
domain. Both the 1935 and 1973 Constitutions, therefore, prohibited the
alienation of all natural resources except agricultural lands of the public
xxx
domain.

(o) To perform such acts and exercise such functions as may be


The 1973 Constitution, however, limited the alienation of lands of the public
necessary for the attainment of the purposes and objectives herein
domain to individuals who were citizens of the Philippines. Private
specified." (Emphasis supplied)
corporations, even if wholly owned by Philippine citizens, were no longer
allowed to acquire alienable lands of the public domain unlike in the 1935
Constitution. Section 11, Article XIV of the 1973 Constitution declared that – PD No. 1084 authorizes PEA to reclaim both foreshore and submerged areas
of the public domain. Foreshore areas are those covered and uncovered by the
ebb and flow of the tide.61 Submerged areas are those permanently under
"Sec. 11. The Batasang Pambansa, taking into account
water regardless of the ebb and flow of the tide.62 Foreshore and submerged
conservation, ecological, and development requirements of the
areas indisputably belong to the public domain63 and are inalienable unless
natural resources, shall determine by law the size of land of the
reclaimed, classified as alienable lands open to disposition, and further
public domain which may be developed, held or acquired by, or
declared no longer needed for public service.
leased to, any qualified individual, corporation, or association, and
the conditions therefor. No private corporation or association
may hold alienable lands of the public domain except by lease not The ban in the 1973 Constitution on private corporations from acquiring
to exceed one thousand hectares in area nor may any citizen hold alienable lands of the public domain did not apply to PEA since it was then,
such lands by lease in excess of five hundred hectares or acquire and until today, a fully owned government corporation. The constitutional ban
by purchase, homestead or grant, in excess of twenty-four hectares. applied then, as it still applies now, only to "private corporations and
No private corporation or association may hold by lease, associations." PD No. 1084 expressly empowers PEA "to hold lands of the
concession, license or permit, timber or forest lands and other public domain" even "in excess of the area permitted to private corporations
timber or forest resources in excess of one hundred thousand by statute." Thus, PEA can hold title to private lands, as well as title to lands
hectares. However, such area may be increased by the Batasang of the public domain.
Pambansa upon recommendation of the National Economic and
Development Authority." (Emphasis supplied)
In order for PEA to sell its reclaimed foreshore and submerged alienable lands
of the public domain, there must be legislative authority empowering PEA to
Thus, under the 1973 Constitution, private corporations could hold alienable sell these lands. This legislative authority is necessary in view of Section 60
lands of the public domain only through lease. Only individuals could now of CA No.141, which states –
acquire alienable lands of the public domain, and private corporations
became absolutely barred from acquiring any kind of alienable land of the
public domain. The constitutional ban extended to all kinds of alienable lands "Sec. 60. x x x; but the land so granted, donated or transferred to a
of the public domain, while the statutory ban under CA No. 141 applied only province, municipality, or branch or subdivision of the
to government reclaimed, foreshore and marshy alienable lands of the public Government shall not be alienated, encumbered or otherwise
domain. disposed of in a manner affecting its title, except when authorized
by Congress; x x x." (Emphasis supplied)

PD No. 1084 Creating the Public Estates Authority


Without such legislative authority, PEA could not sell but only lease its
reclaimed foreshore and submerged alienable lands of the public domain.
On February 4, 1977, then President Ferdinand Marcos issued Presidential Nevertheless, any legislative authority granted to PEA to sell its reclaimed
Decree No. 1084 creating PEA, a wholly government owned and controlled alienable lands of the public domain would be subject to the constitutional ban
corporation with a special charter. Sections 4 and 8 of PD No. 1084, vests on private corporations from acquiring alienable lands of the public domain.
PEA with the following purposes and powers: Hence, such legislative authority could only benefit private individuals.

"Sec. 4. Purpose. The Authority is hereby created for the following Dispositions under the 1987 Constitution
purposes:
The 1987 Constitution, like the 1935 and 1973 Constitutions before it, has
(a) To reclaim land, including foreshore and submerged areas, adopted the Regalian doctrine. The 1987 Constitution declares that all natural
by dredging, filling or other means, or to acquire reclaimed land; resources are "owned by the State," and except for alienable agricultural lands
of the public domain, natural resources cannot be alienated. Sections 2 and 3,
Article XII of the 1987 Constitution state that –
(b) To develop, improve, acquire, administer, deal in, subdivide,
dispose, lease and sell any and all kinds of lands, buildings,
estates and other forms of real property, owned, managed, "Section 2. All lands of the public domain, waters, minerals, coal,
controlled and/or operated by the government; petroleum and other mineral oils, all forces of potential energy,
fisheries, forests or timber, wildlife, flora and fauna, and other
natural resources are owned by the State. With the exception of
(c) To provide for, operate or administer such service as may be
agricultural lands, all other natural resources shall not be
necessary for the efficient, economical and beneficial utilization of
alienated. The exploration, development, and utilization of natural
the above properties.
resources shall be under the full control and supervision of the
State. x x x.
Sec. 5. Powers and functions of the Authority. The Authority shall,
in carrying out the purposes for which it is created, have the
Section 3. Lands of the public domain are classified into
following powers and functions:
agricultural, forest or timber, mineral lands, and national parks.
Agricultural lands of the public domain may be further classified
(a)To prescribe its by-laws. by law according to the uses which they may be
devoted. Alienable lands of the public domain shall be limited to
agricultural lands. Private corporations or associations may not preventing the break-up of farmlands. If the farmland is registered in the name
hold such alienable lands of the public domain except by lease, of a corporation, upon the death of the owner, his heirs would inherit shares in
for a period not exceeding twenty-five years, renewable for not the corporation instead of subdivided parcels of the farmland. This would
more than twenty-five years, and not to exceed one thousand prevent the continuing break-up of farmlands into smaller and smaller plots
hectares in area. Citizens of the Philippines may lease not more from one generation to the next.
than five hundred hectares, or acquire not more than twelve
hectares thereof by purchase, homestead, or grant.
In actual practice, the constitutional ban strengthens the constitutional
limitation on individuals from acquiring more than the allowed area of
Taking into account the requirements of conservation, ecology, and alienable lands of the public domain. Without the constitutional ban,
development, and subject to the requirements of agrarian reform, individuals who already acquired the maximum area of alienable lands of the
the Congress shall determine, by law, the size of lands of the public domain could easily set up corporations to acquire more alienable
public domain which may be acquired, developed, held, or leased public lands. An individual could own as many corporations as his means
and the conditions therefor." (Emphasis supplied) would allow him. An individual could even hide his ownership of a
corporation by putting his nominees as stockholders of the corporation. The
corporation is a convenient vehicle to circumvent the constitutional limitation
The 1987 Constitution continues the State policy in the 1973 Constitution
on acquisition by individuals of alienable lands of the public domain.
banning private corporations from acquiring any kind of alienable land of
the public domain. Like the 1973 Constitution, the 1987 Constitution allows
private corporations to hold alienable lands of the public domain only The constitutional intent, under the 1973 and 1987 Constitutions, is to transfer
through lease. As in the 1935 and 1973 Constitutions, the general law ownership of only a limited area of alienable land of the public domain to a
governing the lease to private corporations of reclaimed, foreshore and qualified individual. This constitutional intent is safeguarded by the provision
marshy alienable lands of the public domain is still CA No. 141. prohibiting corporations from acquiring alienable lands of the public domain,
since the vehicle to circumvent the constitutional intent is removed. The
available alienable public lands are gradually decreasing in the face of an
The Rationale behind the Constitutional Ban
ever-growing population. The most effective way to insure faithful adherence
to this constitutional intent is to grant or sell alienable lands of the public
The rationale behind the constitutional ban on corporations from acquiring, domain only to individuals. This, it would seem, is the practical benefit arising
except through lease, alienable lands of the public domain is not well from the constitutional ban.
understood. During the deliberations of the 1986 Constitutional Commission,
the commissioners probed the rationale behind this ban, thus:
The Amended Joint Venture Agreement

"FR. BERNAS: Mr. Vice-President, my questions have reference


The subject matter of the Amended JVA, as stated in its second Whereas
to page 3, line 5 which says:
clause, consists of three properties, namely:

`No private corporation or association may hold alienable lands of


1. "[T]hree partially reclaimed and substantially eroded islands
the public domain except by lease, not to exceed one thousand
along Emilio Aguinaldo Boulevard in Paranaque and Las Pinas,
hectares in area.'
Metro Manila, with a combined titled area of 1,578,441 square
meters;"
If we recall, this provision did not exist under the 1935
Constitution, but this was introduced in the 1973 Constitution. In
2. "[A]nother area of 2,421,559 square meters contiguous to the
effect, it prohibits private corporations from acquiring alienable
three islands;" and
public lands. But it has not been very clear in jurisprudence what
the reason for this is. In some of the cases decided in 1982 and
1983, it was indicated that the purpose of this is to prevent large 3. "[A]t AMARI's option as approved by PEA, an additional 350
landholdings. Is that the intent of this provision? hectares more or less to regularize the configuration of the
reclaimed area."65
MR. VILLEGAS: I think that is the spirit of the provision.
PEA confirms that the Amended JVA involves "the development of the
Freedom Islands and further reclamation of about 250 hectares x x x," plus an
FR. BERNAS: In existing decisions involving the Iglesia ni Cristo,
option "granted to AMARI to subsequently reclaim another 350 hectares x x
there were instances where the Iglesia ni Cristo was not allowed to
x."66
acquire a mere 313-square meter land where a chapel stood
because the Supreme Court said it would be in violation of this."
(Emphasis supplied) In short, the Amended JVA covers a reclamation area of 750 hectares. Only
157.84 hectares of the 750-hectare reclamation project have been
reclaimed, and the rest of the 592.15 hectares are still submerged areas
In Ayog v. Cusi,64 the Court explained the rationale behind this constitutional
forming part of Manila Bay.
ban in this way:

Under the Amended JVA, AMARI will reimburse PEA the sum of
"Indeed, one purpose of the constitutional prohibition against
P1,894,129,200.00 for PEA's "actual cost" in partially reclaiming the Freedom
purchases of public agricultural lands by private corporations is to
Islands. AMARI will also complete, at its own expense, the reclamation of the
equitably diffuse land ownership or to encourage 'owner-
Freedom Islands. AMARI will further shoulder all the reclamation costs of all
cultivatorship and the economic family-size farm' and to prevent a
the other areas, totaling 592.15 hectares, still to be reclaimed. AMARI and
recurrence of cases like the instant case. Huge landholdings by
PEA will share, in the proportion of 70 percent and 30 percent, respectively,
corporations or private persons had spawned social unrest."
the total net usable area which is defined in the Amended JVA as the total
reclaimed area less 30 percent earmarked for common areas. Title to
However, if the constitutional intent is to prevent huge landholdings, the AMARI's share in the net usable area, totaling 367.5 hectares, will be issued
Constitution could have simply limited the size of alienable lands of the in the name of AMARI. Section 5.2 (c) of the Amended JVA provides that –
public domain that corporations could acquire. The Constitution could have
followed the limitations on individuals, who could acquire not more than 24
"x x x, PEA shall have the duty to execute without delay the
hectares of alienable lands of the public domain under the 1973 Constitution,
necessary deed of transfer or conveyance of the title pertaining to
and not more than 12 hectares under the 1987 Constitution.
AMARI's Land share based on the Land Allocation Plan. PEA,
when requested in writing by AMARI, shall then cause the
If the constitutional intent is to encourage economic family-size farms, issuance and delivery of the proper certificates of title covering
placing the land in the name of a corporation would be more effective in AMARI's Land Share in the name of AMARI, x x x; provided,
that if more than seventy percent (70%) of the titled area at any "D. Conclusion
given time pertains to AMARI, PEA shall deliver to AMARI only
seventy percent (70%) of the titles pertaining to AMARI, until
Reclaimed lands are lands of the public domain. However, by
such time when a corresponding proportionate area of additional
statutory authority, the rights of ownership and disposition over
land pertaining to PEA has been titled." (Emphasis supplied)
reclaimed lands have been transferred to PEA, by virtue of which
PEA, as owner, may validly convey the same to any qualified
Indisputably, under the Amended JVA AMARI will acquire and own a person without violating the Constitution or any statute.
maximum of 367.5 hectares of reclaimed land which will be titled in its
name.
The constitutional provision prohibiting private corporations from
holding public land, except by lease (Sec. 3, Art. XVII, 70 1987
To implement the Amended JVA, PEA delegated to the unincorporated PEA- Constitution), does not apply to reclaimed lands whose ownership
AMARI joint venture PEA's statutory authority, rights and privileges to has passed on to PEA by statutory grant."
reclaim foreshore and submerged areas in Manila Bay. Section 3.2.a of the
Amended JVA states that –
Under Section 2, Article XII of the 1987 Constitution, the foreshore and
submerged areas of Manila Bay are part of the "lands of the public domain,
"PEA hereby contributes to the joint venture its rights and waters x x x and other natural resources" and consequently "owned by the
privileges to perform Rawland Reclamation and Horizontal State." As such, foreshore and submerged areas "shall not be alienated,"
Development as well as own the Reclamation Area, thereby unless they are classified as "agricultural lands" of the public domain. The
granting the Joint Venture the full and exclusive right, authority mere reclamation of these areas by PEA does not convert these inalienable
and privilege to undertake the Project in accordance with the natural resources of the State into alienable or disposable lands of the public
Master Development Plan." domain. There must be a law or presidential proclamation officially
classifying these reclaimed lands as alienable or disposable and open to
disposition or concession. Moreover, these reclaimed lands cannot be
The Amended JVA is the product of a renegotiation of the original JVA dated
classified as alienable or disposable if the law has reserved them for some
April 25, 1995 and its supplemental agreement dated August 9, 1995.
public or quasi-public use.71

The Threshold Issue


Section 8 of CA No. 141 provides that "only those lands shall be declared
open to disposition or concession which have been officially delimited and
The threshold issue is whether AMARI, a private corporation, can acquire and classified."72 The President has the authority to classify inalienable lands of
own under the Amended JVA 367.5 hectares of reclaimed foreshore and the public domain into alienable or disposable lands of the public domain,
submerged areas in Manila Bay in view of Sections 2 and 3, Article XII of the pursuant to Section 6 of CA No. 141. In Laurel vs. Garcia,73 the Executive
1987 Constitution which state that: Department attempted to sell the Roppongi property in Tokyo, Japan, which
was acquired by the Philippine Government for use as the Chancery of the
Philippine Embassy. Although the Chancery had transferred to another
"Section 2. All lands of the public domain, waters, minerals, coal, location thirteen years earlier, the Court still ruled that, under Article 422 74 of
petroleum, and other mineral oils, all forces of potential energy, the Civil Code, a property of public dominion retains such character until
fisheries, forests or timber, wildlife, flora and fauna, and other formally declared otherwise. The Court ruled that –
natural resources are owned by the State. With the exception of
agricultural lands, all other natural resources shall not be
alienated. x x x. "The fact that the Roppongi site has not been used for a long time
for actual Embassy service does not automatically convert it to
patrimonial property. Any such conversion happens only if the
xxx property is withdrawn from public use (Cebu Oxygen and
Acetylene Co. v. Bercilles, 66 SCRA 481 [1975]. A property
Section 3. x x x Alienable lands of the public domain shall be continues to be part of the public domain, not available for
limited to agricultural lands. Private corporations or associations private appropriation or ownership 'until there is a formal
may not hold such alienable lands of the public domain except declaration on the part of the government to withdraw it from
by lease, x x x."(Emphasis supplied) being such' (Ignacio v. Director of Lands, 108 Phil. 335 [1960]."
(Emphasis supplied)
Classification of Reclaimed Foreshore and Submerged Areas
PD No. 1085, issued on February 4, 1977, authorized the issuance of special
land patents for lands reclaimed by PEA from the foreshore or submerged
PEA readily concedes that lands reclaimed from foreshore or submerged areas areas of Manila Bay. On January 19, 1988 then President Corazon C. Aquino
of Manila Bay are alienable or disposable lands of the public domain. In its issued Special Patent No. 3517 in the name of PEA for the 157.84 hectares
Memorandum,67 PEA admits that – comprising the partially reclaimed Freedom Islands. Subsequently, on April 9,
1999 the Register of Deeds of the Municipality of Paranaque issued TCT Nos.
"Under the Public Land Act (CA 141, as amended), reclaimed 7309, 7311 and 7312 in the name of PEA pursuant to Section 103 of PD No.
lands are classified as alienable and disposable lands of the 1529 authorizing the issuance of certificates of title corresponding to land
public domain: patents. To this day, these certificates of title are still in the name of PEA.

'Sec. 59. The lands disposable under this title shall be PD No. 1085, coupled with President Aquino's actual issuance of a special
classified as follows: patent covering the Freedom Islands, is equivalent to an official proclamation
classifying the Freedom Islands as alienable or disposable lands of the public
domain. PD No. 1085 and President Aquino's issuance of a land patent also
(a) Lands reclaimed by the government by dredging, constitute a declaration that the Freedom Islands are no longer needed for
filling, or other means; public service. The Freedom Islands are thus alienable or disposable lands
of the public domain, open to disposition or concession to qualified parties.
x x x.'" (Emphasis supplied)
At the time then President Aquino issued Special Patent No. 3517, PEA had
Likewise, the Legal Task Force68 constituted under Presidential already reclaimed the Freedom Islands although subsequently there were
Administrative Order No. 365 admitted in its Report and Recommendation to partial erosions on some areas. The government had also completed the
then President Fidel V. Ramos, "[R]eclaimed lands are classified as necessary surveys on these islands. Thus, the Freedom Islands were no longer
alienable and disposable lands of the public domain."69 The Legal Task part of Manila Bay but part of the land mass. Section 3, Article XII of the
Force concluded that – 1987 Constitution classifies lands of the public domain into "agricultural,
forest or timber, mineral lands, and national parks." Being neither timber,
mineral, nor national park lands, the reclaimed Freedom Islands necessarily consisting of portions of the reclaimed land, subject to the constitutional ban
fall under the classification of agricultural lands of the public domain. Under on private corporations from acquiring alienable lands of the public domain.
the 1987 Constitution, agricultural lands of the public domain are the only The reclaimed land can be used as payment in kind only if the reclaimed land
natural resources that the State may alienate to qualified private parties. All is first classified as alienable or disposable land open to disposition, and then
other natural resources, such as the seas or bays, are "waters x x x owned by declared no longer needed for public service.
the State" forming part of the public domain, and are inalienable pursuant to
Section 2, Article XII of the 1987 Constitution.
The Amended JVA covers not only the Freedom Islands, but also an
additional 592.15 hectares which are still submerged and forming part of
AMARI claims that the Freedom Islands are private lands because CDCP, Manila Bay. There is no legislative or Presidential act classifying these
then a private corporation, reclaimed the islands under a contract dated submerged areas as alienable or disposable lands of the public domain open
November 20, 1973 with the Commissioner of Public Highways. AMARI, to disposition. These submerged areas are not covered by any patent or
citing Article 5 of the Spanish Law of Waters of 1866, argues that "if the certificate of title. There can be no dispute that these submerged areas form
ownership of reclaimed lands may be given to the party constructing the part of the public domain, and in their present state are inalienable and
works, then it cannot be said that reclaimed lands are lands of the public outside the commerce of man. Until reclaimed from the sea, these submerged
domain which the State may not alienate."75 Article 5 of the Spanish Law of areas are, under the Constitution, "waters x x x owned by the State," forming
Waters reads as follows: part of the public domain and consequently inalienable. Only when actually
reclaimed from the sea can these submerged areas be classified as public
agricultural lands, which under the Constitution are the only natural resources
"Article 5. Lands reclaimed from the sea in consequence of works
that the State may alienate. Once reclaimed and transformed into public
constructed by the State, or by the provinces, pueblos or private
agricultural lands, the government may then officially classify these lands as
persons, with proper permission, shall become the property of the
alienable or disposable lands open to disposition. Thereafter, the government
party constructing such works, unless otherwise provided by the
may declare these lands no longer needed for public service. Only then can
terms of the grant of authority." (Emphasis supplied)
these reclaimed lands be considered alienable or disposable lands of the public
domain and within the commerce of man.
Under Article 5 of the Spanish Law of Waters of 1866, private parties could
reclaim from the sea only with "proper permission" from the State. Private
The classification of PEA's reclaimed foreshore and submerged lands into
parties could own the reclaimed land only if not "otherwise provided by the
alienable or disposable lands open to disposition is necessary because PEA is
terms of the grant of authority." This clearly meant that no one could reclaim
tasked under its charter to undertake public services that require the use of
from the sea without permission from the State because the sea is property of
lands of the public domain. Under Section 5 of PD No. 1084, the functions of
public dominion. It also meant that the State could grant or withhold
PEA include the following: "[T]o own or operate railroads, tramways and
ownership of the reclaimed land because any reclaimed land, like the sea from
other kinds of land transportation, x x x; [T]o construct, maintain and operate
which it emerged, belonged to the State. Thus, a private person reclaiming
such systems of sanitary sewers as may be necessary; [T]o construct, maintain
from the sea without permission from the State could not acquire ownership of
and operate such storm drains as may be necessary." PEA is empowered to
the reclaimed land which would remain property of public dominion like the
issue "rules and regulations as may be necessary for the proper use by private
sea it replaced.76 Article 5 of the Spanish Law of Waters of 1866 adopted the
parties of any or all of the highways, roads, utilities, buildings and/or any of
time-honored principle of land ownership that "all lands that were not
its properties and to impose or collect fees or tolls for their use." Thus, part of
acquired from the government, either by purchase or by grant, belong to the
the reclaimed foreshore and submerged lands held by the PEA would actually
public domain."77
be needed for public use or service since many of the functions imposed on
PEA by its charter constitute essential public services.
Article 5 of the Spanish Law of Waters must be read together with laws
subsequently enacted on the disposition of public lands. In particular, CA No.
Moreover, Section 1 of Executive Order No. 525 provides that PEA "shall be
141 requires that lands of the public domain must first be classified as
primarily responsible for integrating, directing, and coordinating all
alienable or disposable before the government can alienate them. These lands
reclamation projects for and on behalf of the National Government." The
must not be reserved for public or quasi-public purposes.78 Moreover, the
same section also states that "[A]ll reclamation projects shall be approved by
contract between CDCP and the government was executed after the effectivity
the President upon recommendation of the PEA, and shall be undertaken by
of the 1973 Constitution which barred private corporations from acquiring any
the PEA or through a proper contract executed by it with any person or entity;
kind of alienable land of the public domain. This contract could not have
x x x." Thus, under EO No. 525, in relation to PD No. 3-A and PD No.1084,
converted the Freedom Islands into private lands of a private corporation.
PEA became the primary implementing agency of the National Government to
reclaim foreshore and submerged lands of the public domain. EO No. 525
Presidential Decree No. 3-A, issued on January 11, 1973, revoked all laws recognized PEA as the government entity "to undertake the reclamation of
authorizing the reclamation of areas under water and revested solely in the lands and ensure their maximum utilization in promoting public welfare and
National Government the power to reclaim lands. Section 1 of PD No. 3-A interests."79 Since large portions of these reclaimed lands would obviously be
declared that – needed for public service, there must be a formal declaration segregating
reclaimed lands no longer needed for public service from those still needed for
public service.1âwphi1.nêt
"The provisions of any law to the contrary notwithstanding, the
reclamation of areas under water, whether foreshore or inland,
shall be limited to the National Government or any person Section 3 of EO No. 525, by declaring that all lands reclaimed by PEA "shall
authorized by it under a proper contract. (Emphasis supplied) belong to or be owned by the PEA," could not automatically operate to
classify inalienable lands into alienable or disposable lands of the public
domain. Otherwise, reclaimed foreshore and submerged lands of the public
x x x." domain would automatically become alienable once reclaimed by PEA,
whether or not classified as alienable or disposable.
PD No. 3-A repealed Section 5 of the Spanish Law of Waters of 1866 because
reclamation of areas under water could now be undertaken only by the The Revised Administrative Code of 1987, a later law than either PD No.
National Government or by a person contracted by the National Government. 1084 or EO No. 525, vests in the Department of Environment and Natural
Private parties may reclaim from the sea only under a contract with the Resources ("DENR" for brevity) the following powers and functions:
National Government, and no longer by grant or permission as provided in
Section 5 of the Spanish Law of Waters of 1866.
"Sec. 4. Powers and Functions. The Department shall:
Executive Order No. 525, issued on February 14, 1979, designated PEA as the
National Government's implementing arm to undertake "all reclamation (1) x x x
projects of the government," which "shall be undertaken by the PEA or
through a proper contract executed by it with any person or entity." Under
xxx
such contract, a private party receives compensation for reclamation services
rendered to PEA. Payment to the contractor may be in cash, or in kind
(4) Exercise supervision and control over forest lands, alienable admits that reclaimed lands transferred to a branch or subdivision of the
and disposable public lands, mineral resources and, in the process government "shall not be alienated, encumbered, or otherwise disposed of in a
of exercising such control, impose appropriate taxes, fees, charges, manner affecting its title, except when authorized by Congress: x x
rentals and any such form of levy and collect such revenues for the x."85 (Emphasis by PEA)
exploration, development, utilization or gathering of such
resources;
In Laurel vs. Garcia,86 the Court cited Section 48 of the Revised
Administrative Code of 1987, which states that –
xxx
"Sec. 48. Official Authorized to Convey Real Property. Whenever
(14) Promulgate rules, regulations and guidelines on the real property of the Government is authorized by law to be
issuance of licenses, permits, concessions, lease agreements and conveyed, the deed of conveyance shall be executed in behalf of
such other privileges concerning the development, exploration the government by the following: x x x."
and utilization of the country's marine, freshwater, and brackish
water and over all aquatic resources of the country and shall
Thus, the Court concluded that a law is needed to convey any real property
continue to oversee, supervise and police our natural resources;
belonging to the Government. The Court declared that -
cancel or cause to cancel such privileges upon failure, non-
compliance or violations of any regulation, order, and for all other
causes which are in furtherance of the conservation of natural "It is not for the President to convey real property of the
resources and supportive of the national interest; government on his or her own sole will. Any such conveyance
must be authorized and approved by a law enacted by the
Congress. It requires executive and legislative concurrence."
(15) Exercise exclusive jurisdiction on the management and
(Emphasis supplied)
disposition of all lands of the public domain and serve as the sole
agency responsible for classification, sub-classification, surveying
and titling of lands in consultation with appropriate PEA contends that PD No. 1085 and EO No. 525 constitute the legislative
agencies."80 (Emphasis supplied) authority allowing PEA to sell its reclaimed lands. PD No. 1085, issued on
February 4, 1977, provides that –
As manager, conservator and overseer of the natural resources of the State,
DENR exercises "supervision and control over alienable and disposable public "The land reclaimed in the foreshore and offshore area of
lands." DENR also exercises "exclusive jurisdiction on the management and Manila Bay pursuant to the contract for the reclamation and
disposition of all lands of the public domain." Thus, DENR decides whether construction of the Manila-Cavite Coastal Road Project between
areas under water, like foreshore or submerged areas of Manila Bay, should be the Republic of the Philippines and the Construction and
reclaimed or not. This means that PEA needs authorization from DENR Development Corporation of the Philippines dated November 20,
before PEA can undertake reclamation projects in Manila Bay, or in any part 1973 and/or any other contract or reclamation covering the same
of the country. area is hereby transferred, conveyed and assigned to the
ownership and administration of the Public Estates
Authority established pursuant to PD No. 1084; Provided,
DENR also exercises exclusive jurisdiction over the disposition of all lands of
however, That the rights and interests of the Construction and
the public domain. Hence, DENR decides whether reclaimed lands of PEA
Development Corporation of the Philippines pursuant to the
should be classified as alienable under Sections 681 and 782 of CA No. 141.
aforesaid contract shall be recognized and respected.
Once DENR decides that the reclaimed lands should be so classified, it then
recommends to the President the issuance of a proclamation classifying the
lands as alienable or disposable lands of the public domain open to Henceforth, the Public Estates Authority shall exercise the rights
disposition. We note that then DENR Secretary Fulgencio S. Factoran, Jr. and assume the obligations of the Republic of the Philippines
countersigned Special Patent No. 3517 in compliance with the Revised (Department of Public Highways) arising from, or incident to, the
Administrative Code and Sections 6 and 7 of CA No. 141. aforesaid contract between the Republic of the Philippines and the
Construction and Development Corporation of the Philippines.
In short, DENR is vested with the power to authorize the reclamation of areas
under water, while PEA is vested with the power to undertake the physical In consideration of the foregoing transfer and assignment, the
reclamation of areas under water, whether directly or through private Public Estates Authority shall issue in favor of the Republic of the
contractors. DENR is also empowered to classify lands of the public domain Philippines the corresponding shares of stock in said entity with an
into alienable or disposable lands subject to the approval of the President. On issued value of said shares of stock (which) shall be deemed fully
the other hand, PEA is tasked to develop, sell or lease the reclaimed alienable paid and non-assessable.
lands of the public domain.
The Secretary of Public Highways and the General Manager of the
Clearly, the mere physical act of reclamation by PEA of foreshore or Public Estates Authority shall execute such contracts or
submerged areas does not make the reclaimed lands alienable or disposable agreements, including appropriate agreements with the
lands of the public domain, much less patrimonial lands of PEA. Likewise, the Construction and Development Corporation of the Philippines, as
mere transfer by the National Government of lands of the public domain to may be necessary to implement the above.
PEA does not make the lands alienable or disposable lands of the public
domain, much less patrimonial lands of PEA.
Special land patent/patents shall be issued by the Secretary of
Natural Resources in favor of the Public Estates Authority
Absent two official acts – a classification that these lands are alienable or without prejudice to the subsequent transfer to the contractor or
disposable and open to disposition and a declaration that these lands are not his assignees of such portion or portions of the land reclaimed or
needed for public service, lands reclaimed by PEA remain inalienable lands of to be reclaimed as provided for in the above-mentioned contract.
the public domain. Only such an official classification and formal declaration On the basis of such patents, the Land Registration Commission
can convert reclaimed lands into alienable or disposable lands of the public shall issue the corresponding certificate of title." (Emphasis
domain, open to disposition under the Constitution, Title I and Title III 83 of supplied)
CA No. 141 and other applicable laws.84
On the other hand, Section 3 of EO No. 525, issued on February 14, 1979,
PEA's Authority to Sell Reclaimed Lands provides that -

PEA, like the Legal Task Force, argues that as alienable or disposable lands of "Sec. 3. All lands reclaimed by PEA shall belong to or be owned
the public domain, the reclaimed lands shall be disposed of in accordance with by the PEA which shall be responsible for its administration,
CA No. 141, the Public Land Act. PEA, citing Section 60 of CA No. 141, development, utilization or disposition in accordance with the
provisions of Presidential Decree No. 1084. Any and all income consecutive days in any newspaper of general circulation, or
that the PEA may derive from the sale, lease or use of reclaimed where the value of the property does not warrant the expense of
lands shall be used in accordance with the provisions of publication, by notices posted for a like period in at least three
Presidential Decree No. 1084." public places in the locality where the property is to be sold. In the
event that the public auction fails, the property may be sold at a
private sale at such price as may be fixed by the same committee
There is no express authority under either PD No. 1085 or EO No. 525 for
or body concerned and approved by the Commission."
PEA to sell its reclaimed lands. PD No. 1085 merely transferred "ownership
and administration" of lands reclaimed from Manila Bay to PEA, while EO
No. 525 declared that lands reclaimed by PEA "shall belong to or be owned It is only when the public auction fails that a negotiated sale is allowed, in
by PEA." EO No. 525 expressly states that PEA should dispose of its which case the Commission on Audit must approve the selling price.90 The
reclaimed lands "in accordance with the provisions of Presidential Decree No. Commission on Audit implements Section 79 of the Government Auditing
1084," the charter of PEA. Code through Circular No. 89-29691 dated January 27, 1989. This circular
emphasizes that government assets must be disposed of only through public
auction, and a negotiated sale can be resorted to only in case of "failure of
PEA's charter, however, expressly tasks PEA "to develop, improve, acquire,
public auction."
administer, deal in, subdivide, dispose, lease and sell any and all kinds of
lands x x x owned, managed, controlled and/or operated by the
government."87 (Emphasis supplied) There is, therefore, legislative authority At the public auction sale, only Philippine citizens are qualified to bid for
granted to PEA to sell its lands, whether patrimonial or alienable lands of PEA's reclaimed foreshore and submerged alienable lands of the public
the public domain. PEA may sell to private parties its patrimonial domain. Private corporations are barred from bidding at the auction sale of
properties in accordance with the PEA charter free from constitutional any kind of alienable land of the public domain.
limitations. The constitutional ban on private corporations from acquiring
alienable lands of the public domain does not apply to the sale of PEA's
PEA originally scheduled a public bidding for the Freedom Islands on
patrimonial lands.
December 10, 1991. PEA imposed a condition that the winning bidder should
reclaim another 250 hectares of submerged areas to regularize the shape of the
PEA may also sell its alienable or disposable lands of the public domain to Freedom Islands, under a 60-40 sharing of the additional reclaimed areas in
private individuals since, with the legislative authority, there is no longer any favor of the winning bidder.92 No one, however, submitted a bid. On
statutory prohibition against such sales and the constitutional ban does not December 23, 1994, the Government Corporate Counsel advised PEA it could
apply to individuals. PEA, however, cannot sell any of its alienable or sell the Freedom Islands through negotiation, without need of another public
disposable lands of the public domain to private corporations since Section 3, bidding, because of the failure of the public bidding on December 10, 1991.93
Article XII of the 1987 Constitution expressly prohibits such sales. The
legislative authority benefits only individuals. Private corporations remain
However, the original JVA dated April 25, 1995 covered not only the
barred from acquiring any kind of alienable land of the public domain,
Freedom Islands and the additional 250 hectares still to be reclaimed, it also
including government reclaimed lands.
granted an option to AMARI to reclaim another 350 hectares. The original
JVA, a negotiated contract, enlarged the reclamation area to 750
The provision in PD No. 1085 stating that portions of the reclaimed lands hectares.94 The failure of public bidding on December 10, 1991, involving
could be transferred by PEA to the "contractor or his assignees" (Emphasis only 407.84 hectares,95 is not a valid justification for a negotiated sale of 750
supplied) would not apply to private corporations but only to individuals hectares, almost double the area publicly auctioned. Besides, the failure of
because of the constitutional ban. Otherwise, the provisions of PD No. 1085 public bidding happened on December 10, 1991, more than three years before
would violate both the 1973 and 1987 Constitutions. the signing of the original JVA on April 25, 1995. The economic situation in
the country had greatly improved during the intervening period.
The requirement of public auction in the sale of reclaimed lands
Reclamation under the BOT Law and the Local Government Code
Assuming the reclaimed lands of PEA are classified as alienable or disposable
lands open to disposition, and further declared no longer needed for public The constitutional prohibition in Section 3, Article XII of the 1987
service, PEA would have to conduct a public bidding in selling or leasing Constitution is absolute and clear: "Private corporations or associations may
these lands. PEA must observe the provisions of Sections 63 and 67 of CA not hold such alienable lands of the public domain except by lease, x x x."
No. 141 requiring public auction, in the absence of a law exempting PEA Even Republic Act No. 6957 ("BOT Law," for brevity), cited by PEA and
from holding a public auction.88 Special Patent No. 3517 expressly states that AMARI as legislative authority to sell reclaimed lands to private parties,
the patent is issued by authority of the Constitution and PD No. 1084, recognizes the constitutional ban. Section 6 of RA No. 6957 states –
"supplemented by Commonwealth Act No. 141, as amended." This is an
acknowledgment that the provisions of CA No. 141 apply to the disposition of
"Sec. 6. Repayment Scheme. - For the financing, construction,
reclaimed alienable lands of the public domain unless otherwise provided by
operation and maintenance of any infrastructure projects
law. Executive Order No. 654,89 which authorizes PEA "to determine the kind
undertaken through the build-operate-and-transfer arrangement or
and manner of payment for the transfer" of its assets and properties, does not
any of its variations pursuant to the provisions of this Act, the
exempt PEA from the requirement of public auction. EO No. 654 merely
project proponent x x x may likewise be repaid in the form of a
authorizes PEA to decide the mode of payment, whether in kind and in
share in the revenue of the project or other non-monetary
installment, but does not authorize PEA to dispense with public auction.
payments, such as, but not limited to, the grant of a portion or
percentage of the reclaimed land, subject to the constitutional
Moreover, under Section 79 of PD No. 1445, otherwise known as the requirements with respect to the ownership of the land: x x x."
Government Auditing Code, the government is required to sell valuable (Emphasis supplied)
government property through public bidding. Section 79 of PD No. 1445
mandates that –
A private corporation, even one that undertakes the physical reclamation of a
government BOT project, cannot acquire reclaimed alienable lands of the
"Section 79. When government property has become public domain in view of the constitutional ban.
unserviceable for any cause, or is no longer needed, it shall, upon
application of the officer accountable therefor, be inspected by the
Section 302 of the Local Government Code, also mentioned by PEA and
head of the agency or his duly authorized representative in the
AMARI, authorizes local governments in land reclamation projects to pay the
presence of the auditor concerned and, if found to be valueless or
contractor or developer in kind consisting of a percentage of the reclaimed
unsaleable, it may be destroyed in their presence. If found to be
land, to wit:
valuable, it may be sold at public auction to the highest
bidder under the supervision of the proper committee on award or
similar body in the presence of the auditor concerned or other "Section 302. Financing, Construction, Maintenance, Operation,
authorized representative of the Commission, after advertising by and Management of Infrastructure Projects by the Private Sector. x
printed notice in the Official Gazette, or for not less than three xx
xxx Center, Bureau of Medical Services, Department of Health, of the
whole lot, validly sufficient for initial registration under the Land
Registration Act. Such land grant is constitutive of a 'fee simple'
In case of land reclamation or construction of industrial estates, the
title or absolute title in favor of petitioner Mindanao Medical
repayment plan may consist of the grant of a portion or percentage
Center. Thus, Section 122 of the Act, which governs the
of the reclaimed land or the industrial estate constructed."
registration of grants or patents involving public lands, provides
that 'Whenever public lands in the Philippine Islands belonging to
Although Section 302 of the Local Government Code does not contain a the Government of the United States or to the Government of the
proviso similar to that of the BOT Law, the constitutional restrictions on land Philippines are alienated, granted or conveyed to persons or to
ownership automatically apply even though not expressly mentioned in the public or private corporations, the same shall be brought forthwith
Local Government Code. under the operation of this Act (Land Registration Act, Act 496)
and shall become registered lands.'"
Thus, under either the BOT Law or the Local Government Code, the
contractor or developer, if a corporate entity, can only be paid with leaseholds The first four cases cited involve petitions to cancel the land patents and the
on portions of the reclaimed land. If the contractor or developer is an corresponding certificates of titles issued to private parties. These four cases
individual, portions of the reclaimed land, not exceeding 12 hectares96 of non- uniformly hold that the Director of Lands has no jurisdiction over private
agricultural lands, may be conveyed to him in ownership in view of the lands or that upon issuance of the certificate of title the land automatically
legislative authority allowing such conveyance. This is the only way these comes under the Torrens System. The fifth case cited involves the registration
provisions of the BOT Law and the Local Government Code can avoid a under the Torrens System of a 12.8-hectare public land granted by the
direct collision with Section 3, Article XII of the 1987 Constitution. National Government to Mindanao Medical Center, a government unit under
the Department of Health. The National Government transferred the 12.8-
hectare public land to serve as the site for the hospital buildings and other
Registration of lands of the public domain facilities of Mindanao Medical Center, which performed a public service. The
Court affirmed the registration of the 12.8-hectare public land in the name of
Finally, PEA theorizes that the "act of conveying the ownership of the Mindanao Medical Center under Section 122 of Act No. 496. This fifth case is
reclaimed lands to public respondent PEA transformed such lands of the an example of a public land being registered under Act No. 496 without the
public domain to private lands." This theory is echoed by AMARI which land losing its character as a property of public dominion.
maintains that the "issuance of the special patent leading to the eventual
issuance of title takes the subject land away from the land of public domain In the instant case, the only patent and certificates of title issued are those in
and converts the property into patrimonial or private property." In short, PEA the name of PEA, a wholly government owned corporation performing public
and AMARI contend that with the issuance of Special Patent No. 3517 and the as well as proprietary functions. No patent or certificate of title has been
corresponding certificates of titles, the 157.84 hectares comprising the issued to any private party. No one is asking the Director of Lands to cancel
Freedom Islands have become private lands of PEA. In support of their PEA's patent or certificates of title. In fact, the thrust of the instant petition is
theory, PEA and AMARI cite the following rulings of the Court: that PEA's certificates of title should remain with PEA, and the land covered
by these certificates, being alienable lands of the public domain, should not be
1. Sumail v. Judge of CFI of Cotabato,97 where the Court held – sold to a private corporation.

"Once the patent was granted and the corresponding certificate of Registration of land under Act No. 496 or PD No. 1529 does not vest in the
title was issued, the land ceased to be part of the public domain and registrant private or public ownership of the land. Registration is not a mode
became private property over which the Director of Lands has of acquiring ownership but is merely evidence of ownership previously
neither control nor jurisdiction." conferred by any of the recognized modes of acquiring ownership.
Registration does not give the registrant a better right than what the registrant
had prior to the registration.102 The registration of lands of the public domain
2. Lee Hong Hok v. David,98 where the Court declared - under the Torrens system, by itself, cannot convert public lands into private
lands.103
"After the registration and issuance of the certificate and duplicate
certificate of title based on a public land patent, the land covered Jurisprudence holding that upon the grant of the patent or issuance of the
thereby automatically comes under the operation of Republic Act certificate of title the alienable land of the public domain automatically
496 subject to all the safeguards provided therein."3. Heirs of becomes private land cannot apply to government units and entities like PEA.
Gregorio Tengco v. Heirs of Jose Aliwalas,99 where the Court ruled The transfer of the Freedom Islands to PEA was made subject to the
- provisions of CA No. 141 as expressly stated in Special Patent No. 3517
issued by then President Aquino, to wit:
"While the Director of Lands has the power to review homestead
patents, he may do so only so long as the land remains part of the "NOW, THEREFORE, KNOW YE, that by authority of the
public domain and continues to be under his exclusive control; but Constitution of the Philippines and in conformity with the
once the patent is registered and a certificate of title is issued, the provisions of Presidential Decree No. 1084, supplemented by
land ceases to be part of the public domain and becomes private Commonwealth Act No. 141, as amended, there are hereby
property over which the Director of Lands has neither control nor granted and conveyed unto the Public Estates Authority the
jurisdiction." aforesaid tracts of land containing a total area of one million nine
hundred fifteen thousand eight hundred ninety four (1,915,894)
4. Manalo v. Intermediate Appellate Court,100 where the Court held square meters; the technical description of which are hereto
– attached and made an integral part hereof." (Emphasis supplied)

"When the lots in dispute were certified as disposable on May 19, Thus, the provisions of CA No. 141 apply to the Freedom Islands on matters
1971, and free patents were issued covering the same in favor of not covered by PD No. 1084. Section 60 of CA No. 141 prohibits, "except
the private respondents, the said lots ceased to be part of the public when authorized by Congress," the sale of alienable lands of the public
domain and, therefore, the Director of Lands lost jurisdiction over domain that are transferred to government units or entities. Section 60 of CA
the same." No. 141 constitutes, under Section 44 of PD No. 1529, a "statutory lien
affecting title" of the registered land even if not annotated on the certificate of
title.104 Alienable lands of the public domain held by government entities
5.Republic v. Court of Appeals,101 where the Court stated – under Section 60 of CA No. 141 remain public lands because they cannot be
alienated or encumbered unless Congress passes a law authorizing their
"Proclamation No. 350, dated October 9, 1956, of President disposition. Congress, however, cannot authorize the sale to private
Magsaysay legally effected a land grant to the Mindanao Medical corporations of reclaimed alienable lands of the public domain because of the
constitutional ban. Only individuals can benefit from such law.
The grant of legislative authority to sell public lands in accordance with the public domain. Only when qualified private parties acquire these lands
Section 60 of CA No. 141 does not automatically convert alienable lands of will the lands become private lands. In the hands of the government agency
the public domain into private or patrimonial lands. The alienable lands of the tasked and authorized to dispose of alienable of disposable lands of the
public domain must be transferred to qualified private parties, or to public domain, these lands are still public, not private lands.
government entities not tasked to dispose of public lands, before these lands
can become private or patrimonial lands. Otherwise, the constitutional ban
Furthermore, PEA's charter expressly states that PEA "shall hold lands of the
will become illusory if Congress can declare lands of the public domain as
public domain" as well as "any and all kinds of lands." PEA can hold both
private or patrimonial lands in the hands of a government agency tasked to
lands of the public domain and private lands. Thus, the mere fact that
dispose of public lands. This will allow private corporations to acquire
alienable lands of the public domain like the Freedom Islands are transferred
directly from government agencies limitless areas of lands which, prior to
to PEA and issued land patents or certificates of title in PEA's name does not
such law, are concededly public lands.
automatically make such lands private.

Under EO No. 525, PEA became the central implementing agency of the


To allow vast areas of reclaimed lands of the public domain to be transferred
National Government to reclaim foreshore and submerged areas of the public
to PEA as private lands will sanction a gross violation of the constitutional
domain. Thus, EO No. 525 declares that –
ban on private corporations from acquiring any kind of alienable land of the
public domain. PEA will simply turn around, as PEA has now done under
"EXECUTIVE ORDER NO. 525 the Amended JVA, and transfer several hundreds of hectares of these
reclaimed and still to be reclaimed lands to a single private corporation in only
one transaction. This scheme will effectively nullify the constitutional ban in
Designating the Public Estates Authority as the Agency Primarily
Section 3, Article XII of the 1987 Constitution which was intended to diffuse
Responsible for all Reclamation Projects
equitably the ownership of alienable lands of the public domain among
Filipinos, now numbering over 80 million strong.
Whereas, there are several reclamation projects which are ongoing
or being proposed to be undertaken in various parts of the country
This scheme, if allowed, can even be applied to alienable agricultural lands of
which need to be evaluated for consistency with national
the public domain since PEA can "acquire x x x any and all kinds of lands."
programs;
This will open the floodgates to corporations and even individuals acquiring
hundreds of hectares of alienable lands of the public domain under the guise
Whereas, there is a need to give further institutional support to the that in the hands of PEA these lands are private lands. This will result in
Government's declared policy to provide for a coordinated, corporations amassing huge landholdings never before seen in this country -
economical and efficient reclamation of lands; creating the very evil that the constitutional ban was designed to prevent. This
will completely reverse the clear direction of constitutional development in
this country. The 1935 Constitution allowed private corporations to acquire
Whereas, Presidential Decree No. 3-A requires that all reclamation not more than 1,024 hectares of public lands. 105 The 1973 Constitution
of areas shall be limited to the National Government or any person prohibited private corporations from acquiring any kind of public land, and
authorized by it under proper contract; the 1987 Constitution has unequivocally reiterated this prohibition.

Whereas, a central authority is needed to act on behalf of the The contention of PEA and AMARI that public lands, once registered under
National Government which shall ensure a coordinated and Act No. 496 or PD No. 1529, automatically become private lands is contrary
integrated approach in the reclamation of lands; to existing laws. Several laws authorize lands of the public domain to be
registered under the Torrens System or Act No. 496, now PD No. 1529,
Whereas, Presidential Decree No. 1084 creates the Public without losing their character as public lands. Section 122 of Act No. 496, and
Estates Authority as a government corporation to undertake Section 103 of PD No. 1529, respectively, provide as follows:
reclamation of lands and ensure their maximum utilization in
promoting public welfare and interests; and Act No. 496

Whereas, Presidential Decree No. 1416 provides the President with "Sec. 122. Whenever public lands in the Philippine Islands
continuing authority to reorganize the national government belonging to the x x x Government of the Philippine Islands are
including the transfer, abolition, or merger of functions and offices. alienated, granted, or conveyed to persons or the public or private
corporations, the same shall be brought forthwith under the
NOW, THEREFORE, I, FERDINAND E. MARCOS, President of operation of this Act and shall become registered lands."
the Philippines, by virtue of the powers vested in me by the
Constitution and pursuant to Presidential Decree No. 1416, do PD No. 1529
hereby order and direct the following:

"Sec. 103. Certificate of Title to Patents. Whenever public land is


Section 1. The Public Estates Authority (PEA) shall be primarily by the Government alienated, granted or conveyed to any person,
responsible for integrating, directing, and coordinating all the same shall be brought forthwith under the operation of this
reclamation projects for and on behalf of the National Decree." (Emphasis supplied)
Government. All reclamation projects shall be approved by the
President upon recommendation of the PEA, and shall be
undertaken by the PEA or through a proper contract executed by it Based on its legislative history, the phrase "conveyed to any person" in
with any person or entity; Provided, that, reclamation projects of Section 103 of PD No. 1529 includes conveyances of public lands to public
any national government agency or entity authorized under its corporations.
charter shall be undertaken in consultation with the PEA upon
approval of the President.
Alienable lands of the public domain "granted, donated, or transferred to a
province, municipality, or branch or subdivision of the Government," as
x x x ." provided in Section 60 of CA No. 141, may be registered under the Torrens
System pursuant to Section 103 of PD No. 1529. Such registration, however,
is expressly subject to the condition in Section 60 of CA No. 141 that the land
As the central implementing agency tasked to undertake reclamation projects "shall not be alienated, encumbered or otherwise disposed of in a manner
nationwide, with authority to sell reclaimed lands, PEA took the place of affecting its title, except when authorized by Congress." This provision refers
DENR as the government agency charged with leasing or selling reclaimed to government reclaimed, foreshore and marshy lands of the public domain
lands of the public domain. The reclaimed lands being leased or sold by PEA that have been titled but still cannot be alienated or encumbered unless
are not private lands, in the same manner that DENR, when it disposes of expressly authorized by Congress. The need for legislative authority prevents
other alienable lands, does not dispose of private lands but alienable lands of
the registered land of the public domain from becoming private land that can of the reclaimed lands, a transaction considered a sale or alienation under CA
be disposed of to qualified private parties. No. 141,108 the Government Auditing Code,109 and Section 3, Article XII of
the 1987 Constitution.
The Revised Administrative Code of 1987 also recognizes that lands of the
public domain may be registered under the Torrens System. Section 48, The Regalian doctrine is deeply implanted in our legal system. Foreshore and
Chapter 12, Book I of the Code states – submerged areas form part of the public domain and are inalienable. Lands
reclaimed from foreshore and submerged areas also form part of the public
domain and are also inalienable, unless converted pursuant to law into
"Sec. 48. Official Authorized to Convey Real Property. Whenever
alienable or disposable lands of the public domain. Historically, lands
real property of the Government is authorized by law to be
reclaimed by the government are sui generis, not available for sale to private
conveyed, the deed of conveyance shall be executed in behalf of
parties unlike other alienable public lands. Reclaimed lands retain their
the government by the following:
inherent potential as areas for public use or public service. Alienable lands of
the public domain, increasingly becoming scarce natural resources, are to be
(1) x x x distributed equitably among our ever-growing population. To insure such
equitable distribution, the 1973 and 1987 Constitutions have barred private
corporations from acquiring any kind of alienable land of the public domain.
(2) For property belonging to the Republic of the Philippines, but Those who attempt to dispose of inalienable natural resources of the State, or
titled in the name of any political subdivision or of any corporate seek to circumvent the constitutional ban on alienation of lands of the public
agency or instrumentality, by the executive head of the agency or domain to private corporations, do so at their own risk.
instrumentality." (Emphasis supplied)

We can now summarize our conclusions as follows:


Thus, private property purchased by the National Government for expansion
of a public wharf may be titled in the name of a government corporation
regulating port operations in the country. Private property purchased by the 1. The 157.84 hectares of reclaimed lands comprising the Freedom
National Government for expansion of an airport may also be titled in the Islands, now covered by certificates of title in the name of PEA,
name of the government agency tasked to administer the airport. Private are alienable lands of the public domain. PEA may lease these
property donated to a municipality for use as a town plaza or public school lands to private corporations but may not sell or transfer ownership
site may likewise be titled in the name of the municipality. 106 All these of these lands to private corporations. PEA may only sell these
properties become properties of the public domain, and if already registered lands to Philippine citizens, subject to the ownership limitations in
under Act No. 496 or PD No. 1529, remain registered land. There is no the 1987 Constitution and existing laws.
requirement or provision in any existing law for the de-registration of land
from the Torrens System.
2. The 592.15 hectares of submerged areas of Manila Bay remain
inalienable natural resources of the public domain until classified
Private lands taken by the Government for public use under its power of as alienable or disposable lands open to disposition and declared
eminent domain become unquestionably part of the public domain. no longer needed for public service. The government can make
Nevertheless, Section 85 of PD No. 1529 authorizes the Register of Deeds to such classification and declaration only after PEA has reclaimed
issue in the name of the National Government new certificates of title these submerged areas. Only then can these lands qualify as
covering such expropriated lands. Section 85 of PD No. 1529 states – agricultural lands of the public domain, which are the only natural
resources the government can alienate. In their present state, the
592.15 hectares of submerged areas are inalienable and outside
"Sec. 85. Land taken by eminent domain. Whenever any registered the commerce of man.
land, or interest therein, is expropriated or taken by eminent
domain, the National Government, province, city or municipality,
or any other agency or instrumentality exercising such right shall 3. Since the Amended JVA seeks to transfer to AMARI, a private
file for registration in the proper Registry a certified copy of the corporation, ownership of 77.34 hectares110 of the Freedom Islands,
judgment which shall state definitely by an adequate description, such transfer is void for being contrary to Section 3, Article XII of
the particular property or interest expropriated, the number of the the 1987 Constitution which prohibits private corporations from
certificate of title, and the nature of the public use. A memorandum acquiring any kind of alienable land of the public domain.
of the right or interest taken shall be made on each certificate of
title by the Register of Deeds, and where the fee simple is taken, a
4. Since the Amended JVA also seeks to transfer to AMARI
new certificate shall be issued in favor of the National
ownership of 290.156 hectares111 of still submerged areas of
Government, province, city, municipality, or any other agency or
Manila Bay, such transfer is void for being contrary to Section 2,
instrumentality exercising such right for the land so taken. The
Article XII of the 1987 Constitution which prohibits the alienation
legal expenses incident to the memorandum of registration or
of natural resources other than agricultural lands of the public
issuance of a new certificate of title shall be for the account of the
domain. PEA may reclaim these submerged areas. Thereafter, the
authority taking the land or interest therein." (Emphasis supplied)
government can classify the reclaimed lands as alienable or
disposable, and further declare them no longer needed for public
Consequently, lands registered under Act No. 496 or PD No. 1529 are not service. Still, the transfer of such reclaimed alienable lands of the
exclusively private or patrimonial lands. Lands of the public domain may also public domain to AMARI will be void in view of Section 3, Article
be registered pursuant to existing laws. XII of the 1987 Constitution which prohibits private corporations
from acquiring any kind of alienable land of the public domain.
AMARI makes a parting shot that the Amended JVA is not a sale to AMARI
of the Freedom Islands or of the lands to be reclaimed from submerged areas Clearly, the Amended JVA violates glaringly Sections 2 and 3, Article XII of
of Manila Bay. In the words of AMARI, the Amended JVA "is not a sale but a the 1987 Constitution. Under Article 1409112 of the Civil Code, contracts
joint venture with a stipulation for reimbursement of the original cost incurred whose "object or purpose is contrary to law," or whose "object is outside the
by PEA for the earlier reclamation and construction works performed by the commerce of men," are "inexistent and void from the beginning." The Court
CDCP under its 1973 contract with the Republic." Whether the Amended JVA must perform its duty to defend and uphold the Constitution, and therefore
is a sale or a joint venture, the fact remains that the Amended JVA requires declares the Amended JVA null and void ab initio.
PEA to "cause the issuance and delivery of the certificates of title conveying
AMARI's Land Share in the name of AMARI."107
Seventh issue: whether the Court is the proper forum to raise the issue of
whether the Amended JVA is grossly disadvantageous to the government.
This stipulation still contravenes Section 3, Article XII of the 1987
Constitution which provides that private corporations "shall not hold such
Considering that the Amended JVA is null and void ab initio, there is no
alienable lands of the public domain except by lease." The transfer of title and
necessity to rule on this last issue. Besides, the Court is not a trier of facts, and
ownership to AMARI clearly means that AMARI will "hold" the reclaimed
this last issue involves a determination of factual matters.
lands other than by lease. The transfer of title and ownership is a "disposition"
WHEREFORE, the petition is GRANTED. The Public Estates Authority and Will the lease and/or mortgage of a portion of a realty acquired through free
Amari Coastal Bay Development Corporation are PERMANENTLY patent constitute sufficient ground for the nullification of such land grant?
ENJOINED from implementing the Amended Joint Venture Agreement Should such property revert to the State once it is invaded by the sea and thus
which is hereby declared NULL and VOID ab initio. becomes foreshore land?

SO ORDERED. The Case

These are the two questions raised in the petition before us assailing the Court
of Appeals' 1 Decision in CA-G.R. CV No. 02667 promulgated on June 13,
1991 which answered the said questions in the negative. 2 Respondent Court's
dismissed 3 petitioner's appeal and affirmed in toto the decision of the
Regional Trial Court4 of Calauag, Quezon, dated December 28, 1983 in Civil
Case No. C-608. In turn, the Regional Trial Court's decision dismissed
petitioner's complaint for cancellation of the Torrens Certificate of Title of
Respondent Morato and for reversion of the parcel of land subject thereof of
the public domain.

The Facts

The petition of the solicitor general, representing the Republic of the


Philippines, recites the following facts: 5

Sometime in December, 1972, respondent Morato filed a Free Patent


Application No. III-3-8186-B on a parcel of land with an area of 1,265
square meters situated at Pinagtalleran, Calauag, Quezon. On January 16,
1974, the patent was approved and the Register of Deeds of Quezon at
Lucena City issued on February 4, 1974 Original Certificate of Title No.
P-17789. Both the free paten and the title specifically mandate that the
land shall not be alienated nor encumbered within five years from the
date of the issuance of the patent (Sections 118 and 124 of CA No. 141,
as amended).

Subsequently, the District Land Officer in Lucena City, acting upon


reports that respondent Morato had encumbered the land in violation of
the condition of the patent, conducted an investigation. Thereafter, it was
established that the subject land is a portion of the Calauag Bay, five (5)
to six (6) feet deep under water during high tide and two (2) feet deep at
low tide, and not suitable to vegetation. Moreover, on October 24, 1974, a
portion of the land was mortgaged by respondent Morato to respondents
Nenita Co and Antonio Quilatan for P10,000.00 (pp. 2, 25, Folder of
Exhibits). The spouses Quilatan constructed a house on the land. Another
portion of the land was leased to Perfecto Advincula on February 2, 1976
at P100.00 a month, where a warehouse was constructed.

On November 5, 1978, petitioner filed an amended complaint against


respondents Morato, spouses Nenita Co and Antonio Quilatan, and the
Register of Deeds of Quezon for the cancellation of title and reversion of
a parcel of land to the public domain, subject of a free patent in favor of
respondent Morato, on the grounds that the land is a foreshore land and
was mortgaged and leased within the five-year prohibitory period (p. 46,
Records).

After trial, the lower court, on December 28, 1983, rendered a decision
dismissing petitioner's complaint. In finding for private respondents, the
lower court ruled that there was no violation of the 5-year period ban
against alienating or encumbering the land, because the land was merely
leased and not alienated. It also found that the mortgage to Nenita Co and
Antonio Quilatan covered only the improvement and not the land itself.

On appeal, the Court of Appeals affirmed the decision of the trial court.
Thereafter, the Republic of the Philippines filed the present petition. 6

The Issues
G.R. No. 100709 November 14, 1997
Petitioner alleges that the following errors were committed by Respondent
REPUBLIC OF THE PHILIPPINES, represented by the DIRECTOR Court: 7
OF LANDS, petitioner,
vs. I
COURT OF APPEALS, JOSEFINA L. MORATO, SPOUSES NENITA
CO and ANTONIO QUILATAN AND THE REGISTER OF DEEDS OF
QUEZON PROVINCE, respondents. Respondent court erred in holding that the patent granted and
certificate of title issued to Respondent Morato cannot be cancelled
and annulled since the certificate of title becomes indefeasible after Respondent Morato counters by stating that although a "portion of the land
one year from the issuance of the title. was previously leased," it resulted "from the fact that Perfecto Advincula built
a warehouse in the subject land without [her] prior consent." The mortgage
executed over the improvement "cannot be considered a violation of the said
II
grant since it can never affect the ownership." 11 She states further:

Respondent Court erred in holding that the questioned land is part


. . . . the appeal of the petitioner was dismissed not because of the
of a disposable public land and not a foreshore land.
principle of indefeasibility of title but mainly due to failure of the
latter to support and prove the alleged violations of respondent
The Court's Ruling Morato. The records of this case will readily show that although
petitioner was able to establish that Morato committed some acts
during the prohibitory period of 5 years, a perusal thereof will also
The petition is meritorious. show that what petitioner was able to prove never constituted a
violation of the grant. 12
First Issue: Indefeasibility of a Free Patent Title
Respondent-Spouses Quilatan, on the other hand, state that the mortgage
In resolving the first issue against petitioner, Respondent Court held: 8 contract they entered into with Respondent Morato "can never be considered
as [an] 'alienation' inasmuch as the ownership over the property remains with
the owner." 13 Besides, it is the director of lands and not the Republic of the
. . . As ruled in Heirs of Gregorio Tengco vs. Heirs of Jose Alivalas, Philippines who is the real party in interest in this case, contrary to the
168 SCRA 198. ". . . The rule is well-settled that an original provision of the Public Land Act which states that actions for reversion should
certificate of title issued on the strength of a homestead patent be instituted by the solicitor general in the name of Republic of the
partakes of the nature of a certificate of title issued in a judicial Philippines. 14
proceeding, as long as the land disposed of is really part of the
disposable land of the public domain, and becomes indefeasible and
incontrovertible upon the expiration of one year from the date of We find for petitioner.
promulgation of the order of the Director of Lands for the issuance of
the patent. (Republic v. Heirs of Carle, 105 Phil. 1227 (1959); Quoted below are relevant sections of Commonwealth Act No. 141, otherwise
Ingaran v. Ramelo, 107 Phil. 498 (1960); Lopez v. Padilla, (G.R. No. known as the Public Land Act:
L-27559, May 18, 1972, 45 SCRA 44). A homestead patent, one
registered under the Land Registration Act, becomes as indefeasible
as a Torrens Title. (Pamintuan v. San Agustin, 43 Phil. 558 (1982); Sec. 118. Except in favor of the Government or any of its branches,
El Hogar Filipino v. Olviga, 60 Phil. 17 (1934); Duran v. Oliva, 113 units or institutions, or legally constituted banking
Phil. 144 (1961); Pajomayo v. Manipon, G.R. No. L-33676, June 30, corporations, lands acquired under free patent or homestead
1971, 39 SCRA 676). (p. 203). provisions shall not be subject to encumbrance or alienation from
the date of the approval of the application and for a term of five
years from and after the date of issuance of the patent or grant nor
Again, in Lopez vs. Court of Appeals, 169 SCRA 271, citing Iglesia shall they become liable to the satisfaction of any debt contracted
ni Cristo v. Hon. Judge, CFI of Nueva Ecija, Branch I, (123 SCRA prior to the expiration of said period; but the improvements or crops
516 (1983) and Pajomayo, et al. v. Manipon, et al. (39 SCRA 676 on the land may be mortgaged or pledged to qualified persons,
(1971) held that once a homestead patent granted in accordance with associations, or corporations.
the Public Land Act is registered pursuant to Section 122 of Act 496,
the certificate of title issued in virtue of said patent has the force and
effect of a Torrens Title issued under the Land Registration Act. No alienation, transfer, or conveyance of any homestead after five
years and before twenty-five years after issuance of title shall be
valid without the approval of the Secretary of Agriculture and
Indefeasibility of the title, however, may not bar the State, thru the Natural Resources, which approval shall not be denied except on
Solicitor General, from filing an action for reversion, as ruled in constitutional and legal grounds. (As amended by Com. Act No. 456,
Heirs of Gregorio Tengco v. Heirs of Jose Aliwalas, (supra), as approved June 8, 1939.)
follows:

xxx xxx xxx


But, as correctly pointed out by the respondent Court of Appeals, Dr.
Aliwalas' title to the property having become incontrovertible, such
may no longer be collaterally attacked. If indeed there had been any Sec. 121. Except with the consent of the grantee and the approval of
fraud or misrepresentation in obtaining the title, an action for the Secretary of Agriculture and Natural Resources, and solely for
reversion instituted by the Solicitor General would be the proper educational, religious, or charitable purposes or for a right of way, no
remedy (Sec. 101, C.A. No. 141; Director of Lands v. Jugado, G.R. corporation, association, or partnership may acquire or have any
No. L-14702, May 21, 1961, 2 SCRA 32; Lopez v. Padilla, supra). right, title, interest, or property right whatsoever to any land granted
(p. 204). under the free patent, homestead, or individual sale provisions of this
Act or to any permanent improvement on such land. (As amended by
Com. Act No. 615, approved May 5, 1941)
Petitioner contends that the grant of Free Patent (IV-3) 275 and the subsequent
issuance of Original Certificate of Title No. P-17789 to Respondent Josefina
L. Morato were subject to the conditions provided for in Commonwealth Act Sec. 122. No land originally acquired in any manner under the
(CA) No. 141. It alleges that on October 24, 1974, or nine (9) months and provisions of this Act, nor any permanent improvement on such land,
eight (8) days after the grant of the patent, mortgaged a portion of the land" to shall be encumbered, alienation or transferred, except to persons,
Respondent Nenita Co, who thereafter constructed a house thereon. Likewise, corporations, association, or partnerships who may acquire lands of
on February 2, 1976 and "within the five-year prohibitory period," the public domain under this Act or to corporations organized in the
Respondent Morato "leased a portion of the land to Perfecto Advincula at a Philippines authorized therefore by their charters.
monthly rent of P100.00 who, shortly thereafter, constructed a house of
concrete materials on the subject land."9 Further, petitioner argues that the
Except in cases of hereditary successions, no land or any portion
defense of indefeasibility of title is "inaccurate." The original certificate of
thereof originally acquired under the free patent, homestead, or
title issued to Respondent Morato "contains the seeds of its own cancellation":
individual sale provisions of this Act, or any permanent improvement
such certificate specifically states on its face that "it is subject to the
on such land, shall be transferred or assigned to any individual, nor
provisions of Sections 118, 119, 121, 122, 124 of CA No. 141, as
shall such land or any permanent improvement thereon be leased to
amended." 10
such individual, when the area of said land, added to that of this own,
shall exceed one hundred and forty-four hectares. Any transfer,
assignment, or lease made in violation hereto shall be null and void. condition for the grant of a free patent to an applicant, the law requires that the
(As amended by Com Act No. 615, Id.). land should not be encumbered, sold or alienated within five years from the
issuance of
the patent. The sale or the alienation of part of the homestead violates that
xxx xxx xxx
condition. 21

Sec. 124. Any acquisition, conveyance, alienation, transfer, or other


The prohibition against the encumbrance — lease and mortgage included —
contract made or executed in violation of any of the provisions of
of a homestead which, by analogy applies to a free patent, is mandated by the
sections one hundred and eighteen, one hundred and twenty, one
rationale for the grant, viz.: 22
hundred and twenty-one, one hundred and twenty-two, and one
hundred and twenty-three of this Act shall be unlawful and null and
void from its execution and shall produce the effect of annulling and It is well-known that the homestead laws were designed to distribute
cancelling the grant, title, patent, or permit originally issued, disposable agricultural lots of the State to land-destitute citizens for
recognized or confirmed, actually or presumatively, and cause the their home and cultivation. Pursuant to such benevolent intention the
reversion of the property and its improvements to the State. State prohibits the sale or incumbrance of the homestead (Section
(Emphasis supplied) 116) within five years after the grant of the patent. After that five-
year period the law impliedly permits alienation of the homestead;
but in line with the primordial purpose to favor the homesteader and
The foregoing legal provisions clearly proscribe the encumbrance of a parcel
his family the statute provides that such alienation or conveyance
of land acquired under a free patent or homestead within five years from the
(Section 117) shall be subject to the right of repurchase by the
grant of such patent. Furthermore, such encumbrance results in the
homesteader, his widow or heirs within five years. This section 117
cancellation of the grant and the reversion of the land to the public domain.
is undoubtedly a complement of section 116. It aims to preserve and
Encumbrance has been defined as "[a]nything that impairs the use or transfer
keep in the family of the homesteader that portion of public land
of property; anything which constitutes a burden on the title; a burden or
which the State had gratuitously given to him. It would, therefore, be
charge upon property; a claim or lien upon property." It may be a "legal claim
in keeping with this fundamental idea to hold, as we hold, that the
on an estate for the discharge of which the estate is liable; and embarrassment
right to repurchase exists not only when the original homesteader
of the estate or property so that it cannot be disposed of without being subject
makes the conveyance, but also when it is made by his widow or
to it; an estate, interest, or right in lands, diminishing their value to the general
heirs. This construction is clearly deducible from the terms of the
owner; a liability resting upon an estate." 15 Do the contracts of lease and
statute.
mortgage executed within five (5) years from the issuance of the patent
constitute an "encumbrance" and violate the terms and conditions of such
patent? Respondent Court answered in the negative: 16 By express provision of Section 118 of Commonwealth Act 141 and in
conformity with the policy of the law, any transfer or alienation of a free
patent or homestead within five years from the issuance of the patent is
From the evidence adduced by both parties, it has been proved that
proscribed. Such transfer nullifies said alienation and constitutes a cause for
the area of the portion of the land, subject matter of the lease contract
the reversion of the property to the State.
(Exh. "B") executed by and between Perfecto Advincula and Josefina
L. Morato is only 10 x 12 square meters, where the total area of the
land granted to Morato is 1,265 square meters. It is clear from this The prohibition against any alienation or encumbrance of the land grant is a
that the portion of the land leased by Advincula does not significantly proviso attached to the approval of every application. 23 Prior to the fulfillment
affect Morato's ownership and possession. Above all, the of the requirements of law, Respondent Morato had only an inchoate right to
circumstances under which the lease was executed do not reflect a the property; such property remained part of the public domain and, therefore,
voluntary and blatant intent to violate the conditions provided for in not susceptible to alienation or encumbrance. Conversely, when a
the patent issued in her favor. On the contrary, Morato was "homesteader has complied with all the terms and conditions which entitled
compelled to enter into that contract of lease him to a patent for [a] particular tract of public land, he acquires a vested
out of sympathy and the goodness of her heart to accommodate a interest therein and has to be regarded an equitable owner
fellow man. . . . thereof." 24 However, for Respondent Morato's title of ownership over the
patented land to be perfected, she should have complied with the requirements
of the law, one of which was to keep the property for herself and her family
It is indisputable, however, that Respondent Morato cannot fully use or enjoy
within the prescribed period of five (5) years. Prior to the fulfillment of all
the land during the duration of the lease contract. This restriction on the
requirements of the law, Respondent Morato's title over the property was
enjoyment of her property sufficiently meets the definition of an encumbrance
incomplete. Accordingly, if the requirements are not complied with, the State
under Section 118 of the Public Land Act, because such contract "impairs the
as the grantor could petition for the annulment of the patent and the
use of the property" by the grantee. In a contract of lease which is consensual,
cancellation of the title.
bilateral, onerous and commutative, the owner temporarily grants the use of
his or her property to another who undertakes to pay rent therefor. 17 During
the term of the lease, the grantee of the patent cannot enjoy the beneficial use Respondent Morato cannot use the doctrine of the indefeasibility of her
of the land leased. As already observed, the Public Land Act does not permit a Torrens title to bar the state from questioning its transfer or encumbrance. The
grantee of a free patent from encumbering any portion of such land. Such certificate of title issued to her clearly stipulated that its award was "subject to
encumbrance is a ground for the nullification of the award. the conditions provided for in Sections 118, 119, 121, 122 and 124 of
Commonwealth Act (CA) No. 141." Because she violated Section 118, the
reversion of the property to the public domain necessarily follows, pursuant to
Morato's resort to equity, i.e. that the lease was executed allegedly out of the
Section 124.
goodness of her heart without any intention of violating the law, cannot help
her. Equity, which has been aptly described as "justice outside legality," is
applied only in the absence of, and never against, statutory law or judicial Second Issue:  Foreshore Land Revert to the Public Domain
rules of procedure. Positive rules prevail over all abstract arguments based on
equity contra legem. 18
There is yet another reason for granting this petition.

Respondents failed to justify their position that the mortgage should not be
Although Respondent Court found that the subject land was foreshore land, it
considered an encumbrance. Indeed, we do not find any support for such
nevertheless sustained the award thereof to Respondent Morato: 25
contention. The questioned mortgage falls squarely within the term
"encumbrance" proscribed by Section 118 of the Public Land Act. 19 Verily, a
mortgage constitutes a legal limitation on the estate, and the foreclosure of First of all, the issue here is whether the land in question, is really
such mortgage would necessarily result in the auction of the property. 20 part of the foreshore lands. The Supreme Court defines foreshore
land in the case of Republic vs. Alagad, 169 SCRA 455, 464, as
follows:
Even if only part of the property has been sold or alienated within the
prohibited period of five years from the issuance of the patent, such alienation
is a sufficient cause for the reversion of the whole estate to the State. As a
Otherwise, where the rise in water level is due to, the While at the time of the grant of free patent to
"extraordinary" action of nature, rainful, for instance, the respondent Morato, the land was not reached by the
portions inundated thereby are not considered part of the bed or water, however, due to gradual sinking of the land
basin of the body of water in question. It cannot therefore be said caused by natural calamities, the sea advances had
to be foreshore land but land outside of the public dominion, and permanently invaded a portion of subject land. As
land capable of registration as private property. disclosed at the trial, through the testimony of the
court-appointed commissioner, Engr. Abraham B. Pili,
the land was under water during high tide in the month
A foreshore land, on the other hand has been defined as follows:
of August 1978. The water margin covers half of the
property, but during low tide, the water is about a
. . . that part of (the land) which is between high kilometer (TSN, July 19, 1979, p. 12). Also, in 1974,
and low water and left dry by the flux and reflux of the after the grant of the patent, the land was covered with
tides . . . . (Republic vs. C.A., Nos. L-43105, L-43190, vegetation, but it disappeared in 1978 when the land
August 31, 1984, 131 SCRA 532; Government vs. Colegio was reached by the tides (Exh. "E-1", "E-14"). In fact,
de San Jose, 53 Phil 423) in its decision dated December 28, 1983, the lower
court observed that the erosion of the land was caused
by natural calamities that struck the place in 1977 (Cf.
The strip of land that lies between the high and low water Decision, pp. 17-18). 26
marks and that is alternatively wet and dry according to the
flow of the tide. (Rep. vs. CA, supra, 539).
Respondent-Spouses Quilatan argue, however, that it is "unfair and unjust if
Josefina Morato will be deprived of the whole property just because a portion
The factual findings of the lower court regarding the nature of the parcel of thereof was immersed in water for reasons not her own doing." 27
land in question reads:

As a general rule, findings of facts of the Court of Appeals are binding and
Evidence disclose that the marginal area of the land radically conclusive upon this Court, unless such factual findings are palpably
changed sometime in 1937 up to 1955 due to a strong earthquake unsupported by the evidence on record or unless the judgment itself is based
followed by frequent storms eventually eroding the land. From 1955 on a misapprehension of facts. 28 The application for a free patent was made in
to 1968, however, gradual reclamation was undertaken by the lumber 1972. From the undisputed factual findings of the Court of Appeals, however,
company owned by the Moratos. Having thus restored the land thru the land has since become foreshore. Accordingly, it can no longer be subject
mostly human hands employed by the lumber company, the area of a free patent under the Public Land Act. Government of the Philippine
continued to be utilized by the owner of the sawmill up to the time of Islands vs. Cabañgis  29 explained the rationale for this proscription:
his death in 1965. On or about March 17, 1973, there again was a
strong earthquake unfortunately causing destruction to hundreds of
residential houses fronting the Calauag Bay including the Santiago Article 339, subsection 1, of the Civil Code, reads:
Building, a cinema house constructed of concrete materials. The
catastrophe totally caused the sinking of a concrete bridge at
Art. 339. Property of public ownership is —
Sumulong river also in the municipality of Calauag, Quezon.

1. That devoted to public use, such as roads, canals, rivers, torrents,


On November 13, 1977 a typhoon code named "Unding" wrought
ports and bridges constructed by the State, riverbanks, shores,
havoc as it lashed the main land of Calauag, Quezon causing again
roadsteads, and that of a similar character.
great erosion this time than that which the area suffered in 1937. The
Court noted with the significance of the newspaper clipping entitled
"Baryo ng Mangingisda Kinain ng Dagat" (Exh. "11"). xxx xxx xxx

x x x           x x x          x x x Article 1, case 3, of the law of Waters of August 3, 1866, provides as


follows:
Evidently this was the condition of the land when on or about
December 5, 1972 defendant Josefina L. Morato filed with the Art. 1. The following are part of the national domain open to public
Bureau of Lands her free patent application. The defendant Josefina use.
Morato having taken possession of the land after the demise of Don
Tomas Morato, she introduced improvement and continued
xxx xxx xxx
developing the area, planted it to coconut tree. Having applied for a
free patent, defendant had the land area surveyed and an approved
plan (Exh. "9") based on the cadastral survey as early as 1927 (Exh. 3. The Shores. By the shore is understood that space covered and
"10") was secured. The area was declared for taxation purposes in the uncovered by the movement of the tide. Its interior or terrestrial limit
name of defendant Josefina Morato denominated as Tax Declaration is the line reached by the highest equinoctal tides. Where the tides are
No. 4115 (Exh. "8") and the corresponding realty taxes religiously not appreciable, the shore begins on the land side at the line reached
paid as shown by Exh. "8-A"). (pp. 12-14, DECISION). by the sea during ordinary storms or tempests.

Being supported by substantial evidence and for failure of the In the case of Aragon vs. Insular Government (19 Phil. 223), with
appellant to show cause which would warrant disturbance, the reference to article 339 of the Civil Code just quoted, this Court said:
aforecited findings of the lower court, must be respected.
We should not be understood, by this decision, to hold that in a case
Petitioner correctly contends, however, that Private Respondent Morato of gradual encroachment or erosion by the ebb and flow of the tide,
cannot own foreshore land: private property may not become "property of public ownership." as
defined in article 339 of the code, where it appear that the owner has
to all intents and purposes abandoned it and permitted it to be totally
Through the encroachment or erosion by the ebb and flow of the tide,
destroyed, so as to become a part of the "playa" (shore of the sea),
a portion of the subject land was invaded by the waves and sea
"rada" (roadstead), or the like. . . .
advances. During high tide, at least half of the land (632.5 square
meters) is 6 feet deep under water and three (3) feet deep during low
tide. The Calauag Bay shore has extended up to a portion of the In the Enciclopedia Juridica Española, volume XII, page 558, we
questioned land. read the following:
With relative frequency the opposite phenomenon occurs; that is, the city officials have designated as flea markets, and the private respondents
sea advances and private properties are permanently invaded by the (stallholders) to vacate the streets.
waves, and in this case they become part of the shore or breach. The
then pass to the public domain, but the owner thus dispossessed does
On January 5, 1979, MMC Ordinance No. 79-02 was enacted by the
not retain any right to the natural products resulting from their new
Metropolitan Manila Commission, designating certain city and municipal
nature; it is a de facto case of eminent domain, and not subject to
streets, roads and open spaces as sites for flea markets. Pursuant, thereto, the
indemnity.
Caloocan City mayor opened up seven (7) flea markets in that city. One of
those streets was the "Heroes del '96" where the petitioner lives. Upon
In comparison, Article 420 of the Civil Code provides: application of vendors Rodolfo Teope, Mila Pastrana, Carmen Barbosa, Merle
Castillo, Bienvenido Menes, Nancy Bugarin, Jose Manuel, Crisaldo
Paguirigan, Alejandro Castron, Ruben Araneta, Juanita and Rafael Malibaran,
Art. 420. The following things are property of public dominion:
and others, the respondents city mayor and city engineer, issued them licenses
to conduct vending activities on said street.
(1) Those intended for public use, such as roads, canals, rivers,
torrents, ports and bridges constructed by the State, banks, shores,
In 1987, Antonio Martinez, as OIC city mayor of Caloocan City, caused the
roadsteads, and others of similar character;
demolition of the market stalls on Heroes del '96, V. Gozon and Gonzales
streets. To stop Mayor Martinez' efforts to clear the city streets, Rodolfo
(2) Those which belong to the State, without being for public use, Teope, Mila Pastrana and other stallowners filed an action for prohibition
and are intended for some public service or for the development of against the City of Caloocan, the OIC City Mayor and the City Engineer
the national wealth. and/or their deputies (Civil Case No. C-12921) in the Regional Trial Court of
Caloocan City, Branch 122, praying the court to issue a writ of preliminary
injunction ordering these city officials to discontinue the demolition of their
When the sea moved towards the estate and the tide invaded it, the invaded stalls during the pendency of the action.
property became foreshore land and passed to the realm of the public domain.
In fact, the Court in Government vs. Cabangis 30 annulled the registration of
land subject of cadastral proceedings when the parcel subsequently became The court issued the writ prayed for. However, on December 20, 1987, it
foreshore land. 31 In another case, the Court voided the registration decree of a dismissed the petition and lifted the writ of preliminary injunction which it
trial court and held that said court had no jurisdiction to award foreshore land had earlier issued. The trial court observed that:
to any private person or entity. 32 The subject land in this case, being foreshore
land, should therefore be returned to the public domain.
A perusal of Ordinance 2, series of 1979 of the Metropolitan Manila
Commission will show on the title itself that it is an ordinance ––
WHEREFORE, the petition is GRANTED. This Court hereby REVERSES
and SETS ASIDE the assailed Decision of Respondent Court and ORDERS
Authorizing and regulating the use of certain city and/or municipal
the CANCELLATION of Free Patent No. (IV-3) 275 issued to Respondent
streets, roads and open spaces within Metropolitan Manila as sites for
Morato and the subsequent Original Certificate of Title No. P-17789. The
flea market and/or vending areas, under certain terms and
subject land therefore REVERTS to the State. No costs.
conditions, subject to the approval of the Metropolitan Manila
Commission,  and for other purposes
SO ORDERED.
which is further amplified in Section 2 of the said ordinance, quoted
hereunder:

Sec. 2. The streets, roads and open spaces to be used as sites for flea
markets (tiangge) or vending areas; the design, measurement or
specification of the structures, equipment and apparatuses to be used or
put up; the allowable distances; the days and time allowed for the conduct
of the businesses and/or activities herein authorized; the rates or fees or
charges to be imposed, levied and collected; the kinds of merchandise,
goods and commodities sold and services rendered; and other matters and
activities related to the establishment, maintenance and management and
operation of flea markets and vending areas, shall be determined and
prescribed by the mayors of the cities and municipalities in the
Metropolitan Manila where the same are located, subject to the approval
of the Metropolitan Manila Commission and consistent with the
guidelines hereby prescribed.

Further, it is so  provided in the guidelines under the said Ordinance No. 2
of the MMC that —

G.R. No. 93654 May 6, 1992 Sec. 6. In the establishment, operation, maintenance and management of
flea markets and vending areas, the following guidelines, among others,
FRANCISCO U. DACANAY, petitioner, shall be observed:
vs.
MAYOR MACARIO ASISTIO, JR., CITY ENGR. LUCIANO SARNE, xxx xxx xxx
JR. of Kalookan City, Metro Manila, MILA PASTRANA AND/OR
RODOLFO TEOFE, STALLHOLDERS AND REPRESENTING CO-
STALLHOLDERS, respondents. (m) That the permittee shall remove the equipment, facilities and other
appurtenances used by him in the conduct of his business after the close
or termination of business hours. (Emphasis ours; pp. 15-16, Rollo.)
May public streets or thoroughfares be leased or licensed to market
stallholders by virtue of a city ordinance or resolution of the Metro Manila
Commission? This issue is posed by the petitioner, an aggrieved Caloocan The trial court found that Heroes del '96, Gozon and Gonzales streets are of
City resident who filed a special civil action of mandamus against the public dominion, hence, outside the commerce of man:
incumbent city mayor and city engineer, to compel these city officials to
remove the market stalls from certain city streets which the aforementioned
The Heroes del '96 street, V. Gozon street and Gonzales street, being of (4) . . .
public dominion must, therefore, be outside of the commerce of man.
Considering the nature of the subject premises, the following
(c) Prevent the encroachment of private buildings and fences
jurisprudence co/principles are applicable on the matter:
on the streets and public places;

1) They cannot be alienated or leased or otherwise be the subject


xxx xxx xxx
matter of contracts. (Municipality of Cavite vs. Rojas, 30 Phil.
602);
(j) Inspect and supervise the construction, repair, removal
and safety of private buildings;
2) They cannot be acquired by prescription against the state
(Insular Government vs. Aldecoa, 19 Phil. 505). Even
municipalities can not acquire them for use as communal lands xxx xxx xxx
against the state (City of Manila vs. Insular Government, 10 Phil.
327);
(k) With the previous approval of the City Mayor in each
case, order the removal of materials employed in the
3) They are not subject to attachment and execution (Tan Toco vs. construction or repair of any building or structures made in
Municipal Council of Iloilo, 49 Phil. 52); violation of law or ordinance, and cause buildings and
structures dangerous to the public to made secure or torn
down;
4) They cannot be burdened by any voluntary easement (2-II Colin
& Capitant 520) (Tolentino, Civil Code of the Phils., Vol. II, 1983
Ed. pp. 29-30). x x x           x x x          x x x

In the aforecited case of Municipality of Cavite vs. Rojas, it was Further, the Charter of the City of Caloocan, Republic Act No. 5502,
held that properties for public use may not be leased to private Art. VII, Sec. 27, par. g, 1 and m, grants the City Engineer similar
individuals. Such a lease is null and void for the reason that a powers. (Emphasis supplied; pp. 17-20, Rollo.)
municipal council cannot withdraw part of the plaza from public
use. If possession has already been given, the lessee must restore
However, shortly after the decision came out, the city administration in
possession by vacating it and the municipality must thereupon
Caloocan City changed hands. City Mayor Macario Asistio, Jr., as successor
restore to him any sums it may have collected as rent.
of Mayor Martinez, did not pursue the latter's policy of clearing and cleaning
up the city streets.
In the case of City of Manila vs. Gerardo Garcia, 19 SCRA 413,
the Supreme Court held:
Invoking the trial court's decision in Civil Case No. C-12921, Francisco U.
Dacanay, a concerned citizen, taxpayer and registered voter of Barangay 74,
The property being a public one, the Manila Mayors did Zone 7, District II of Caloocan City, who resides on Heroes del '96 Street, one
not have the authority to give permits, written or oral, of the affected streets, wrote a letter dated March 7, 1988 to Mayor Asistio,
to the squatters, and that the permits granted are Jr., calling his attention to the illegally-constructed stalls on Heroes del '96
therefore considered null and void. Street and asked for their demolition.

This doctrine was reiterated in the case of Baguio Dacanay followed up that letter with another one dated April 7, 1988
Citizens Action Inc. vs. The City Council, 121 SCRA addressed to the mayor and the city engineer, Luciano Sarne, Jr. (who
368, where it was held that: replaced Engineer Arturo Samonte), inviting their attention to the Regional
Trial Court's decision in Civil Case No. 12921. There was still no response.
An ordinance legalizing the occupancy by squatters of
public land is null and void. Dacanay sought President Corazon C. Aquino's intervention by writing her a
letter on the matter. His letter was referred to the city mayor for appropriate
action. The acting Caloocan City secretary, Asuncion Manalo, in a letter dated
The authority of respondent Municipality of Makati to demolish the
August 1, 1988, informed the Presidential Staff Director that the city officials
shanties of the petitioner's members is mandated by
were still studying the issue of whether or not to proceed with the demolition
P.D. 772, and Sec. 1 of Letter of Instruction No. 19 orders certain public
of the market stalls.
officials, one of whom is the Municipal Mayor to remove all illegal
constructions including buildings on and along esteros and river banks,
those along railroad tracks and those built without permits on public or Dacanay filed a complaint against Mayor Asistio and Engineer Sarne (OMB-
private property (Zansibarian Residents Association vs. Mun. of Makati, 0-89-0146) in the Office of the OMBUDSMAN. In their letter-comment dated
135 SCRA 235). The City Engineer is also among those required to April 3, 1989, said city officials explained that in view of the huge number of
comply with said Letter of Instruction. stallholders involved, not to mention their dependents, it would be harsh and
inhuman to eject them from the area in question, for their relocation would not
be an easy task.
The occupation and use of private individuals of sidewalks and other
public places devoted for public use constitute both public and private
nuisances and nuisance per se, and this applies to even case involving the In reply, Dacanay maintained that respondents have been derelict in the
use or lease of public places under permits and licenses issued by performance of their duties and through manifest partiality constituting a
competent authority, upon the theory that such holders could not take violation of Section 3(e) of R.A. 3019, have caused undue injury to the
advantage of their unlawful permits and license and claim that the land in Government and given unwarranted benefits to the stallholders.
question is a part of a public street or a public place devoted to public use,
hence, beyond the commerce of man. (Padilla, Civil Code Annotated,
After conducting a preliminary investigation, the OMBUDSMAN rendered a
Vol. II, p. 59, 6th Ed., citing Umali vs. Aquino, IC. A. Rep. 339.)
final evaluation and report on August 28, 1989, finding that the respondents'
inaction is purely motivated by their perceived moral and social responsibility
From the aforequoted jurisprudence/principles, the Court opines that toward their constituents, but "the fact remains that there is an omission of an
defendants have the right to demolish the subject stalls of the act which ought to be performed, in clear violation of Sections 3(e) and (f) of
plaintiffs, more so when Section 185, par. 4 of Batas Pambansa Blg. Republic Act 3019."  (pp. 83-84, Rollo.) The OMBUDSMAN recommended
337, otherwise known as the Local Government Code provides that the filing of the corresponding information in court.
the City Engineer shall:
As the stallholders continued to occupy Heroes del '96 Street, through the
tolerance of the public respondents, and in clear violation of the decision it
Civil Case No. C-12921, Dacanay filed the present petition for mandamus on
June 19, 1990, praying that the public respondents be ordered to enforce the
final decision in Civil Case No. C-12921 which upheld the city mayor's
authority to order the demolition of market stalls on V. Gozon, Gonzales and
Heroes del '96 Streets and to enforce P.D. No. 772 and other pertinent laws.

On August 16, 1990, the public respondents, through the City Legal Officer,
filed their Comment' on the petition. The Office of the Solicitor General asked
G.R. No. 92013 July 25, 1990
to be excused from filing a separate Comment in behalf of the public
respondents. The City Legal Officer alleged that the vending area was
transferred to Heroes del '96 Street to decongest Malonzo Street, which is SALVADOR H. LAUREL, petitioner,
comparatively a busier thoroughfare; that the transfer was made by virtue of vs.
Barangay Resolution No. 30 s'78 dated January 15, 1978; that while the RAMON GARCIA, as head of the Asset Privatization Trust, RAUL
resolution was awaiting approval by the Metropolitan Manila Commission, MANGLAPUS, as Secretary of Foreign Affairs, and CATALINO
the latter passed Ordinance No. 79-2, authorizing the use of certain streets and MACARAIG, as Executive Secretary, respondents.
open spaces as sites for flea markets and/or vending areas; that pursuant
thereto, Acting MMC Mayor Virgilio P. Robles issued Executive Order No.
135 dated January 10, 1979, ordering the establishment and operation of flea G.R. No. 92047 July 25, 1990
markets in specified areas and created the Caloocan City Flea Market
Authority as a regulatory body; and that among the sites chosen and approved DIONISIO S. OJEDA, petitioner,
by the Metro Manila Commission, Heroes del '96 Street has considered "most vs.
viable and progressive, lessening unemployment in the city and servicing the EXECUTIVE SECRETARY MACARAIG, JR., ASSETS
residents with affordable basic necessities." PRIVATIZATION TRUST CHAIRMAN RAMON T. GARCIA,
AMBASSADOR RAMON DEL ROSARIO, et al., as members of the
The petition for mandamus is meritorious. PRINCIPAL AND BIDDING COMMITTEES ON THE
UTILIZATION/DISPOSITION PETITION OF PHILIPPINE
GOVERNMENT PROPERTIES IN JAPAN, respondents.
There is no doubt that the disputed areas from which the private respondents'
market stalls are sought to be evicted are public streets, as found by the trial
court in Civil Case No. C-12921. A public street is property for public use These are two petitions for prohibition seeking to enjoin respondents,
hence outside the commerce of man (Arts. 420, 424, Civil Code). Being their representatives and agents from proceeding with the bidding for the
outside the commerce of man, it may not be the subject of lease or other sale of the 3,179 square meters of land at 306 Roppongi, 5-Chome
contract (Villanueva et al. vs. Castañeda and Macalino, 15 SCRA 142, citing Minato-ku Tokyo, Japan scheduled on February 21, 1990. We granted
the Municipality of Cavite vs. Rojas, 30 SCRA 602; Espiritu vs. Municipal the prayer for a temporary restraining order effective February 20, 1990.
Council of Pozorrubio, 102 Phil. 869; and Muyot vs. De la Fuente, 48 O.G. One of the petitioners (in G.R. No. 92047) likewise prayes for a writ of
4860). mandamus to compel the respondents to fully disclose to the public the
basis of their decision to push through with the sale of the Roppongi
property inspire of strong public opposition and to explain the
As the stallholders pay fees to the City Government for the right to occupy proceedings which effectively prevent the participation of Filipino
portions of the public street, the City Government, contrary to law, has been citizens and entities in the bidding process.
leasing portions of the streets to them. Such leases or licenses are null and
void for being contrary to law. The right of the public to use the city streets
may not be bargained away through contract. The interests of a few should not The oral arguments in G.R. No. 92013, Laurel v. Garcia, et al. were heard
prevail over the good of the greater number in the community whose health, by the Court on March 13, 1990. After G.R. No. 92047, Ojeda v. Secretary
peace, safety, good order and general welfare, the respondent city officials are Macaraig, et al. was filed, the respondents were required to file a
under legal obligation to protect. comment by the Court's resolution dated February 22, 1990. The two
petitions were consolidated on March 27, 1990 when the memoranda of
the parties in the Laurel case were deliberated upon.
The Executive Order issued by Acting Mayor Robles authorizing the use of
Heroes del '96 Street as a vending area for stallholders who were granted
licenses by the city government contravenes the general law that reserves city The Court could not act on these cases immediately because the
streets and roads for public use. Mayor Robles' Executive Order may not respondents filed a motion for an extension of thirty (30) days to file
infringe upon the vested right of the public to use city streets for the purpose comment in G.R. No. 92047, followed by a second motion for an extension
they were intended to serve: i.e., as arteries of travel for vehicles and of another thirty (30) days which we granted on May 8, 1990, a third
pedestrians. As early as 1989, the public respondents bad started to look for motion for extension of time granted on May 24, 1990 and a fourth
feasible alternative sites for flea markets. They have had more than ample motion for extension of time which we granted on June 5, 1990 but calling
time to relocate the street vendors. the attention of the respondents to the length of time the petitions have
been pending. After the comment was filed, the petitioner in G.R. No.
92047 asked for thirty (30) days to file a reply. We noted his motion and
WHEREFORE, it having been established that the petitioner and the general resolved to decide the two (2) cases.
public have a legal right to the relief demanded and that the public
respondents have the corresponding duty, arising from public office, to clear
the city streets and restore them to their specific public purpose (Enriquez vs. I
Bidin, 47 SCRA 183; City of Manila vs. Garcia et al., 19 SCRA, 413 citing
Unson vs. Lacson, 100 Phil. 695), the respondents City Mayor and City The subject property in this case is one of the four (4) properties in Japan
Engineer of Caloocan City or their successors in office are hereby ordered to acquired by the Philippine government under the Reparations
immediately enforce and implement the decision in Civil Case No. C-1292 Agreement entered into with Japan on May 9, 1956, the other lots being:
declaring that Heroes del '96, V. Gozon, and Gonzales Streets are public
streets for public use, and they are ordered to remove or demolish, or cause to
be removed or demolished, the market stalls occupying said city streets with (1) The Nampeidai Property at 11-24 Nampeidai-machi, Shibuya-ku,
utmost dispatch within thirty (30)days from notice of this decision. This Tokyo which has an area of approximately 2,489.96 square meters, and is
decision is immediately executory. at present the site of the Philippine Embassy Chancery;

SO ORDERED. (2) The Kobe Commercial Property at 63 Naniwa-cho, Kobe, with an


area of around 764.72 square meters and categorized as a commercial lot
now being used as a warehouse and parking lot for the consulate staff; government in favor of selling the property to non-Filipino citizens and
and entities. These petitions have been consolidated and are resolved at the
same time for the objective is the same - to stop the sale of the Roppongi
property.
(3) The Kobe Residential Property at 1-980-2 Obanoyama-cho,
Shinohara, Nada-ku, Kobe, a residential lot which is now vacant.
The petitioner in G.R. No. 92013 raises the following issues:
The properties and the capital goods and services procured from the
Japanese government for national development projects are part of the (1) Can the Roppongi property and others of its kind be alienated by the
indemnification to the Filipino people for their losses in life and property Philippine Government?; and
and their suffering during World War II.
(2) Does the Chief Executive, her officers and agents, have the authority
The Reparations Agreement provides that reparations valued at $550 and jurisdiction, to sell the Roppongi property?
million would be payable in twenty (20) years in accordance with annual
schedules of procurements to be fixed by the Philippine and Japanese
Petitioner Dionisio Ojeda in G.R. No. 92047, apart from questioning the
governments (Article 2, Reparations Agreement). Rep. Act No. 1789, the
authority of the government to alienate the Roppongi property assails the
Reparations Law, prescribes the national policy on procurement and
constitutionality of Executive Order No. 296 in making the property
utilization of reparations and development loans. The procurements are
available for sale to non-Filipino citizens and entities. He also questions
divided into those for use by the government sector and those for private
the bidding procedures of the Committee on the Utilization or Disposition
parties in projects as the then National Economic Council shall
of Philippine Government Properties in Japan for being discriminatory
determine. Those intended for the private sector shall be made available
against Filipino citizens and Filipino-owned entities by denying them the
by sale to Filipino citizens or to one hundred (100%) percent Filipino-
right to be informed about the bidding requirements.
owned entities in national development projects.

II
The Roppongi property was acquired from the Japanese government
under the Second Year Schedule and listed under the heading
"Government Sector", through Reparations Contract No. 300 dated June In G.R. No. 92013, petitioner Laurel asserts that the Roppongi property
27, 1958. The Roppongi property consists of the land and building "for and the related lots were acquired as part of the reparations from the
the Chancery of the Philippine Embassy" (Annex M-D to Memorandum Japanese government for diplomatic and consular use by the Philippine
for Petitioner, p. 503). As intended, it became the site of the Philippine government. Vice-President Laurel states that the Roppongi property is
Embassy until the latter was transferred to Nampeidai on July 22, 1976 classified as one of public dominion, and not of private ownership under
when the Roppongi building needed major repairs. Due to the failure of Article 420 of the Civil Code (See infra).
our government to provide necessary funds, the Roppongi property has
remained undeveloped since that time.
The petitioner submits that the Roppongi property comes under
"property intended for public service" in paragraph 2 of the above
A proposal was presented to President Corazon C. Aquino by former provision. He states that being one of public dominion, no ownership by
Philippine Ambassador to Japan, Carlos J. Valdez, to make the property any one can attach to it, not even by the State. The Roppongi and related
the subject of a lease agreement with a Japanese firm - Kajima properties were acquired for "sites for chancery, diplomatic, and
Corporation — which shall construct two (2) buildings in Roppongi and consular quarters, buildings and other improvements" (Second Year
one (1) building in Nampeidai and renovate the present Philippine Reparations Schedule). The petitioner states that they continue to be
Chancery in Nampeidai. The consideration of the construction would be intended for a necessary service. They are held by the State in
the lease to the foreign corporation of one (1) of the buildings to be anticipation of an opportune use. (Citing 3 Manresa 65-66). Hence, it
constructed in Roppongi and the two (2) buildings in Nampeidai. The cannot be appropriated, is outside the commerce of man, or to put it in
other building in Roppongi shall then be used as the Philippine Embassy more simple terms, it cannot be alienated nor be the subject matter of
Chancery. At the end of the lease period, all the three leased buildings contracts (Citing Municipality of Cavite v. Rojas, 30 Phil. 20 [1915]).
shall be occupied and used by the Philippine government. No change of Noting the non-use of the Roppongi property at the moment, the
ownership or title shall occur. (See Annex "B" to Reply to Comment) The petitioner avers that the same remains property of public dominion so
Philippine government retains the title all throughout the lease period long as the government has not used it for other purposes nor adopted
and thereafter. However, the government has not acted favorably on this any measure constituting a removal of its original purpose or use.
proposal which is pending approval and ratification between the parties.
Instead, on August 11, 1986, President Aquino created a committee to
The respondents, for their part, refute the petitioner's contention by
study the disposition/utilization of Philippine government properties in
saying that the subject property is not governed by our Civil Code but by
Tokyo and Kobe, Japan through Administrative Order No. 3, followed by
the laws of Japan where the property is located. They rely upon the rule
Administrative Orders Numbered 3-A, B, C and D.
of lex situs  which is used in determining the applicable law regarding the
acquisition, transfer and devolution of the title to a property. They also
On July 25, 1987, the President issued Executive Order No. 296 entitling invoke Opinion No. 21, Series of 1988, dated January 27, 1988 of the
non-Filipino citizens or entities to avail of separations' capital goods and Secretary of Justice which used the lex situs in explaining the
services in the event of sale, lease or disposition. The four properties in inapplicability of Philippine law regarding a property situated in Japan.
Japan including the Roppongi were specifically mentioned in the first
"Whereas" clause.
The respondents add that even assuming for the sake of argument that
the Civil Code is applicable, the Roppongi property has ceased to become
Amidst opposition by various sectors, the Executive branch of the property of public dominion. It has become patrimonial property because
government has been pushing, with great vigor, its decision to sell the it has not been used for public service or for diplomatic purposes for over
reparations properties starting with the Roppongi lot. The property has thirteen (13) years now (Citing Article 422, Civil Code) and because
twice been set for bidding at a minimum floor price of $225 million. The the intention by the Executive Department and the Congress to convert it
first bidding was a failure since only one bidder qualified. The second to private use  has been manifested by overt acts, such as, among others:
one, after postponements, has not yet materialized. The last scheduled (1) the transfer of the Philippine Embassy to Nampeidai (2) the issuance
bidding on February 21, 1990 was restrained by his Court. Later, the of administrative orders for the possibility of alienating the four
rules on bidding were changed such that the $225 million floor price government properties in Japan; (3) the issuance of Executive Order No.
became merely a suggested floor price. 296; (4) the enactment by the Congress of Rep. Act No. 6657 [the
Comprehensive Agrarian Reform Law] on June 10, 1988 which contains
a provision stating that funds may be taken from the sale of Philippine
The Court finds that each of the herein petitions raises distinct issues.
properties in foreign countries; (5) the holding of the public bidding of
The petitioner in G.R. No. 92013 objects to the alienation of the Roppongi
the Roppongi property but which failed; (6) the deferment by the Senate
property to anyone while the petitioner in G.R. No. 92047 adds as a
in Resolution No. 55 of the bidding to a future date; thus an
principal objection the alleged unjustified bias of the Philippine
acknowledgment by the Senate of the government's intention to remove collective ownership for general use and enjoyment, an application to the
the Roppongi property from the public service purpose; and (7) the satisfaction of collective needs, and resides in the social group. The
resolution of this Court dismissing the petition in Ojeda v. Bidding purpose is not to serve the State as a juridical person, but the citizens; it
Committee, et al., G.R. No. 87478 which sought to enjoin the second is intended for the common and public welfare and cannot be the object
bidding of the Roppongi property scheduled on March 30, 1989. of appropration. (Taken from 3 Manresa, 66-69; cited in Tolentino,
Commentaries on the Civil Code of the Philippines, 1963 Edition, Vol. II,
p. 26).
III

The applicable provisions of the Civil Code are:


In G.R. No. 94047, petitioner Ojeda once more asks this Court to rule on
the constitutionality of Executive Order No. 296. He had earlier filed a
petition in G.R. No. 87478 which the Court dismissed on August 1, 1989. ART. 419. Property is either of public dominion or
He now avers that the executive order contravenes the constitutional of private ownership.
mandate to conserve and develop the national patrimony stated in the
Preamble of the 1987 Constitution. It also allegedly violates:
ART. 420. The following things are property of
public dominion
(1) The reservation of the ownership and acquisition of alienable lands of
the public domain to Filipino citizens. (Sections 2 and 3, Article XII,
(1) Those intended for public use, such as roads,
Constitution; Sections 22 and 23 of Commonwealth Act 141).
canals, rivers, torrents, ports and bridges
constructed by the State, banks shores roadsteads,
(2) The preference for Filipino citizens in the grant of rights, privileges and others of similar character;
and concessions covering the national economy and patrimony (Section
10, Article VI, Constitution);
(2) Those which belong to the State, without being
for public use, and are intended for some public
(3) The protection given to Filipino enterprises against unfair competition service or for the development of the national
and trade practices; wealth.

(4) The guarantee of the right of the people to information on all matters ART. 421. All other property of the State, which is
of public concern (Section 7, Article III, Constitution); not of the character stated in the preceding article, is
patrimonial property.
(5) The prohibition against the sale to non-Filipino citizens or entities not
wholly owned by Filipino citizens of capital goods received by the The Roppongi property is correctly classified under paragraph 2 of
Philippines under the Reparations Act (Sections 2 and 12 of Rep. Act No. Article 420 of the Civil Code as property belonging to the State and
1789); and intended for some public service.

(6) The declaration of the state policy of full public disclosure of all Has the intention of the government regarding the use of the property
transactions involving public interest (Section 28, Article III, been changed because the lot has been Idle for some years? Has it become
Constitution). patrimonial?

Petitioner Ojeda warns that the use of public funds in the execution of an The fact that the Roppongi site has not been used for a long time for
unconstitutional executive order is a misapplication of public funds He actual Embassy service does not automatically convert it to patrimonial
states that since the details of the bidding for the Roppongi property property. Any such conversion happens only if the property is withdrawn
were never publicly disclosed until February 15, 1990 (or a few days from public use (Cebu Oxygen and Acetylene Co. v. Bercilles, 66 SCRA
before the scheduled bidding), the bidding guidelines are available only in 481 [1975]). A property continues to be part of the public domain, not
Tokyo, and the accomplishment of requirements and the selection of available for private appropriation or ownership until there is a formal
qualified bidders should be done in Tokyo, interested Filipino citizens or declaration on the part of the government to withdraw it from being such
entities owned by them did not have the chance to comply with Purchase (Ignacio v. Director of Lands, 108 Phil. 335 [1960]).
Offer Requirements on the Roppongi. Worse, the Roppongi shall be sold
for a minimum price of $225 million from which price capital gains tax
The respondents enumerate various pronouncements by concerned
under Japanese law of about 50 to 70% of the floor price would still be
public officials insinuating a change of intention. We emphasize, however,
deducted.
that an abandonment of the intention to use the Roppongi property for
public service and to make it patrimonial property under Article 422 of
IV the Civil Code must be definite Abandonment cannot be inferred from the
non-use alone specially if the non-use was attributable not to the
government's own deliberate and indubitable will but to a lack of
The petitioners and respondents in both cases do not dispute the fact that
financial support to repair and improve the property (See Heirs of Felino
the Roppongi site and the three related properties were through
Santiago v. Lazaro, 166 SCRA 368 [1988]). Abandonment must be a
reparations agreements, that these were assigned to the government
certain and positive act based on correct legal premises.
sector and that the Roppongi property itself was specifically designated
under the Reparations Agreement to house the Philippine Embassy.
A mere transfer of the Philippine Embassy to Nampeidai in 1976 is not
relinquishment of the Roppongi property's original purpose. Even the
The nature of the Roppongi lot as property for public service is expressly
failure by the government to repair the building in Roppongi is not
spelled out. It is dictated by the terms of the Reparations Agreement and
abandonment since as earlier stated, there simply was a shortage of
the corresponding contract of procurement which bind both the
government funds. The recent Administrative Orders authorizing a study
Philippine government and the Japanese government.
of the status and conditions of government properties in Japan were
merely directives for investigation but did not in any way signify a clear
There can be no doubt that it is of public dominion unless it is intention to dispose of the properties.
convincingly shown that the property has become patrimonial. This, the
respondents have failed to do.
Executive Order No. 296, though its title declares an "authority to sell",
does not have a provision in its text expressly authorizing the sale of the
As property of public dominion, the Roppongi lot is outside the four properties procured from Japan for the government sector. The
commerce of man. It cannot be alienated. Its ownership is a special executive order does not declare that the properties lost their public
character. It merely intends to make the properties available to
foreigners and not to Filipinos alone in case of a sale, lease or other it is the foreign law which should determine who can acquire the
disposition. It merely eliminates the restriction under Rep. Act No. 1789 properties  so that the constitutional limitation on acquisition of lands of
that reparations goods may be sold only to Filipino citizens and one the public domain to Filipino citizens and entities wholly owned by
hundred (100%) percent Filipino-owned entities. The text of Executive Filipinos is inapplicable. We see no point in belaboring whether or not
Order No. 296 provides: this opinion is correct. Why should we discuss who can acquire the
Roppongi lot when there is no showing that it can be sold?
Section 1. The provisions of Republic Act No. 1789,
as amended, and of other laws to the contrary The subsequent approval on October 4, 1988 by President Aquino of the
notwithstanding, the above-mentioned properties recommendation by the investigating committee to sell the Roppongi
can be made available for sale, lease or any other property was premature or, at the very least, conditioned on a valid
manner of disposition to non-Filipino citizens or to change in the public character of the Roppongi property. Moreover, the
entities owned by non-Filipino citizens. approval does not have the force and effect of law since the President
already lost her legislative powers. The Congress had already convened
for more than a year.
Executive Order No. 296 is based on the wrong premise or assumption
that the Roppongi and the three other properties were earlier converted
into alienable real properties. As earlier stated, Rep. Act No. 1789 Assuming for the sake of argument, however, that the Roppongi property
differentiates the procurements for the government sector and the private is no longer of public dominion, there is another obstacle to its sale by the
sector (Sections 2 and 12, Rep. Act No. 1789). Only the private sector respondents.
properties can be sold to end-users who must be Filipinos or entities
owned by Filipinos. It is this nationality provision which was amended by
There is no law authorizing its conveyance.
Executive Order No. 296.

Section 79 (f) of the Revised Administrative Code of 1917 provides


Section 63 (c) of Rep. Act No. 6657 (the CARP Law) which provides as
one of the sources of funds for its implementation, the proceeds of the
disposition of the properties of the Government in foreign countries, did Section 79 (f ) Conveyances and contracts to which
not withdraw the Roppongi property from being classified as one of the Government is a party. — In cases in which the
public dominion when it mentions Philippine properties abroad. Section Government of the Republic of the Philippines is a
63 (c) refers to properties which are alienable and not to those reserved party to any deed or other instrument conveying the
for public use or service. Rep Act No. 6657, therefore, does not authorize title to real estate or to any other property the value
the Executive Department to sell the Roppongi property. It merely of which is in excess of one hundred thousand pesos,
enumerates possible sources of future funding to augment (as and when the respective Department Secretary shall prepare
needed) the Agrarian Reform Fund created under Executive Order No. the necessary papers which, together with the
299. Obviously any property outside of the commerce of man cannot be proper recommendations, shall be submitted to the
tapped as a source of funds. Congress of the Philippines for approval by the
same. Such deed, instrument, or contract shall be
executed and signed by the President of the
The respondents try to get around the public dominion character of the
Philippines on behalf of the Government of the
Roppongi property by insisting that Japanese law and not our Civil Code
Philippines unless the Government of the
should apply.
Philippines unless the authority therefor be
expressly vested by law in another officer.
It is exceedingly strange why our top government officials, of all people, (Emphasis supplied)
should be the ones to insist that in the sale of extremely valuable
government property, Japanese law and not Philippine law should
The requirement has been retained in Section 48, Book I of the
prevail. The Japanese law - its coverage and effects, when enacted, and
Administrative Code of 1987 (Executive Order No. 292).
exceptions to its provision — is not presented to the Court It is simply
asserted that the lex loci rei sitae or Japanese law should apply without
stating what that law provides. It is a ed on faith that Japanese law would SEC. 48. Official Authorized to Convey Real Property.
allow the sale. — Whenever real property of the Government
is authorized by law to be conveyed, the deed of
conveyance shall be executed in behalf of the
We see no reason why a conflict of law rule should apply when no conflict
government by the following:
of law situation exists. A conflict of law situation arises only when: (1)
There is a dispute over the title or ownership  of an immovable, such that
the capacity to take and transfer immovables, the formalities of (1) For property belonging to and titled in the name
conveyance, the essential validity and effect of the transfer, or the of the Republic of the Philippines, by the President,
interpretation and effect of a conveyance, are to be determined (See unless the authority therefor is expressly vested by
Salonga, Private International Law, 1981 ed., pp. 377-383); and (2) A law in another officer.
foreign law on land ownership and its conveyance is asserted to conflict
with a domestic law on the same matters. Hence, the need to determine
(2) For property belonging to the Republic of the
which law should apply.
Philippines but titled in the name of any political
subdivision or of any corporate agency or
In the instant case, none of the above elements exists. instrumentality, by the executive head of the agency
or instrumentality. (Emphasis supplied)
The issues are not concerned with validity of ownership or title. There is
no question that the property belongs to the Philippines. The issue is the It is not for the President to convey valuable real property of the
authority of the respondent officials to validly dispose of property government on his or her own sole will. Any such conveyance must be
belonging to the State. And the validity of the procedures adopted to authorized and approved by a law enacted by the Congress. It requires
effect its sale. This is governed by Philippine Law. The rule of lex executive and legislative concurrence.
situs does not apply.
Resolution No. 55 of the Senate dated June 8, 1989, asking for the
The assertion that the opinion of the Secretary of Justice sheds light on deferment of the sale of the Roppongi property does not withdraw the
the relevance of the lex situs rule is misplaced. The opinion does not property from public domain much less authorize its sale. It is a mere
tackle the alienability of the real properties procured through reparations resolution; it is not a formal declaration abandoning the public character
nor the existence in what body of the authority to sell them. In discussing of the Roppongi property. In fact, the Senate Committee on Foreign
who are capable of acquiring the lots, the Secretary merely explains that Relations is conducting hearings on Senate Resolution No. 734 which
raises serious policy considerations and calls for a fact-finding It is for what it stands for, and for what it could
investigation of the circumstances behind the decision to sell the never bring back to life, that its significance today
Philippine government properties in Japan. remains undimmed, inspire of the lapse of 45 years
since the war ended, inspire of the passage of 32
years since the property passed on to the Philippine
The resolution of this Court in Ojeda v. Bidding Committee, et al.,
government.
supra,  did not pass upon the constitutionality of Executive Order No. 296.
Contrary to respondents' assertion, we did not uphold the authority of
the President to sell the Roppongi property. The Court stated that the Roppongi is a reminder that cannot — should not —
constitutionality of the executive order was not the real issue and that be dissipated ... (Rollo-92047, p. 9)
resolving the constitutional question was "neither necessary nor finally
determinative of the case." The Court noted that "[W]hat petitioner
It is indeed true that the Roppongi property is valuable not so much
ultimately questions is the use of the proceeds of the disposition of the
because of the inflated prices fetched by real property in Tokyo but more
Roppongi property." In emphasizing that "the decision of the Executive
so because of its symbolic value to all Filipinos — veterans and civilians
to dispose of the Roppongi property to finance the CARP ... cannot be
alike. Whether or not the Roppongi and related properties will eventually
questioned" in view of Section 63 (c) of Rep. Act No. 6657, the Court did
be sold is a policy determination where both the President and Congress
not acknowledge the fact that the property became alienable nor did it
must concur. Considering the properties' importance and value, the laws
indicate that the President was authorized to dispose of the Roppongi
on conversion and disposition of property of public dominion must be
property. The resolution should be read to mean that in case the
faithfully followed.
Roppongi property is re-classified to be patrimonial and alienable by
authority of law, the proceeds of a sale may be used for national economic
development projects including the CARP. WHEREFORE, IN VIEW OF THE FOREGOING, the petitions are
GRANTED. A writ of prohibition is issued enjoining the respondents
from proceeding with the sale of the Roppongi property in Tokyo, Japan.
Moreover, the sale in 1989 did not materialize. The petitions before us
The February 20, 1990 Temporary Restraining Order is made
question the proposed 1990 sale of the Roppongi property. We are
PERMANENT.
resolving the issues raised in these petitions, not the issues raised in 1989.

SO ORDERED.
Having declared a need for a law or formal declaration to withdraw the
Roppongi property from public domain to make it alienable and a need
for legislative authority to allow the sale of the property, we see no
compelling reason to tackle the constitutional issues raised by petitioner
Ojeda.

The Court does not ordinarily pass upon constitutional questions unless
these questions are properly raised in appropriate cases and their
resolution is necessary for the determination of the case (People v. Vera,
65 Phil. 56 [1937]). The Court will not pass upon a constitutional question
although properly presented by the record if the case can be disposed of
on some other ground such as the application of a statute or general law
(Siler v. Louisville and Nashville R. Co., 213 U.S. 175, [1909], Railroad
Commission v. Pullman Co., 312 U.S. 496 [1941]).

The petitioner in G.R. No. 92013 states why the Roppongi property
should not be sold:

The Roppongi property is not just like any piece of


property. It was given to the Filipino people in
reparation for the lives and blood of Filipinos who
died and suffered during the Japanese military
occupation, for the suffering of widows and orphans
who lost their loved ones and kindred, for the homes
and other properties lost by countless Filipinos
during the war. The Tokyo properties are a
monument to the bravery and sacrifice of the
Filipino people in the face of an invader; like the
monuments of Rizal, Quezon, and other Filipino
heroes, we do not expect economic or financial
benefits from them. But who would think of selling
these monuments? Filipino honor and national
dignity dictate that we keep our properties in Japan
as memorials to the countless Filipinos who died and G.R. No. L-24440             March 28, 1968
suffered. Even if we should become paupers we
should not think of selling them. For it would be as
if we sold the lives and blood and tears of our THE PROVINCE OF ZAMBOANGA DEL NORTE, plaintiff-appellee,
countrymen. (Rollo- G.R. No. 92013, p.147) vs.
CITY OF ZAMBOANGA, SECRETARY OF FINANCE and
COMMISSIONER OF INTERNAL REVENUE, defendants-appellants.
The petitioner in G.R. No. 92047 also states:

Prior to its incorporation as a chartered city, the Municipality of Zamboanga


Roppongi is no ordinary property. It is one ceded by used to be the provincial capital of the then Zamboanga Province. On October
the Japanese government in atonement for its past 12, 1936, Commonwealth Act 39 was approved converting the Municipality
belligerence for the valiant sacrifice of life and limb of Zamboanga into Zamboanga City. Sec. 50 of the Act also provided that —
and for deaths, physical dislocation and economic
devastation the whole Filipino people endured in
World War II.           Buildings and properties which the province shall abandon
upon the transfer of the capital to another place will be acquired
and paid for by the City of Zamboanga at a price to be fixed by the           All buildings, properties and assets belonging to the former
Auditor General. province of Zamboanga and located within the City of Zamboanga
are hereby transferred, free of charge, in favor of the said City of
Zamboanga. (Stressed for emphasis).
          The properties and buildings referred to consisted of 50 lots and some
buildings constructed thereon, located in the City of Zamboanga and covered
individually by Torrens certificates of title in the name of Zamboanga           Consequently, the Secretary of Finance, on July 12, 1961, ordered the
Province. As far as can be gleaned from the records, 1 said properties were Commissioner of Internal Revenue to stop from effecting further payments to
being utilized as follows — Zamboanga del Norte and to return to Zamboanga City the sum of P57,373.46
taken from it out of the internal revenue allotment of Zamboanga del Norte.
Zamboanga City admits that since the enactment of Republic Act 3039,
No. of Lots Use P43,030.11 of the P57,373.46 has already been returned to it.
1 ................................................ Capitol Site
3 ................................................ School Site
          This constrained plaintiff-appellee Zamboanga del Norte to file on
3 ................................................ Hospital Site March 5, 1962, a complaint entitled "Declaratory Relief with Preliminary
3 ................................................ Leprosarium Mandatory Injunction" in the Court of First Instance of Zamboanga del Norte
1 ................................................ Curuan School against defendants-appellants Zamboanga City, the Secretary of Finance and
1 ................................................ Trade School the Commissioner of Internal Revenue. It was prayed that: (a) Republic Act
2 ................................................ Burleigh School 3039 be declared unconstitutional for depriving plaintiff province of property
2 ................................................HighSchool Playground without due process and just compensation; (b) Plaintiff's rights and
obligations under said law be declared; (c) The Secretary of Finance and the
9 ................................................ Burleighs
Internal Revenue Commissioner be enjoined from reimbursing the sum of
1 ................................................ Hydro-Electric Site (Magay) P57,373.46 to defendant City; and (d) The latter be ordered to continue paying
1 ................................................ San Roque the balance of P704,220.05 in quarterly installments of 25% of its internal
23 ................................................ vacant revenue allotments.
          It appears that in 1945, the capital of Zamboanga Province was
transferred to Dipolog. 2 Subsequently, or on June 16, 1948, Republic Act 286           On June 4, 1962, the lower court ordered the issuance of preliminary
was approved creating the municipality of Molave and making it the capital of injunction as prayed for. After defendants filed their respective answers, trial
Zamboanga Province. was held. On August 12, 1963, judgment was rendered, the dispositive portion
of which reads:
          On May 26, 1949, the Appraisal Committee formed by the Auditor
General, pursuant to Commonwealth Act 39, fixed the value of the properties           WHEREFORE, judgment is hereby rendered declaring
and buildings in question left by Zamboanga Province in Zamboanga City at Republic Act No. 3039 unconstitutional insofar as it deprives
P1,294,244.00. 3 plaintiff Zamboanga del Norte of its private properties, consisting
of 50 parcels of land and the improvements thereon under
          On June 6, 1952, Republic Act 711 was approved dividing the province certificates of title (Exhibits "A" to "A-49") in the name of the
of Zamboanga into two (2): Zamboanga del Norte and Zamboanga del Sur. As defunct province of Zamboanga; ordering defendant City of
to how the assets and obligations of the old province were to be divided Zamboanga to pay to the plaintiff the sum of P704,220.05 payment
between the two new ones, Sec. 6 of that law provided: thereof to be deducted from its regular quarterly internal revenue
allotment equivalent to 25% thereof every quarter until said
amount shall have been fully paid; ordering defendant Secretary of
          Upon the approval of this Act, the funds, assets and other Finance to direct defendant Commissioner of Internal Revenue to
properties and the obligations of the province of Zamboanga shall deduct 25% from the regular quarterly internal revenue allotment
be divided equitably between the Province of Zamboanga del for defendant City of Zamboanga and to remit the same to plaintiff
Norte and the Province of Zamboanga del Sur by the President of Zamboanga del Norte until said sum of P704,220.05 shall have
the Philippines, upon the recommendation of the Auditor General. been fully paid; ordering plaintiff Zamboanga del Norte to execute
through its proper officials the corresponding public instrument
          Pursuant thereto, the Auditor General, on January 11, 1955, apportioned deeding to defendant City of Zamboanga the 50 parcels of land and
the assets and obligations of the defunct Province of Zamboanga as follows: the improvements thereon under the certificates of title (Exhibits
54.39% for Zamboanga del Norte and 45.61% for Zamboanga del Sur. "A" to "A-49") upon payment by the latter of the aforesaid sum of
Zamboanga del Norte therefore became entitled to 54.39% of P1,294,244.00, P704,220.05 in full; dismissing the counterclaim of defendant City
the total value of the lots and buildings in question, or P704,220.05 payable of Zamboanga; and declaring permanent the preliminary
by Zamboanga City. mandatory injunction issued on June 8, 1962, pursuant to the order
of the Court dated June 4, 1962. No costs are assessed against the
defendants.
          On March 17, 1959, the Executive Secretary, by order of the President,
issued a ruling 4 holding that Zamboanga del Norte had a vested right as owner
(should be co-owner pro-indiviso) of the properties mentioned in Sec. 50 of           It is SO ORDERED.
Commonwealth Act 39, and is entitled to the price thereof, payable by
Zamboanga City. This ruling revoked the previous Cabinet Resolution of July           Subsequently, but prior to the perfection of defendants' appeal, plaintiff
13, 1951 conveying all the said 50 lots and buildings thereon to Zamboanga province filed a motion to reconsider praying that Zamboanga City be ordered
City for P1.00, effective as of 1945, when the provincial capital of the then instead to pay the P704,220.05 in lump sum with 6% interest per annum. Over
Zamboanga Province was transferred to Dipolog. defendants' opposition, the lower court granted plaintiff province's motion.

          The Secretary of Finance then authorized the Commissioner of Internal           The defendants then brought the case before Us on appeal.
Revenue to deduct an amount equal to 25% of the regular internal revenue
allotment for the City of Zamboanga for the quarter ending March 31, 1960,
then for the quarter ending June 30, 1960, and again for the first quarter of the           Brushing aside the procedural point concerning the property of
fiscal year 1960-1961. The deductions, all aggregating P57,373.46, was declaratory relief filed in the lower court on the assertion that the law had
credited to the province of Zamboanga del Norte, in partial payment of the already been violated and that plaintiff sought to give it coercive effect, since
P764,220.05 due it. assuming the same to be true, the Rules anyway authorize the conversion of
the proceedings to an ordinary action, 5 We proceed to the more important and
principal question of the validity of Republic Act 3039.
          However, on June 17, 1961, Republic Act 3039 was approved
amending Sec. 50 of Commonwealth Act 39 by providing that —
          The validity of the law ultimately depends on the nature of the 50 lots
and buildings thereon in question. For, the matter involved here is the extent
of legislative control over the properties of a municipal corporation, of which VS. CANTOS 13 held squarely that a municipal lot which had always been
a province is one. The principle itself is simple: If the property is owned by devoted to school purposes is one dedicated to public use and is not
the municipality (meaning municipal corporation) in its public and patrimonial property of a municipality.
governmental capacity, the property is public and Congress has absolute
control over it. But if the property is owned in its private or proprietary
          Following this classification, Republic Act 3039 is valid insofar as it
capacity, then it is patrimonial and Congress has no absolute control. The
affects the lots used as capitol site, school sites and its grounds, hospital and
municipality cannot be deprived of it without due process and payment of just
leprosarium sites and the high school playground sites — a total of 24 lots —
compensation. 6
since these were held by the former Zamboanga province in its governmental
capacity and therefore are subject to the absolute control of Congress. Said
          The capacity in which the property is held is, however, dependent on lots considered as public property are the following:
the use to which it is intended and devoted. Now, which of two norms, i.e.,
that of the Civil Code or that obtaining under the law of Municipal
Corporations, must be used in classifying the properties in question? TCT
Lot Number Use
Number
..................................... .....................................
          The Civil Code classification is embodied in its Arts. 423 and 424 2200 4-B Capitol Site
. .
which provide: ..................................... .....................................
2816 149 School Site
. .
          ART. 423. The property of provinces, cities, and ..................................... ..................................... Hospital
3281 1224
municipalities is divided into property for public use and . . Site
patrimonial property. ..................................... ..................................... Hospital
3282 1226
. . Site
          ART. 424. Property for public use, in the provinces, cities, ..................................... ..................................... Hospital
3283 1225
and municipalities, consists of the provincial roads, city streets, . . Site
municipal streets, the squares, fountains, public waters, ..................................... .....................................
3748 434-A-1 School Site
promenades, and public works for public service paid for by said . .
provinces, cities, or municipalities. ..................................... .....................................
5406 171 School Site
. .
All other property possessed by any of them is patrimonial and ..................................... ..................................... High School
5564 168
shall be governed by this Code, without prejudice to the provisions . . Play-ground
of special laws. (Stressed for emphasis). ..................................... 157 & ..................................... Trade
5567
. 158 . School
          Applying the above cited norm, all the properties in question, except the ..................................... ..................................... High School
5583 167
two (2) lots used as High School playgrounds, could be considered as . . Play-ground
patrimonial properties of the former Zamboanga province. Even the capital ..................................... ..................................... Curuan
6181 (O.C.T.)
site, the hospital and leprosarium sites, and the school sites will be considered . . School
patrimonial for they are not for public use. They would fall under the phrase ..................................... .....................................
11942 926 Leprosarium
"public works for public service" for it has been held that under the ejusdem . .
generis rule, such public works must be for free and indiscriminate use by ..................................... .....................................
anyone, just like the preceding enumerated properties in the first paragraph of 11943 927 Leprosarium
. .
Art 424. 7 The playgrounds, however, would fit into this category. ..................................... .....................................
11944 925 Leprosarium
. .
          This was the norm applied by the lower court. And it cannot be said that ..................................... ..................................... Burleigh
5557 170
its actuation was without jurisprudential precedent for in Municipality of . . School
Catbalogan v. Director of Lands, 8 and in Municipality of Tacloban v. ..................................... ..................................... Burleigh
Director of Lands, 9 it was held that the capitol site and the school sites in 5562 180
. . School
municipalities constitute their patrimonial properties. This result is ..................................... .....................................
understandable because, unlike in the classification regarding State properties, 5565 172-B Burleigh
. .
properties for public service in the municipalities are not classified as public. ..................................... .....................................
Assuming then the Civil Code classification to be the chosen norm, the lower 5570 171-A Burleigh
. .
court must be affirmed except with regard to the two (2) lots used as
playgrounds. ..................................... .....................................
5571 172-C Burleigh
. .
..................................... .....................................
          On the other hand, applying the norm obtaining under the principles 5572 174 Burleigh
. .
constituting the law of Municipal Corporations, all those of the 50 properties ..................................... .....................................
in question which are devoted to public service are deemed public; the rest 5573 178 Burleigh
. .
remain patrimonial. Under this norm, to be considered public, it is enough that
..................................... .....................................
the property be held and, devoted for governmental purposes like local 5585 171-B Burleigh
. .
administration, public education, public health, etc. 10
..................................... .....................................
5586 173 Burleigh
. .
          Supporting jurisprudence are found in the following cases: (1) ..................................... .....................................
HINUNANGAN V. DIRECTOR OF LANDS, 11 where it was stated that "... 5587 172-A Burleigh
. .
where the municipality has occupied lands distinctly for public purposes, such           We noticed that the eight Burleigh lots above described are adjoining
as for the municipal court house, the public school, the public market, or other each other and in turn are between the two lots wherein the Burleigh schools
necessary municipal building, we will, in the absence of proof to the contrary, are built, as per records appearing herein and in the Bureau of Lands. Hence,
presume a grant from the States in favor of the municipality; but, as indicated there is sufficient basis for holding that said eight lots constitute the
by the wording, that rule may be invoked only as to property which is used appurtenant grounds of the Burleigh schools, and partake of the nature of the
distinctly for public purposes...." (2) VIUDA DE TANTOCO V. same.
MUNICIPAL COUNCIL OF ILOILO 12 held that municipal properties
necessary for governmental purposes are public in nature. Thus, the auto
trucks used by the municipality for street sprinkling, the police patrol           Regarding the several buildings existing on the lots above-mentioned,
automobile, police stations and concrete structures with the corresponding lots the records do not disclose whether they were constructed at the expense of
used as markets were declared exempt from execution and attachment since the former Province of Zamboanga. Considering however the fact that said
they were not patrimonial properties. (3) MUNICIPALITY OF BATANGAS buildings must have been erected even before 1936 when Commonwealth Act
39 was enacted and the further fact that provinces then had no power to           Defendants' claim that plaintiff and its predecessor-in-interest are
authorize construction of buildings such as those in the case at bar at their "guilty of laches is without merit. Under Commonwealth Act 39, Sec. 50, the
own expense, 14 it can be assumed that said buildings were erected by the cause of action in favor of the defunct Zamboanga Province arose only in
National Government, using national funds. Hence, Congress could very well 1949 after the Auditor General fixed the value of the properties in question.
dispose of said buildings in the same manner that it did with the lots in While in 1951, the Cabinet resolved transfer said properties practically for
question. free to Zamboanga City, a reconsideration thereof was seasonably sought. In
1952, the old province was dissolved. As successor-in-interest to more than
half of the properties involved, Zamboanga del Norte was able to get a
          But even assuming that provincial funds were used, still the buildings
reconsideration of the Cabinet Resolution in 1959. In fact, partial payments
constitute mere accessories to the lands, which are public in nature, and so,
were effected subsequently and it was only after the passage of Republic Act
they follow the nature of said lands, i.e., public. Moreover, said buildings,
3039 in 1961 that the present controversy arose. Plaintiff brought suit in 1962.
though located in the city, will not be for the exclusive use and benefit of city
All the foregoing, negative laches.
residents for they could be availed of also by the provincial residents. The
province then — and its successors-in-interest — are not really deprived of
the benefits thereof.           It results then that Zamboanga del Norte is still entitled to collect from
the City of Zamboanga the former's 54.39% share in the 26 properties which
are patrimonial in nature, said share to computed on the basis of the valuation
          But Republic Act 3039 cannot be applied to deprive Zamboanga del
of said 26 properties as contained in Resolution No. 7, dated March 26, 1949,
Norte of its share in the value of the rest of the 26 remaining lots which are
of the Appraisal Committee formed by the Auditor General.
patrimonial properties since they are not being utilized for distinctly,
governmental purposes. Said lots are:
          Plaintiff's share, however, cannot be paid in lump sum, except as to the
P43,030.11 already returned to defendant City. The return of said amount to
TCT Number Lot Number Use defendant was without legal basis. Republic Act 3039 took effect only on June
Mydro, 17, 1961 after a partial payment of P57,373.46 had already been made. Since
5577 ...................................... 177 ......................................
Magay the law did not provide for retroactivity, it could not have validly affected a
127- completed act. Hence, the amount of P43,030.11 should be immediately
13198 ...................................... ...................................... San Roque
0 returned by defendant City to plaintiff province. The remaining balance, if
5569 ...................................... 169 ...................................... Burleigh 15 any, in the amount of plaintiff's 54.39% share in the 26 lots should then be
5558 ...................................... 175 ...................................... Vacant paid by defendant City in the same manner originally adopted by the Secretary
5559 ...................................... 188 ...................................... " of Finance and the Commissioner of Internal Revenue, and not in lump sum.
Plaintiff's prayer, particularly pars. 5 and 6, read together with pars. 10 and 11
5560 ...................................... 183 ...................................... " of the first cause of action recited in the complaint 17 clearly shows that the
5561 ...................................... 186 ...................................... " relief sought was merely the continuance of the quarterly payments from the
5563 ...................................... 191 ...................................... " internal revenue allotments of defendant City. Art. 1169 of the Civil Code on
5566 ...................................... 176 ...................................... " reciprocal obligations invoked by plaintiff to justify lump sum payment is
5568 ...................................... 179 ...................................... " inapplicable since there has been so far in legal contemplation no complete
5574 ...................................... 196 ...................................... " delivery of the lots in question. The titles to the registered lots are not yet in
the name of defendant Zamboanga City.
181-
5575 ...................................... ...................................... "
A
181-           WHEREFORE, the decision appealed from is hereby set aside and
5576 ...................................... ...................................... " another judgment is hereby entered as follows:.
B
5578 ...................................... 182 ...................................... "
5579 ...................................... 197 ...................................... "           (1) Defendant Zamboanga City is hereby ordered to return to plaintiff
5580 ...................................... 195 ...................................... " Zamboanga del Norte in lump sum the amount of P43,030.11 which the
159- former took back from the latter out of the sum of P57,373.46 previously paid
5581 ...................................... ...................................... " to the latter; and
B
5582 ...................................... 194 ...................................... "
5584 ...................................... 190 ...................................... "           (2) Defendants are hereby ordered to effect payments in favor of
5588 ...................................... 184 ...................................... " plaintiff of whatever balance remains of plaintiff's 54.39% share in the 26
patrimonial properties, after deducting therefrom the sum of P57,373.46, on
5589 ...................................... 187 ...................................... "
the basis of Resolution No. 7 dated March 26, 1949 of the Appraisal
5590 ...................................... 189 ...................................... " Committee formed by the Auditor General, by way of quarterly payments
5591 ...................................... 192 ...................................... " from the allotments of defendant City, in the manner originally adopted by the
5592 ...................................... 193 ...................................... " Secretary of Finance and the Commissioner of Internal Revenue. No costs. So
5593 ...................................... 185 ...................................... " ordered.
7379 ...................................... 4147 ...................................... "
          Moreover, the fact that these 26 lots are registered  strengthens the
proposition that they are truly private in nature. On the other hand, that the 24
lots used for governmental purposes are also registered is of no significance
since registration cannot convert public property to private. 16

          We are more inclined to uphold this latter view. The controversy here is G.R. No. 218269, June 06, 2018
more along the domains of the Law of Municipal Corporations — State vs.
Province — than along that of Civil Law. Moreover, this Court is not inclined IN RE: APPLICATION FOR LAND REGISTRATION
to hold that municipal property held and devoted to public service is in the
same category as ordinary private property. The consequences are dire. As
ordinary private properties, they can be levied upon and attached. They can SUPREMA T. DUMO, Petitioner, v. REPUBLIC OF THE
even be acquired thru adverse possession — all these to the detriment of the PHILIPPINES, Respondent.
local community. Lastly, the classification of properties other than those for
public use in the municipalities as patrimonial under Art. 424 of the Civil DECISION
Code — is "... without prejudice to the provisions of special laws." For
purpose of this article, the principles, obtaining under the Law of Municipal
Corporations can be considered as "special laws". Hence, the classification of The Case
municipal property devoted for distinctly governmental purposes as public
should prevail over the Civil Code classification in this particular case.
This is a petition for review on certiorari under Rule 45 of the Rules of Court. siblings. Dumo traces her title from her mother, Trinidad, who purchased the
Petitioner Suprema T. Dumo (Dumo) challenges the 28 January 2014 lots from Florencio Mabalay in August 1951. Mabalay was Dumo's maternal
Decision1 and the 19 May 2015 Resolution2 of the Court of Appeals (CA) in grandfather. Mabalay, on the other hand, purchased the properties from Carlos
CA-G.R. CV No. 95732, which modified the Joint Decision of the Regional Calica.
Trial Court (RTC), Branch 67, Bauang, La Union, in Civil Case No. 1301-Bg
for Accion Reivindicatoria3 and LRC Case No. 270-Bg for Application for
The heirs of Espinas opposed Dumo's application for land registration on the
Land Registration.4
ground that the properties sought to be registered by Dumo are involved in
the accion reivindicatoria case. Thus, the RTC consolidated the land
The Facts registration case with the Complaint for Recovery of Ownership, Possession
and Damages.
Severa Espinas, Erlinda Espinas, Aurora Espinas, and Virginia Espinas filed a
Complaint for Recovery of Ownership, Possession and Damages with Prayer The Office of the Solicitor General entered its appearance and filed its
for Writ of Preliminary Injunction against the heirs of Bernarda M. Trinidad opposition for the State in the land registration case.
(Trinidad), namely, Leticia T. Valmonte, Lydia T. Nebab, Purita T. Tanag,
Gloria T. Antolin, Nilo Trinidad, Elpidio Trinidad, Fresnida T. Saldana,
The Ruling of the RTC
Nefresha T. Tolentino, and Dumo. The plaintiffs are the heirs of Marcelino
Espinas (Espinas), who died intestate on 6 November 1991, leaving a parcel
of land (Subject Property) covered by Tax Declaration No. 13823-A, which On 2 July 2010, the RTC rendered its Joint Decision, finding that the Subject
particularly described the property as follows: Property was owned by the heirs of Espinas. The RTC ordered the dismissal
of Dumo's land registration application on the ground of lack of registerable
title, and ordered Dumo to restore ownership and possession of the lots to the
A parcel of land located [in] Paringao, Bauang, La Union classified as
heirs of Espinas. The dispositive portion of the Joint Decision reads:
unirrigated Riceland with an area of 1,065 square meters covered by Tax
Declaration No. 13823-A, bounded on the North by Felizarda N. Mabalay; on
the East by Pedro Trinidad; on the South by Girl Scout[s] of the Philippines WHEREFORE, premises considered[,] judgment is rendered:
and on the West by China Sea and assessed at P460.00.5
In LRC Case No. 270-Bg: Ordering the dismissal of the land registration on
The Subject Property was purchased by Espinas from Carlos Calica through a [the] ground of lack of registerable title on the part of Suprema Dumo.
Deed of Absolute Sale dated 19 October 1943. Espinas exercised acts of
dominion over the Subject Property by appointing a caretaker to oversee and
In Civil Case No. 1301-Bg: Declaring the Heirs of Marcelino Espinas as the
administer the property. In 1963, Espinas executed an affidavit stating his
owners of the lots subject of [the] application; ordering the applicant-
claim of ownership over the Subject Property. Espinas had also been paying
defendant Suprema Dumo to restore ownership and possession of the lots in
realty taxes on the Subject Property.
question to the Heirs of Marcelino Espinas.

Meanwhile, on 6 February 1987, the heirs of Trinidad executed a Deed of


SO ORDERED.7
Partition with Absolute Sale over a parcel of land covered by Tax Declaration
No. 17276, which particularly described the property as follows:
The RTC found that based on the evidence presented, the heirs of Espinas had
a better right to the Subject Property. In particular, the RTC found that based
A parcel of sandy land located [in] Paringao, Bauang, La Union, bounded on
on the records of the Bureau of Lands, the lot of Espinas was previously
the North by Emiliana Estepa, on the South by Carlos Calica and Girl Scout[s]
surveyed and approved by the Bureau of Lands and when the survey was
Camp and on the West by China Sea, containing an area of 1[,]514 square
made for Trinidad, there was already an approved plan for Espinas. Also, the
meters more or less, with an assessed value [of] P130.00.6
RTC found that the tax declarations submitted by Dumo in support of her
application failed to prove any rights over the land. Specifically, the tax
Finding that the Deed of Partition with Absolute Sale executed by the heirs of declaration of Mabalay, from whom Dumo traces her title, showed that the
Trinidad included the Subject Property, the heirs of Espinas filed a Complaint land was first described as bounded on the west by Espinas. The subsequent
for Recovery of Ownership, Possession and Damages to protect their interests tax declaration in the name of Trinidad, which cancelled the tax declaration in
(Civil Case No. 1301-Bg). The heirs of Espinas also sought a Temporary the name of Mabalay, showed that the land was no longer bounded on the
Restraining Order to enjoin the Writ of Partial Execution of the Decision in west by Espinas, but rather, by the China Sea. The area of the lot also
Civil Case No. 881, a Forcible Entry complaint filed by the heirs of Trinidad increased from 3,881 to 5,589 square meters. All of the subsequent tax
against them. declarations submitted by Dumo covering the lot in the name of her mother
stated that the lot was no longer bounded on the west by Espinas, but rather,
by the China Sea. The RTC held that the only logical explanation to the
In the Complaint for Recovery of Ownership, Possession and Damages,
inconsistency in the description of the land and the corresponding area thereof
Dumo, one of the defendants therein, filed a Motion to Dismiss based on res
is that the lot of Espinas was included in the survey conducted for Trinidad.
judicata. Dumo argued that Espinas had already applied for the registration of
the Subject Property and that such application had been dismissed. The
dismissal of the land registration application of Espinas was affirmed by the The RTC also rejected the theory of Dumo that the lot of Espinas was eaten
CA, and attained finality on 5 December 1980. by the sea. The RTC found that during the ocular inspection, it was
established that the lots adjoining the lot of Espinas on the same shoreline
were not inundated by the sea. To hold the theory posited by Dumo to be true,
The Motion to Dismiss filed by Dumo was denied by the RTC, which held
the RTC reasoned that all the adjoining lots should also have been inundated
that the land registration case cannot operate as a bar to the Complaint for
by the sea. However, it was established through the ocular inspection that the
Recovery of Ownership, Possession and Damages because the decision in the
lots adjoining the property of Espinas on the same shoreline remained the
land registration case did not definitively and conclusively adjudicate the
same, and thus the Subject Property had not been eaten by the sea.
ownership of the Subject Property in favor of any of the parties.

The Ruling of the CA


The heirs of Trinidad thereafter filed their collective Answer, where they
denied the material allegations in the complaint.
The CA rendered its Decision dated 28 January 2014, affirming the RTC's
decision dismissing the application for land registration of Dumo, and finding
Additionally, Dumo filed an application for registration of two parcels of land,
that she failed to demonstrate that she and her predecessors-in interest
covered by Advance Plan of Lot Nos. 400398 and 400399 with a total area of
possessed the property in the manner required by law to merit the grant of her
1,273 square meters (LRC Case No. 270-Bg). Dumo alleged that the lots
application for land registration.
belonged to her mother and that she and her siblings inherited them upon their
mother's death. She further alleged that through a Deed of Partition with
Absolute Sale dated 6 February 1987, she acquired the subject lots from her
The CA, however, modified the decision of the RTC insofar as it found that Essentially, Dumo argues that the CA committed a reversible error because
the Subject Property belonged to the heirs of Espinas. The CA found that (1) the issue of whether she was in open, continuous, exclusive and notorious
since the property still belonged to the public domain, and the heirs of Espinas possession of the land since 12 June 1945 was not an issue in the RTC; (2) the
were not able to establish their open, continuous, exclusive and notorious requirement of possession and occupation from 12 June 1945 is not essential
possession and occupation of the land under a bona fide claim of ownership to her application since she has acquired title over the land by prescription; (3)
since 12 June 1945 or earlier, it was erroneous for the RTC to declare the heirs she has proven that the land applied for has already been declared alienable
of Espinas as the owners of the Subject Property. and disposable; and (4) her right to due process was violated since the issues
considered by the CA were not properly raised during the trial.
The dispositive portion of the Decision of the CA reads:
We find that none of Dumo's arguments deserve any merit.
WHEREFORE, premises considered, the Appeal is PARTLY GRANTED and
the assailed Joint Decision issued by the court a quo is hereby MODIFIED in Going beyond the issues raised in the RTC and due process of law
that the Complaint for Accion Reivindicatoria (Civil Case No. 1301-Bg) filed
by plaintiffs-appellees is DISMISSED for lack of cause of action.
Dumo argues that the issue of whether the possession started on 12 June 1945
or earlier was never raised in the RTC. She also argues that no issue was
The Decision is AFFIRMED in all other respects. raised as to whether or not the land that she seeks to register is alienable and
disposable. Thus, Dumo argues that the CA erred, and also violated her right
to due process, when it considered these issues in determining whether or not
SO ORDERED.8
the application for land registration should be granted.

Dumo filed a Motion for Partial Reconsideration and subsequently, an


We do not agree.
Omnibus Motion for Entry of Judgment and to Resolve, asking the CA to
issue an entry of judgment insofar as the civil case is concerned and to declare
the land registration case submitted for resolution without any In an application for land registration, it is elementary that the applicant has
comment/opposition. The CA denied both motions in a Resolution dated 19 the burden of proving, by clear, positive and convincing evidence, that her
May 2015.9 alleged possession and occupation were of the nature and duration required by
law.11 Thus, it was upon Dumo to prove that she and her predecessors-in-
interest possessed and occupied the land sought to be registered in the nature
Hence, this petition.
and duration required by law.

The Issues
Dumo cannot validly argue that she was not afforded due process when the
CA considered to review the evidence she herself offered to support her
In this petition, Dumo seeks a reversal of the decision of the CA, and raises application for land registration. On the contrary, she was given every
the following arguments: opportunity to submit the documents to establish her right to register the land.
She simply failed to do so.
A. THE HONORABLE COURT OF APPEALS COMMITTED A
REVERSIBLE ERROR WHEN, IN DENYING THE PETITION FOR LAND When Dumo filed with the RTC the application for registration of her land,
REGISTRATION, IT WENT BEYOND THE ISSUES RAISED, THEREBY she was asking the RTC to confirm her incomplete title. The requirements for
VIOLATING OR CONTRAVENING THE RULING OF THIS judicial confirmation of imperfect title are found in Section 14 of Presidential
HONORABLE COURT IN, AMONG OTHERS, "LAM V. CHUA, 426 SCRA Decree No. 1529 (PD No. 1529), which provides:
29; DEPARTMENT OF AGRARIAN REFORM V. FRANCO, 471 SCRA 74;
BERNAS V. COURT OF APPEALS, 225 SCRA 119; PROVINCE OF
Section 14. Who may apply. The following persons may file in the proper
QUEZON V. MARTE, 368 SCRA 145 AND FIVE STAR BUS CO., INC. V.
Court of First Instance an application for registration of title to land, whether
COURT OF APPEALS, 259 SCRA 120."
personally or through their duly authorized representatives:

B. THE HONORABLE COURT OF APPEALS COMMITTED A


(1) Those who by themselves or through their predecessors-in-interest have
REVERSIBLE ERROR WHEN, IN DENYING THE PETITION FOR LAND
been in open, continuous, exclusive and notorious possession and occupation
[REGISTRATION], IT RULED THAT PETITIONER AND HER
of alienable and disposable lands of the public domain under a bona fide claim
PREDECESSORS-IN-INTEREST FAILED TO PROVE CONTINUOUS,
of ownership since June 12, 1945, or earlier.
EXCLUSIVE, AND ADVERSE POSSESSION AND OCCUPATION OF
THE SUBJECT PROPERTY IN THE CONCEPT OF [AN] OWNER FROM
JUNE 12, 1945 OR EARLIER, THEREBY VIOLATING OR (2) Those who have acquired ownership of private lands by prescription under
CONTRAVENING THE RULING OF THIS HONORABLE COURT IN the provision of existing laws.
"REPUBLIC OF THE PHILIPPINES VERSUS COURT OF APPEALS, 448
SCRA 442."
(3) Those who have acquired ownership of private lands or abandoned river
beds by right of accession or accretion under the existing laws.
C. THAT, IN ANY EVENT, AND WITHOUT PREJUDICE TO THE
FOREGOING, THE HONORABLE COURT OF AP[P]EALS COMMITTED
(4) Those who have acquired ownership of land in any other manner provided
A REVERSIBLE ERROR WHEN, IN DENYING THE PETITION FOR
for by law.
LAND REGISTRATION, IT FAILED TO CONSIDER PETITIONER'S
EXHIBIT 'A' WHICH WAS FORMALLY OFFERED TO PROVE THAT
THE SUBJECT PROPERTY WAS DISPOSIBLE [sic] AND ALIENABLE xxxx
TO WHICH THE RESPONDENT MADE NO OBJECTION[.]
Thus, it is necessary in an application for land registration that the court
D. THAT FURTHER, AND WITHOUT PREJUDICE TO THE determines whether or not an applicant fulfills the requirements under any of
FOREGOING, THE HONORABLE COURT OF APPEALS COMMITTED the paragraphs of Section 14 of PD No. 1529.
A REVERSIBLE ERROR WHEN, IN DENYING THE PETITION FOR
LAND REGISTRATION, IT FAILED TO CONSIDER THE SUPPORTING
Simply put, when Dumo filed her application for the registration of the lots
EVIDENCE THEREFOR, AGAIN, WITHOUT OBJECTION FROM THE
she claims to have inherited from her mother and bought from her siblings, the
RESPONDENT, THEREBY DEPRIVING PETITIONER OF HER
issue of whether she complied with all the requirements was the very crux of
FUNDAMENTAL RIGHT TO DUE PROCESS OF LAW.10
the application. It cannot be argued that because the Republic failed to oppose
or raise the issue in the RTC, the CA may no longer consider this issue. On
The Ruling of the Court the contrary, the classification of the land sought to be registered, and the
duration and nature of the possession and occupation have always been, and 1917. Nevertheless, in the absence of incompatibility between Section 1827 of
will always be the issues in an application for land registration. It would truly the Revised Administrative Code of 1917 and the provisions of the
be absurd for Dumo, or any other applicant for land registration, to expect the Administrative Code of 1987, we can grant that Section 1827 has not been
courts to grant the application without first determining if the requisites under repealed.21 This is in view of the repealing clause in Section 27, Final
the law have been complied with. Provisions, Book VII of the Administrative Code of 1987, which provides:

The CA had every right to look into the compliance by Dumo with the Section 27. All laws, decrees, orders, rules and regulations, or portions
requirements for the registration of the land, and we find that the CA correctly thereof, inconsistent with this Code are hereby repealed or modified
found that Dumo has acquired no registerable title to the lots she seeks to accordingly.
register.
The authority of the Department Head under Section 1827 of the Revised
Registration of land under Section 14(1) Administrative Code of 1917 is merely to classify public forest lands as public
agricultural lands. Agricultural lands of the public domain are, by themselves,
not alienable and disposable. Section 1827 of the Revised Administrative
To reiterate, under Section 14(1) of PD No. 1529, Dumo had the burden of
Code of 1917 provides:
proving the following:

Section 1827. Assignment of Forest Land for Agricultural Purposes. – Lands


(1) that the land or property forms part of the alienable and disposable in public forests, not including forest reserves, upon the certification of the
lands of the public domain; Director of Forestry that said lands are better adapted and more valuable for
agricultural than for forest purposes and not required by the public interests to
(2) that the applicant and his predecessors-in-interest have been in open, be kept under forest, shall be declared by the Department Head to be
continuous, exclusive, and notorious possession and occupation of agricultural lands. (Emphasis supplied)
the same; and

(3) that it is under a bona fide claim of ownership since 12 June 1945, or There is nothing in Section 1827 that authorizes the Department Head to
earlier.12 classify agricultural lands into alienable or disposable lands of the public
domain. The power to classify public lands as agricultural lands is separate
The first requirement is to prove that the land sought to be registered is
and distinct from the power to declare agricultural lands as alienable and
alienable and disposable land of the public domain. This is because under the
disposable. The power to alienate agricultural lands of the public domain can
Regalian Doctrine, as embodied in the 1987 Philippine Constitution, lands
never be inferred from the power to classify public lands as agricultural. Thus,
which do not clearly appear to be within private ownership are presumed to
public lands classified as agricultural and used by the Bureau of Plant Industry
belong to the State.13 Thus, in an application for land registration, the
of the Department of Agriculture for plant research or plant propagation are
applicant has the burden of overcoming the presumption that the State owns
not necessarily alienable and disposable lands of the public domain despite
the land applied for, and proving that the land has already been classified as
being classified as agricultural lands. For such agricultural lands to be
alienable and disposable.14 To overcome the presumption that the land belongs
alienable and disposable, there must be an express proclamation by the
to the State, the applicant must prove by clear and incontrovertible evidence at
President declaring such agricultural lands as alienable and disposable.
the time of application that the land has been classified as alienable and
disposable land of the public domain.
Agricultural land, the only classification of land which may be classified as
alienable and disposable under the 1987 Philippine Constitution, may still be
Classification of lands of the public domain may be found under Article XII
reserved for public or quasi-public purposes which would prohibit the
of the 1987 Philippine Constitution. More specifically, Section 3 of Article
alienation or disposition of such land. Section 8 of CA No. 141 provides:
XII classifies lands of the public domain into (1) agricultural, (2) forest or
timber, (3) mineral lands, and (4) national parks.15 Of these four
classifications, only agricultural lands may be alienated and disposed of by the Section 8. Only those lands shall be declared open to disposition or
State. concession which have been officially delimited and classified and, when
practicable, surveyed, and which have not been reserved for public or
quasi-public uses, nor appropriated by the Government, nor in any manner
The 1987 Philippine Constitution also provides that "agricultural lands of the
become private property, nor those on which a private right authorized and
public domain may be further classified by law according to the uses to
recognized by this Act or any other valid law may be claimed, or which,
which they may be devoted."16 Based on the foregoing, it is clear that the
having been reserved or appropriated, have ceased to be so. However, the
classification of lands of the public domain is first and foremost provided by
President may, for reasons of public interest, declare lands of the public
the Constitution itself. Of the classifications of lands of the public domain,
domain open to disposition before the same have had their boundaries
agricultural lands may further be classified by law, according to the uses it
established or been surveyed, or may, for the same reason, suspend their
may be devoted to.
concession or disposition until they are again declared open to concession
or disposition by proclamation duly published or by Act of the National
The classification of lands of the public domain into agricultural lands, as well Assembly. (Emphasis supplied)
as their further classification into alienable and disposable lands of the public
domain, is a legislative prerogative which may be exercised only through the
Thus, to be alienable and disposable, lands of the public domain must be
enactment of a valid law. This prerogative has long been exercised by the
expressly declared as alienable and disposable by executive or administrative
legislative department through the enactment of Commonwealth Act No. 141
proclamation pursuant to law or by an Act of Congress.
(CA No. 141) or the Public Land Act of 1936. 17 Section 6 of CA No. 141
remains to this day the existing general law governing the classification of
lands of the public domain into alienable and disposable lands of the public Even if the Department Head has the power to classify public forest lands as
domain.18 agricultural under Section 1827 of the Revised Administrative Code of 1917,
this does not include the power to classify public agricultural lands as
alienable and disposable lands of the public domain. The power
Section 182719of the Revised Administrative Code of 191720 merely
to further classify agricultural lands as alienable and disposable has not been
authorizes the Department Head to classify as agricultural lands those forest
granted in any way to the Department Head under the Revised Administrative
lands which are better adapted and more valuable for agricultural purposes.
Code of 1917. This authority was given only to the Governor-General under
Section 1827 does not authorize the Department Head to classify agricultural
Section 64 of the Revised Administrative Code of 1917, as superseded by
lands as alienable and disposable lands as this power is expressly delegated by
Section 9 of Republic Act (RA) No. 2874 (Public Land Act of 1919), and as
the same Revised Administrative Code of 1917 solely to the Governor-
in turn further superseded by Section 6 of CA No. 141 (Public Land Act of
General.
1936), which is the existing specific provision of law governing the
classification of lands of the public domain into alienable and disposable lands
The existing administrative code under the 1987 Philippine Constitution is of the public domain. This delegated power is a discretionary power, to be
Executive Order No. 292 or the Administrative Code of 1987. This existing exercised based on the sound discretion of the President.
code did not reenact Section 1827 of the Revised Administrative Code of
Under Section 64 of the Revised Administrative Code of 1917, the (b) Timber, and
classification of lands of the public domain into alienable and disposable lands (c) Mineral lands,
of the public domain could only be made by the Governor-General. While
Section 1827 of the Revised Administrative Code of 1917 gave to the
Department Head the power to classify public forest lands as public
and may at any time and in a like manner transfer such lands from one class to
agricultural lands, the very same law in its Section 64 expressly reserved to
another, for the purposes of their administration and disposition. (Emphasis
the Governor-General the power to declare for "public sale x x x any of the
supplied)
public domain of the Philippines." Section 64 of the Revised Administrative
Code of 1917 provides:
Thus, under all laws during the American regime, from the Revised
Administrative Code of 1917 up to and including CA No. 141, only the
Section 64. Particular powers and duties of Governor-General of the
Governor-General or President could classify lands of the public domain into
Philippines. – In addition to his general supervisory authority, the Governor-
alienable and disposable lands. No other government official was empowered
General of the Philippines shall have such specific powers and duties as are
by statutory law during the American regime. Under the 1935, 22 197323 and
expressly conferred or imposed on him by law and also, in particular, the
198724 Philippine Constitutions, the power to declare or classify lands of the
powers and duties set forth in this chapter.
public domain as alienable and disposable lands belonged to Congress. This
legislative power is still delegated to the President under Section 6 of CA No.
Among such special powers and duties shall be: 141 since this Section 6 was never repealed by Congress despite successive
amendments to CA No. 141 after the adoption of the 1935, 1973 and the 1987
Philippine Constitutions.25
(a) x x x

Under Section 13 of PD No. 705, otherwise known as the Revised Forestry


xxxx
Code of the Philippines, the Department of Environment and Natural
Resources (DENR) Secretary has been delegated by law the discretionary
(d) To reserve from settlement or public sale and for specific public uses power to classify as alienable and disposable forest lands of the public domain
any of the public domain of the (Philippine Islands) Philippines the use of no longer needed for forest reserves. Section 13 of the Revised Forestry Code
which is not otherwise directed by law, the same thereafter remaining of the Philippines, which was enacted on 19 May 1975, provides:
subject to the specific public uses indicated in the executive order by
which such reservation is made, until otherwise provided by law or
Section 13. System of Land Classification.– The Department Head shall
executive order.
study, devise, determine and prescribe the criteria, guidelines and methods for
the proper and accurate classification and survey of all lands of the public
(e) To reserve from sale or other disposition and for specific public uses or domain into agricultural, industrial or commercial, residential, resettlement,
service, any land belonging to the private domain of the Government of the mineral, timber or forest, and grazing lands, and into such other classes as
(Philippine Islands) Philippines, the use of which is not otherwise directed by now or may hereafter be provided by law, rules and regulations.
law; and thereafter such land shall not be subject to sale or other disposition
and shall be used for the specific purposes directed by such executive order
In the meantime, the Department Head shall simplify through inter-bureau
until otherwise provided by law.
action the present system of determining which of the unclassified lands of the
public domain are needed for forest purposes and declare them as permanent
x x x x (Emphasis supplied) forest to form part of the forest reserves. He shall declare those classified
and determined not to be needed for forest purposes as alienable and
disposable lands, the administrative jurisdiction and management of which
Likewise, under Section 9 of RA No. 2874, the classification of lands of shall be transferred to the Bureau of Lands: Provided, That mangrove and
public domain into alienable and disposable lands could only be made by the other swamps not needed for shore protection and suitable for fishpond
Governor-General, thus: purposes shall be released to, and be placed under the administrative
jurisdiction and management of, the Bureau of Fisheries and Aquatic
Section 9. For the purposes of their government and disposition, the lands of Resources. Those still to be classified under the present system shall continue
the public domain alienable or open to disposition shall be classified, to remain as part of the public forest. (Emphasis supplied)
according to the use or purposes to which such lands are destined, as follows:
Section 3, Article XII of the 1987 Philippine Constitution states: "x x x.
(a) Agricultural Alienable lands of the public domain shall be limited to agricultural lands. x x
x." Thus, the unclassified lands of the public domain, not needed for forest
reserve purposes, must first be declared agricultural lands of the public
(b) Commercial, industrial, or for similar productive purposes. domain before the DENR Secretary can declare them alienable and
disposable. Under the foregoing Section 13 of PD No. 705, the DENR
(c) Educational, charitable, and other similar purposes. Secretary has no discretionary power to classify unclassified lands of the
public domain, not needed for forest reserve purposes, into agricultural lands.
However, the DENR Secretary can invoke his power under Section 1827 of
(d) Reservations for town sites, and for public and quasi-public uses. the Revised Administrative Code of 1917 to classify forest lands into
agricultural lands. Once so declared as agricultural lands of the public domain,
The Governor-General, upon recommendation by the Secretary of the DENR Secretary can then invoke his delegated power under Section 13 of
Agriculture and Natural Resources, shall from time to time make the PD No. 705 to declare such agricultural lands as alienable and disposable
classification provided for in this section, and may, at any time and in a lands of the public domain.
similar manner, transfer lands from one class to another. (Emphasis
supplied) This Court has recognized in numerous cases the authority of the DENR
Secretary to classify agricultural lands of the public domain as alienable and
Similarly, under Section 6 of CA No. 141, the existing law on the disposable lands of the public domain.26 As we declared in Republic of the
matter, only the President can classify lands of the public domain into Philippines v. Heirs of Fabio,27 "the DENR Secretary is the only other public
alienable or disposable lands, thus: official empowered by law to approve a land classification and declare such
land as alienable and disposable."
Section 6. The President, upon the recommendation of the Secretary of
Agriculture and Commerce, shall from time to time classify the lands of the Consequently, as the President's and the DENR Secretary's discretionary
public domain into — power to classify land as alienable and disposable is merely delegated to them
under CA No. 141 and PD No. 705, respectively, they may not redelegate the
same to another office or officer. What has once been delegated by Congress
(a) Alienable or disposable, can no longer be further delegated or redelegated by the original delegate to
another, as expressed in the Latin maxim — Delegata potestas non potest Secretary may no longer delegate the power to issue such certification as the
delegari.28 Thus, in Aquino-Sarmiento v. Morato,29 this Court ruled: power to classify lands of the public domain as alienable and disposable lands
is in itself a delegated power under CA No. 141 and PD No. 705.
The power to classify motion pictures into categories such as "General
Patronage" or "For Adults Only" is vested with the respondent Board itself Moreover, we have repeatedly stated that a CENRO or PENRO certification is
and not with the Chairman thereof (Sec. 3 [e], PD 1986). As Chief Executive not enough to prove the alienable and disposable nature of the property sought
Officer, respondent Morato's function as Chairman of the Board calls for the to be registered because the only way to prove the classification of the land is
implementation and execution, not modification or reversal, of the decisions through the original classification approved by the DENR Secretary or the
or orders of the latter (Sec. 5 [a], Ibid.). The power of classification having President himself. This Court has clearly held:
been reposed by law exclusively with the respondent Board, it has no
choice but to exercise the same as mandated by law, i.e., as a collegial
Further, it is not enough for the PENRO or CENRO to certify that a land is
body, and not transfer it elsewhere or discharge said power through the
alienable and disposable. The applicant for land registration must prove
intervening mind of another. Delegata potestas non potest delegari —a
that the DENR Secretary had approved the land classification and
delegated power cannot be delegated. And since the act of classification
released the land of the public domain as alienable and disposable, and
involves an exercise of the Board's discretionary power with more reason
that the land subject of the application for registration falls within the
the Board cannot, by way of the assailed resolution, delegate said power
approved area per verification through survey by the PENRO or CENRO. In
for it is an established rule in administrative law that discretionary
addition, the applicant for land registration must present a copy of the original
authority cannot be a subject of delegation. (Emphasis supplied)
classification approved by the DENR Secretary and certified as a true copy by
the legal custodian of the official records. These facts must be established to
Under the 1987 Philippine Constitution, the power to classify agricultural prove that the land is alienable and disposable. Respondent failed to do so
lands of the public domain into alienable and disposable lands of the public because the certifications presented by respondent do not, by themselves,
domain is exercised "by law" or through legislative enactment. In accordance prove that the land is alienable and disposable.34 (Emphasis supplied)
with Section 6 of CA No. 141, this power is delegated to the President who
may, based on his sound discretion, classify agricultural lands as alienable and
A CENRO or PENRO certification is insufficient to prove the alienable and
disposable lands of the public domain. This delegated power to so classify
disposable nature of the land sought to be registered it is the original
public agricultural lands may no longer be redelegated by the President – what
classification by the DENR Secretary or the President which is essential to
has once been delegated may no longer be delegated to another. Likewise, the
prove that the land is indeed alienable and disposable. This has been
same discretionary power has been delegated "by law" to the DENR Secretary
consistently upheld by this Court in subsequent land registration cases.
who, of course, cannot redelegate the same to his subordinates.
Recently, in Republic of the Philippines v. Nicolas,35 which cited Republic of
the Philippines v. Lualhati,36 the Court rejected the attempt of the applicant to
As it is only the President or the DENR Secretary who may classify as prove the alienable and disposable character of the land through PENRO or
alienable and disposable the lands of the public domain, an applicant for land CENRO certifications. The Court held:
registration must prove that the land sought to be registered has been declared
by the President or DENR Secretary as alienable and disposable land of the
[N]one of the documents submitted by respondent to the trial court indicated
public domain. To establish such character, jurisprudence has been clear on
that the subject property was agricultural or part of the alienable and
what an applicant must submit to clearly establish that the land forms part of
disposable lands of the public domain. At most, the CENRO Report and
the alienable and disposable lands of the public domain.
Certification stated that the land was not covered by any kind of public land
application. This was far from an adequate proof of the classification of the
In Republic of the Philippines v. T.A.N. Properties, Inc.,30 this Court has held land. In fact, in Republic v. Lualhati, the Court rejected an attempt to prove
that an applicant must present a copy of the original classification approved by the alienability of public land using similar evidence:
the DENR Secretary and certified as a true copy by the legal custodian of the
official records. Additionally, a certificate of land classification status issued
Here, respondent failed to establish, by the required evidence, that the land
by the Community Environment and Natural Resources Office (CENRO) or
sought to be registered has been classified as alienable or disposable land of
the Provincial Environment and Natural Resources Office (PENRO) of the
the public domain. The records of this case merely bear certifications from the
DENR and approved by the DENR Secretary must also be presented to prove
DENR-CENRO, Region IV, Antipolo City, stating that no public land
that the land subject of the application for registration is alienable and
application or land patent covering the subject lots is pending nor are the lots
disposable) and that it falls within the approved area per verification through
embraced by any administrative title. Said CENRO certifications, however, do
survey by the PENRO or CENRO.31 In Republic of the Philippines v.
not even make any pronouncement as to the alienable character of the lands in
Roche,32 we clearly stated:
question for they merely recognize the absence of any pending land patent
application, administrative title, or government project being conducted
[T]he applicant bears the burden of proving the status of the land. In this thereon. But even granting that they expressly declare that the subject
connection, the Court has held that he must present a certificate of land lands form part of the alienable and disposable lands of the public
classification status issued by the Community Environment and Natural domain, these certifications remain insufficient for purposes of granting
Resources Office (CENRO) or the Provincial Environment and Natural respondent's application for registration. As constantly held by this
Resources Office (PENRO) of the DENR. He must also prove that the Court, it is not enough for the CENRO to certify that a land is alienable
DENR Secretary had approved the land classification and released the and disposable. The applicant for land registration must prove that the
land as alienable and disposable, and that it is within the approved area DENR Secretary had approved the land classification and released the
per verification through survey by the CENRO or PENRO. Further, the land of the public domain as alienable and disposable, and that the land
applicant must present a copy of the original classification approved by the subject of the application for registration falls within the approved area
DENR Secretary and certified as true copy by the legal custodian of the per verification through survey by the PENRO or CENRO. Unfortunately
official records. These facts must be established by the applicant to prove that for respondent, the evidence submitted clearly falls short of the requirements
the land is alienable and disposable.33 (Emphasis supplied) for original registration in order to show the alienable character of the lands
subject herein. (Emphasis supplied)
To repeat, there are two (2) documents which must be presented: first, a copy
of the original classification approved by the Secretary of the DENR and In this case, Dumo failed to submit any of the documents required to prove
certified as a true copy by the legal custodian of the official records, that the land she seeks to register is alienable and disposable land of the public
and second, a certificate of land classification status issued by the CENRO or domain.
the PENRO based on the land classification approved by the DENR Secretary.
The requirement set by this Court in Republic of the Philippines v. T.A.N
Response to the Concurring and Dissenting Opinion of Justice Caguioa
Properties, Inc. that both these documents be based on the land classification
approved by the DENR Secretary is not a mere superfluity. This requirement
stems from the fact that the alienable and disposable classification of The Concurring and Dissenting Opinion of Justice Caguioa suggests that
agricultural land may be made by the President or DENR Secretary. And certifications of land classification status issued by the CENRO and PENRO
while the DENR Secretary may perform this act in the regular course of should be deemed sufficient to prove the alienable and disposable character of
business, this does not extend to the CENRO or PENRO – the DENR the property if these certifications bear references to the land classification
maps and the original classification issued and signed by the DENR Secretary. (c) Public records, kept in the Philippines, of private documents required by
This suggestion clearly undermines the requirements set by this Court law to be entered therein.
in Republic of the Philippines v. T.A.N. Properties, Inc.37 where the Court
expressly stated that it is not enough for the CENRO or PENRO to certify that
In turn, for the record of public documents referred to in paragraph (a) of
the land sought to be registered is alienable and disposable. What is required
Section 19, Rule 132 to be admissible, it must be evidenced by an official
from the applicant in a land registration proceeding is to prove that the DENR
publication thereof or by a copy attested by the officer having the legal
Secretary had approved the land classification and released the land of the
custody of the record, or by his deputy.46Moreover, to be prima
public domain as alienable and disposable, and that the land subject of the
facie evidence of the facts stated in public documents, such documents
application for registration falls within the approved area per verification
must consist of entries in public records made in the performance of a
through survey by the PENRO or CENRO. Quite clearly, the Court
duty by a public officer.47 This requirement can be satisfied only if a certified
definitively stated that to prove that the land is alienable and disposable, the
true copy of the proclamation by the President or the order of the DENR
applicant must present a certified true copy of the original classification
Secretary classifying the land as alienable and disposable is presented to the
approved by the DENR Secretary or the proclamation made by the President.
trial court.
Only the certified true copy of the original classification approved by the
DENR Secretary or the President will prove to the courts that indeed, the land
sought to be registered is alienable and disposable. Quite clearly, certifications by the CENRO or PENRO do not comply with the
conditions for admissibility of evidence. The CENRO or the PENRO is not
the official repository or legal custodian of the issuances of the President or
That the certifications of the CENRO or PENRO contain references to the
DENR Secretary classifying lands as alienable and disposable lands of the
original classification approved by the DENR Secretary is not enough to
public domain. Thus, the certifications made by the CENRO or PENRO
prove that the land is alienable and disposable. Mere references made in the
cannot prove the alienable and disposable character of the land, which can
certifications to the classification of land as approved by the DENR Secretary
only be ascertained through the classification made by the President or DENR
are simply insufficient. The trial court must be given a certified true copy of
Secretary, the only public officials who may classify lands into alienable and
the classification made by the DENR Secretary or the President because it is
disposable lands of the public domain. The Concurring and Dissenting
the only acceptable and sufficient proof of the alienable and disposable
Opinion alleges that the CENRO serves as a repository of the land
character of the land. In Republic of the Philippines v. T.A.N. Properties,
classification maps, and as such, authorizes the CENRO to issue certified true
Inc.,38the Court required the submission of the certified true copy of the
copies of the approved land classification maps. While the CENRO may issue
land classification approved by the DENR Secretary precisely because
certified true copies of these land classification maps, these maps are not the
mere references made by the CENRO and PENRO to the land
required certified true copy of the original proclamation or order classifying
classification were deemed insufficient. For instance, CENRO and PENRO
the public land as alienable and disposable. Moreover, these maps are not in
may inadvertently make references to an original classification approved by
the possession of the officials who have custody of the original proclamation
the DENR Secretary which does not cover the land sought to be registered, or
or order classifying the public land as alienable and disposable. Again, the
worse, to a non-existent original classification. This is the very evil that the
best evidence of the alienable and disposable nature of the land is the certified
ruling in Republic of the Philippines v. T.A.N. Properties, Inc.39 seeks to
true copy of the classification made by the President or the DENR Secretary –
avoid. Justice Caguioa's suggestion resurrects the very evil banished by this
not the certified true copy issued by the CENRO of its land classification
Court in Republic of the Philippines v. T.A.N Properties, Inc. 40
maps.

Decisions of this Court form part of the legal system of the


It is also worthy to note that in Republic of the Philippines v. T.A.N.
Philippines41 and thus the CENRO, PENRO, and the DENR must follow the
Properties, Inc.,48 we have already discussed the value of certifications issued
decision made by this Court in Republic of the Philippines v. T.A.N
by the CENRO or PENRO in land registration cases:
Properties, Inc.42The ruling of this Court requiring the submission of the
certified true copy of the original classification as approved by the DENR
Secretary cannot be overturned or amended by the CENRO or PENRO The CENRO and Regional Technical Director, FMS-DENR, certifications do
or even by the DENR. The DENR, CENRO, and PENRO must follow the not fall within the class of public documents contemplated in the first sentence
law as laid down by this Court in Republic of the Philippines v. T.A.N. of Section 23 of Rule 132. The certifications do not reflect "entries in public
Properties, Inc.43 It is not this Court that should amend its ruling in Republic records made in the performance of a duty by a public officer", such as entries
of the Philippines v. T.A.N Properties, Inc.44 to conform to the administrative made by the Civil Registrar in the books of registries, or by a ship captain in
rules of the DENR, CENRO, or PENRO reversing the final ruling of this the ship's logbook. The certifications are not the certified copies or
Court in Republic of the Philippines v. T.A.N. Properties, Inc.45 The authority authenticated reproductions of original official records in the legal
given by the Administrative Order of the DENR to the CENRO and PENRO custody of a government office. The certifications are not even records of
to issue certifications of land classification status does not and cannot reverse public documents. The certifications are conclusions unsupported by
the clear requirement laid down by the Court for applicants of land adequate proof, and thus have no probative value. Certainly, the
registration to submit the certified true copy of the original classification certifications cannot be considered prima facie evidence of the facts stated
approved by the DENR Secretary to prove the alienable and disposable therein.
character of the land.
The CENRO and Regional Technical Director, FMS-DENR, certifications do
To repeat, in a judicial confirmation of imperfect title under Section 14(1) of not prove that Lot 10705-B falls within the alienable and disposable land as
PD No. 1529, the applicant has the burden of proving that the land sought to proclaimed by the DENR Secretary. Such government certifications do not, by
be registered is alienable and disposable land of the public domain. In turn, their mere issuance, prove the facts stated therein. Such government
the best evidence of the alienable and disposable nature of the land is the certifications may fall under the class of documents contemplated in the
certified true copy of the original proclamation made by the President or second sentence of Section 23 of Rule 132. As such, the certifications
DENR Secretary, in accordance with CA No. 141 or PD No. 705. Submitting are prima facie evidence of their due execution and date of issuance but they
a mere certification by the CENRO or PENRO with references to the original do not constitute prima facie evidence of the facts stated therein.49 (Emphasis
classification made by the President or the DENR Secretary is sorely supplied)
inadequate since it has no probative value as a public document to prove the
alienable and disposable character of the public land.
The certification issued by the CENRO or PENRO, by itself, does not prove
the alienable and disposable character of the land sought to be registered. The
Under Section 19, Rule 132 of the Rules of Court, public documents are: certification should always be accompanied by the original or certified true
copy of the original classification approved by the DENR Secretary or the
President.
(a) The written official acts, or records of the official acts of the sovereign
authority, official bodies and tribunals, and public officers, whether of the
Philippines, or of a foreign country; Substantial Compliance with the Requirements of Section 14(1)

(b) Documents acknowledged before a notary public except last wills and Dumo argues that the Certification from the Regional Surveys Division,
testaments; and which was formally offered as Exhibit "A" and not opposed by the Republic,
should be considered substantial compliance with the requirement that the
applicant must submit the certified true copy of the original classification of need not be from 12 June 1945 or earlier, and that as long as such land is
the land as approved by the DENR Secretary. classified as alienable and disposable when the application is filed, then the
first requirement under the law is fulfilled. The Court held:
We do not agree.
Petitioner suggests an interpretation that the alienable and disposable
character of the land should have already been established since June 12, 1945
The fact that the Republic did not oppose the formal offer of evidence of
or earlier. This is not borne out by the plain meaning of Section 14(1). "Since
Dumo in the RTC does not have the effect of proving or impliedly admitting
June 12, 1945," as used in the provision, qualifies its antecedent phrase "under
that the land is alienable and disposable. The alienable and disposable
a bona fide claim of ownership." Generally speaking, qualifying words restrict
character of the land must be proven by clear and incontrovertible evidence. It
or modify only the words or phrases to which they are immediately
may not be impliedly admitted, as Dumo vehemently argues. It was the duty
associated, and not those distantly or remotely located. Ad proximum
of Dumo to prove that the land she sought to register is alienable and
antecedents fiat relation nisi impediatur sentencia.
disposable land of the public domain. This burden would have been
discharged by submitting the required documents – a copy of the original
classification approved by the DENR Secretary and certified as a true copy by Besides, we are mindful of the absurdity that would result if we adopt
the legal custodian thereof, and a certificate of land classification status issued petitioner's position. Absent a legislative amendment, the rule would be,
by the CENRO or the PENRO based on the approved land classification by adopting the OSG's view, that all lands of the public domain which were not
the DENR Secretary. Without these, the applicant simply fails to prove that declared alienable or disposable before June 12, 1945 would not be
the land sought to be registered forms part of the alienable and disposable susceptible to original registration, no matter the length of unchallenged
lands of the public domain and thus, it may not be susceptible to private possession by the occupant. Such interpretation renders paragraph (1) of
ownership. As correctly pointed out by the CA, the land is presumed to belong Section 14 virtually inoperative and even precludes the government from
to the State as part of the public domain. giving it effect even as it decides to reclassify public agricultural lands as
alienable and disposable. The unreasonableness of the situation would even be
aggravated considering that before June 12, 1945, the Philippines was not yet
Another requirement under Section 14(1) of PD No. 1529 is to prove that the
even considered an independent state.
applicant and her predecessors-in-interest have been in open, continuous,
exclusive, and notorious possession and occupation of the land under
a bonafide claim of ownership since 12 June 1945 or earlier. Instead, the more reasonable interpretation of Section 14(1) is that it merely
requires the property sought to be registered as already alienable and
disposable at the time the application for registration of title is filed. If the
In this case, the CA found that Dumo and her predecessors-in-interest have
State, at the time the application is made, has not yet deemed it proper to
been in possession of the land only from 1948, which is the earliest date of the
release the property for alienation or disposition, the presumption is that the
tax declaration presented by Dumo. This fact is expressly admitted by Dumo.
government is still reserving the right to utilize the property; hence, the need
Thus, from this admission alone, it is clear that she failed to prove her and her
to preserve its ownership in the State irrespective of the length of adverse
predecessors-in-interest's possession and occupation of the land for the
possession even if in good faith. However, if the property has already been
duration required by law — from 12 June 1945 or earlier.
classified as alienable and disposable, as it is in this case, then there is already
an intention on the part of the State to abdicate its exclusive prerogative over
Dumo, however, argues that it does not matter that her possession dates only the property.51
back to 1948 because this Court has allegedly stated that even if the
possession or occupation started after 12 June 1945, this does not bar the grant
Thus, it did not state that the possession and occupation from 12 June 1945 or
of an application for registration of land.
earlier are no longer required. It merely clarified when the land should have
been classified as alienable and disposable to meet the requirements of
Again, we do not agree with Dumo. Section 14(1) of PD No. 1529. The property sought to be registered must be
declared alienable and disposable at the time of the filing of the application
for registration.52 This does not require that the land be declared alienable and
To determine whether possession or occupation from 12 June 1945 or earlier disposable from 12 June 1945 or earlier.
is material, one has to distinguish if the application for the registration of land
is being made under paragraph 1 or paragraph 2 of Section 14 of PD No.
1529. The relevant paragraphs provide: Registration of land under Section 14(2)

Section 14. Who may apply. The following persons may file in the proper Dumo also argues that she has the right to register the land because she and
Court of First Instance an application for registration of title to land, whether her predecessors-in-interest have already acquired the land through
personally or through their duly authorized representatives: prescription. She states that she and her predecessors-in-interest have been in
possession and occupation of the land for fifty-six (56) years, and thus she has
already acquired ownership of the land by prescription.
(1) Those who by themselves or through their predecessors-in-interest have
been in open, continuous, exclusive and notorious possession and occupation
of alienable and disposable lands of the public domain under a bona fide claim Again, we disagree.
of ownership since June 12, 1945, or earlier.
It is true that under Section 14 of PD No. 1529, one may acquire ownership of
(2) Those who have acquired ownership of private lands by prescription under the land by prescription. Particularly, paragraph 2 of Section 14 provides that
the provision of existing laws. "those who have acquired ownership of private lands by prescription under the
provision of existing laws" may file an application for registration of title to
land. The existing law mentioned in PD No. 1529 is the Civil Code of the
xxxx Philippines. In Heirs of Malabanan v. Republic of the Philippines,53 we
applied the civil law concept of prescription as embodied in the Civil Code to
Thus, it is clear that if the applicant is applying for the registration of land interpret Section 14(2) of PD No. 1529. This Court held:
under paragraph 1, possession and occupation of the alienable and disposable
land of the public domain under a bona fide claim of ownership should have The second source is Section 14(2) of P.D. 1529 itself, at least by implication,
commenced from 12 June 1945 or earlier. If, however, the applicant is relying as it applies the rules on prescription under the Civil Code, particularly
on the second paragraph of Section 14 to register the land, then it is true that a Article 1113 in relation to Article 1137. Note that there are two kinds of
different set of requirements applies, and possession and occupation from 12 prescription under the Civil Code – ordinary acquisitive prescription and
June 1945 or earlier are not required. extraordinary acquisitive prescription, which, under Article 1137, is
completed "through uninterrupted adverse possession... for thirty years,
The reliance of Dumo on Republic of the Philippines v. Court of Appeals50 is without need of title or of good faith."54 (Boldfacing and underscoring
misplaced. The pronouncement of the Court in relation to the phrase "June 12, supplied)
1945 or earlier" was that the alienable and disposable classification of the land
Section 14(2) of PD No. 1529 puts into operation the entire regime of Alienable and disposable lands of the public domain are those that are to be
prescription under the Civil Code, particularly Article 1113 in relation to disposed of to private individuals by sale or application, because their
Article 1137.55 Article 1113 provides that "[p]roperty of the State or any of its disposition to private individuals is for the development of the national
subdivisions not patrimonial in character shall not be the object of wealth. Thus, homesteads, which are granted to individuals from alienable and
prescription." Thus, it is clear that the land must be patrimonial before it may disposable lands of the public domain, are for the development of agriculture
be susceptible of acquisitive prescription. Indeed, Section 14(2) of PD No. which would redound to the development of national wealth. However, until
1529 provides that one may acquire ownership of private lands by the lands are alienated or disposed of to private individuals, they remain
prescription. "alienable lands of the public domain," as expressly classified by the 1987
Philippine Constitution.
Land of the public domain is converted into patrimonial property when there
is an express declaration by the State that the public dominion property is no Lands of the public domain become patrimonial property only when they are
longer intended for public service or the development of the national no longer intended for public use or public service or the development of
wealth.56 Without such declaration, acquisitive prescription does not start to national wealth. Articles 421 and 422 of the Civil Code expressly provide:
run, even if such land is alienable and disposable and the applicant is in
possession and occupation thereof. We have held:
Article 421. All other property of the State, which is not of the character stated
in the preceding article, is patrimonial property
Accordingly, there must be an express declaration by the State that the public
dominion property is no longer intended for public service or the development
Article 422. Property of public dominion, when no longer intended for public
of the national wealth or that the property has been converted into patrimonial.
use or for public service, shall form part of the patrimonial property of the
Without such express declaration, the property, even if classified as alienable
State.
or disposable, remains property of the public dominion, pursuant to Article
420(2), and thus incapable of acquisition by prescription. It is only when such
alienable and disposable lands are expressly declared by the State to be no In turn, the intention that the property is no longer needed for public use,
longer intended for public service or for the development of the national public service or the development of national wealth may only be ascertained
wealth that the period of acquisitive prescription can begin to run. Such through an express declaration by the State. We have clearly held:
declaration shall be in the form of a law duly enacted by Congress or a
Presidential Proclamation in cases where the President is duly authorized by
Accordingly, there must be an express declaration by the State that the public
law.57
dominion property is no longer intended for public service or the development
of the national wealth or that the property has been converted into
Mere classification of agricultural land as alienable and disposable does not patrimonial. Without such express declaration, the property, even if
make such land patrimonial property of the State – an express declaration by classified as alienable or disposable, remains property of the public
the State that such land is no longer intended for public use, public service or dominion, pursuant to Article 420(2), and thus incapable of acquisition
the development of national wealth is imperative. This is because even with by prescription. It is only when such alienable and disposable lands are
such classification, the land remains to be part of the lands of the public expressly declared by the State to be no longer intended for public service or
domain. In Navy Officers' Village Association, Inc. v. Republic of the for the development of the national wealth that the period of acquisitive
Philippines,58 we stated: prescription can begin to run. Such declaration shall be in the form of a law
duly enacted by Congress or a Presidential Proclamation in cases where the
President is duly authorized by law.59 (Emphasis supplied)
Lands of the public domain classified as reservations for public or quasi-
public uses are non-alienable and shall not be subject to disposition,
although they are, by the general classification under Section 6 of C.A. Without an express declaration that the land is no longer needed for public
No. 141, alienable and disposable lands of the public domain, until use, public service or the development of national wealth, it should be
declared open for disposition by proclamation of the President. (Emphasis presumed that the lands of the public domain, whether alienable and
supplied) disposable or not, remain belonging to the State under the Regalian Doctrine.
We have already recognized that the classification of land as alienable and
disposable does not make such property patrimonial. In Dream Village
Under CA No. 141, the power given to the President to classify lands as
Neighborhood Association, Inc. v. Bases Conversion Development
alienable and disposable extends only to  lands of the public domain. Lands
Authority,60 the Court held:
of the public domain are public lands intended for public use, or without being
for public use, are intended for some public service or for the development of
national wealth. Lands of the public domain, like alienable or disposable lands One question laid before us is whether the area occupied by Dream Village is
of the public domain, are not private lands. Article 420 of the Civil Code susceptible of acquisition by prescription. In Heirs of Mario Malabanan v.
provides: Republic, it was pointed out that from the moment R.A. No. 7227 was
enacted, the subject military lands in Metro Manila became alienable and
disposable. However, it was also clarified that the said lands did not thereby
Art. 420. The following things are property of public dominion:
become patrimonial, since the BCDA law makes the express reservation that
they are to be sold in order to raise funds for the conversion of the former
(1) Those intended for public use, such as roads, canals, rivers, torrents, ports American bases in Clark and Subic. The Court noted that the purpose of the
and bridges constructed by the State, banks, shores, roadsteads, and others of law can be tied to either "public service" or "the development of national
similar character; wealth" under Article 420(2) of the Civil Code, such that the lands remain
property of the public dominion, albeit their status is now alienable and
disposable. The Court then explained that it is only upon their sale to a
(2) Those which belong to the State, without being for public use, and are
private person or entity as authorized by the BCDA law that they become
intended for some public service or for the development of the national
private property and cease to be property of the public dominion:
wealth.

For as long as the property belongs to the State, although already classified


Classifying lands as alienable and disposable does not take away from the fact
as alienable or disposable, it remains property of the public dominion if x
that these lands still belong to the public domain. These lands belonged to the
x x it is "intended for some public service or for the development of the
public domain before they were classified as alienable and disposable and
national wealth."
they still remain to be lands of the public domain after such classification. In
fact, these lands are classified in Section 3, Article XII of the 1987
Philippine Constitution as "[a]lienable lands of the public domain." The Thus, under Article 422 of the Civil Code, public domain lands become
alienable and disposable character of the land merely gives the State the patrimonial property only if there is a declaration that these are alienable or
authority to alienate and dispose of such land if it deems that the land is no disposable, together with an express government manifestation that the
longer needed for public use, public service or the development of national property is already patrimonial or no longer retained for public service or the
wealth. development of national wealth. x x x. (Emphasis supplied)
The alienable and disposable character of public agricultural land does not works in relation to Section 48(b) of CA No. 141 in the registration of
convert the land to patrimonial property. It merely gives the State the alienable and disposable lands of the public domain:
authority to alienate or dispose the agricultural land, in accordance with law. It
is only when (1) there is an express government manifestation that the land is
It is clear that Section 48 of the Public Land Act is more descriptive of the
already patrimonial or no longer intended for public use, public service or the
nature of the right enjoyed by the possessor than Section 14 of the Property
development of national wealth, or (2) land which has been classified as
Registration Decree, which seems to presume the pre-existence of the right,
alienable and disposable land is actually alienated and disposed of by the
rather than establishing the right itself for the first time. It is proper to assert
State, that such land becomes patrimonial.
that it is the Public Land Act, as amended by P.D. No. 1073 effective 25
January 1977, that has primarily established the right of a Filipino citizen who
In the present case, Dumo not only failed to prove that the land sought to be has been in "open, continuous, exclusive, and notorious possession and
registered is alienable and disposable, but also utterly failed to submit any occupation of alienable and disposable lands of the public domain, under
evidence to establish that such land has been converted into patrimonial a bona fide claim of acquisition of ownership, since June 12, 1945" to perfect
property by an express declaration by the State. To repeat, acquisitive or complete his title by applying with the proper court for the confirmation of
prescription only applies to private lands as expressly provided in Article his ownership claim and the issuance of the corresponding certificate of title.
1113 of the Civil Code. To register land acquired by prescription under PD
No. 1529 (in relation to the Civil Code of the Philippines), the applicant must
Section 48 can be viewed in conjunction with the afore-quoted Section 11 of
prove that the land is not merely alienable and disposable, but that it has also
the Public Land Act, which provides that public lands suitable for agricultural
been converted into patrimonial property of the State. Prescription will start to
purposes may be disposed of by confirmation of imperfect or incomplete
run only from the time the land has become patrimonial.61 Unless the alienable
titles, and given the notion that both provisions declare that it is indeed the
and disposable land of the public domain is expressly converted into
Public Land Act that primarily establishes the substantive ownership of the
patrimonial property, there is no way for acquisitive prescription to set in
possessor who has been in possession of the property since 12 June 1945. In
under Article 1113 of the Civil Code.
turn, Section 14(a) of the Property Registration Decree recognizes the
substantive right granted under Section 48(b) of the Public Land Act, as
However, another mode of prescription specifically governs the acquisitive well as provides the corresponding original registration procedure for the
prescription of alienable and disposable lands of the public domain. CA judicial confirmation of an imperfect or incomplete title. 62 (Emphasis
No. 141 provides for the modes of disposing alienable and disposable supplied)
agricultural lands of the public domain:
Thus, the applicant for registration of the alienable and disposable land of the
Section 11. Public lands suitable for agricultural purposes can be disposed of public domain claims his right to register the land under Section 48(b) of CA
only as follows, and not otherwise: No. 141 and the procedure for registration is found under Section 14(1) of PD
No. 1529 which provides that "those who by themselves or through their
predecessors-in-interest have been in open, continuous, exclusive and
(1) For homestead settlement; notorious possession and occupation of alienable and disposable lands of
the public domain under a bona fide claim of ownership since June 12, 1945,
(2) By sale; or earlier" may file in the proper court their application for land registration.
The basis for application of judicial confirmation of title over alienable and
(3) By lease; and disposable land of the public domain is not acquisitive prescription under the
(4) By confirmation of imperfect or incomplete titles: Civil Code, but rather, the fulfillment of the requirements under Section 48(b)
of CA No. 141.
(a) By judicial legalization; or
To summarize the discussion and reiterate the guidelines set by this Court
(b) By administrative legalization (free patent). (Emphasis
in Heirs of Malabanan v. Republic of the Philippines,63 we state:
supplied)
In turn, Section 48 of the same law provides for those who may apply for
confirmation of their imperfect or incomplete title by judicial application: 1. If the applicant or his predecessors-in-interest have been in open,
continuous, exclusive and notorious possession and occupation of the land
sought to be registered under a bona fide claim of ownership since 12 June
Section 48. The following-described citizens of the Philippines, 1945 or earlier, the applicant must prove that the land has been classified by
occupying lands of the public domain or claiming to own any such lands or the Executive department as alienable and disposable land of the public
an interest therein, but whose titles have not been perfected or completed, may domain. This is covered by Section 14(1) of PD No. 1529 in relation to
apply to the Court of First Instance of the province where the land is located Section 48(b) of CA No. 141.
for confirmation of their claims and the issuance of a certificate of title
therefor, under the Land Registration Act, to wit:
While it is not necessary that the land has been alienable and disposable since
12 June 1945 or earlier, the applicant must prove that the President or DENR
xxxx Secretary has classified the land as alienable and disposable land of the public
domain at any time before the application was made.
(b) Those who by themselves or through their predecessors-in-interest have
been in open, continuous, exclusive, and notorious possession and occupation 2. If the occupation and possession of the land commenced at any time after
of alienable and disposable lands of the public domain, under a bona fide 12 June 1945, the applicant may still register the land if he or his
claim of acquisition of ownership, since June 12, 1945, or earlier, immediately predecessors-in-interest have complied with the requirements of acquisitive
preceding the filing of the applications for confirmation of title, except when prescription under the Civil Code after the land has been expressly declared
prevented by war or force majeure. These shall be conclusively presumed to as patrimonial property or no longer needed for public use, public service or
have performed all the conditions essential to a Government grant and shall be the development of national wealth. This is governed by Section 14(2) of PD
entitled to a certificate of title under the provisions of this chapter. (Emphasis No. 1529 in relation to the Civil Code.
supplied)
Under the Civil Code, acquisitive prescription, whether ordinary or
It is clear from the foregoing provisions that for lands of the public domain, extraordinary, applies only to private property. Thus, the applicant must
one may apply for an administrative grant from the government, through prove when the land sought to be registered was expressly declared as
homestead, sale, lease or free patent, or apply for the confirmation of their title patrimonial property because it is only from this time that the period for
in accordance with the conditions provided under Section 48(b) of CA No. acquisitive prescription would start to run.
141. PD No. 1529 provides for the original registration procedure for the
judicial confirmation of an imperfect or incomplete title. It must also be noted
that the wording in Section 48(b) of CA No. 141 is similar to that found in Based on the foregoing, we find that the CA committed no reversible error in
Section 14(1) of PD No. 1529. The similarity in wording has already been finding that Dumo had no registerable title over the land she seeks to register.
explained by this Court when it recognized that Section 14(1) of PD No. 1529 She failed to prove her right under either Section 14(1) or Section 14(2) of PD
No. 1529. She failed to prove that the land she seeks to register was alienable
and disposable land of the public domain. She failed to prove her and her
predecessors-in-interest's possession and occupation since 12 June 1945 or
earlier. Thus, she has no right under Section 14(1) of PD No. 1529. While she
argues that she and her predecessors-in-interest have been in possession and
occupation of the land for 56 years, she failed to prove that the land has been
expressly declared as patrimonial property. Therefore, she also has no right
under Section 14(2) of PD No. 1529.

WHEREFORE, the petition is DENIED. The assailed decision and


resolution of the Court of Appeals are AFFIRMED.

SO ORDERED.

S-ar putea să vă placă și